SlideShare una empresa de Scribd logo
1 de 143
Descargar para leer sin conexión
ANÁLISIS DE CIRCUITOS BIFÁSICOS Y TRIFÁSICOS
Luis E. Iparraguirre Vásquez

R

Estator

N
s
t
Rotor
r

T

S
S

r

ˆ
E
ˆ
ER

ˆ
ES

ˆ
ET

ωt

ˆ
E

2008
Luis E. Iparraguirre Vásquez
Profesor Asociado
de la Escuela de Ingeniería Electrónica
de la Universidad Privada Antenor Orrego
Trujillo - Perú

ANÁLISIS DE CIRCUITOS BIFÁSICOS Y
TRIFÁSICOS

Reservados todos los derechos
Esta obra es propiedad intelectual del autor
Prohibida su reproducción parcial o total por
cualquier medio, sin permiso por escrito del
autor
Análisis de circuitos bifásicos y trifásicos

1

LEIV.
INTRODUCCIÓN
Para el consumo de grandes cantidades de energía eléctrica demanda generación,
transmisión y distribución de la energía eléctrica, que se logra mediante los circuitos
eléctricos trifásicos. El estudio de los circuitos polifásicos constituye un análisis
exhaustivo de los sistemas bifásicos y trifásicos, balanceados o desbalanceados, que se
estudian en el presente texto.
La generación, transmisión y distribución de la energía eléctrica es el gran negocio de
las grandes compañías eléctricas, utilizando diversos combustibles como el carbón, el
gas natural, petróleo para generar energía eléctrica en centrales térmicas; o utilizar la
energía potencial gravitatoria de grandes masas de agua y que por medio de turbinas
acopladas con los ejes de los generadores conforman las centrales hidráulicas.
El voltaje del generador en las centrales, se eleva mediante una transformador para la
transmisión a grandes distancias, debido a que las centrales, generalmente no están
ubicadas ceca de los centros de consumo, y también para tener un mínimo de pérdidas
en las líneas de transmisión.
La energía eléctrica se genera, transmite y distribuye en sistema trifásico, y sólo cuando
esta cerca de los centros de consumo se reduce su tensión a los valores de consumo
( baja tensión), y también se cambia de trifásico a monofásico de acuerdo a lo solicitado
por los usuarios.
Las líneas de transmisión generalmente están conformadas por ternas, siendo soportadas
por torres de madera o metálicos, debidamente aisladas, y en baja tensión puede estar
conformada por tres hilos o cuatro hilos; uno de los cuales está conectado a tierra y es el
de menor calibre.
El caso ideal es que los sistemas polifásicos son diseñados para operar en estado
balanceado o equilibrado, pero en condiciones normales de operación de los circuitos
polifásicos, operan cerca del equilibrio, es decir que las corrientes de cada fase deben
tener los mismos valores eficaces, para que todas las fases del sistema polifásico
distribuyan su potencia aproximadamente iguales.
La estructura básica de un sistema polifásico consiste en fuentes de voltaje conectadas a
cargas a través de transformadores y líneas de transmisión. La omisión del
transformador en el sistema polifásico, simplifica el análisis sin afectar la comprensión
básica de los cálculos implicados.
En el texto se describe el análisis de los circuitos bifásicos y trifásicos de corriente
alterna, los que son conformados por dos y tres fases respectivamente, con tensiones de
la misma frecuencia. Las corrientes alternas fueron adoptadas notablemente por Nikola
Tesla (1856 – 1943), quien obtuvo diez patentes para motores de inducción de corriente
alterna en 1895

Análisis de circuitos bifásicos y trifásicos

2

LEIV.
1
Hz, frecuencia muy alta
3
para el funcionamiento de los motores eléctricos. Gradualmente se estandarizó en
Estados Unidos la frecuencia de 60 Hz, por el cual el ojo humano no puede detectar el
parpadeo que efectúa la corriente alterna senoidal en una lámpara de incandescencia,
pero si es notorio a frecuencias menores. Las frecuencias mayores tienen como efecto
aumento de pérdida de energía en el circuito magnético de las máquinas eléctricas

Los primeros alternadores de Westinghouse fueron de 133

Fundamentalmente la ventaja que presentan los circuitos trifásicos frente a los
monofásicos es que a igualdad de potencia a transmitir y pérdidas en las líneas o
conductores, las líneas trifásicas son mas económicas que los monofásicos, permitiendo
un ahorro del 25 % en el peso de los conductores (líneas).
Otra ventaja es que la potencia instantánea de un sistema trifásico balanceado es
constante, independiente del tiempo, permitiendo que los motores trifásicos tengan un
par uniforme, evitando vibraciones y esfuerzos en el rotor del motor.
Los motores trifásicos pueden arrancar por sí mismos, sin embargo los motores
monofásicos necesitan un dispositivo para conseguir el arranque.

Luis E. Iparraguirre Vásquez

Análisis de circuitos bifásicos y trifásicos

3

LEIV.
CONTENIDO

Sistemas Polifásicos
Generador Bifásico
Tensiones de fase
Tensiones de líneas
Generador trifásico
Conexiones de los arrollamientos de fuerza en un generador trifásico
Conexión estrella ( Y )
• Secuencia de fases directa o positiva
• Secuencia de fases inversa o negativa
Conexión en delta ( ∆ )
• Secuencia de fases directa o positiva
• Secuencia de fases inversa o negativa
Cargas trifásicas
Cargas trifásicas balanceadas
Cargas trifásicas desbalanceadas
Circuitos trifásicos balanceados
Generador trifásico en Y, con carga trifásica balanceada en Y
en secuencia directa
Tensiones de fase del generador
Corrientes de líneas
Potencias aparentes de fases
Potencia aparente total trifásica
Potencia activa total
Potencia reactiva total
Generador trifásico en ∆, con carga trifásica balanceada en ∆
en secuencia directa
Tensiones de fase del generador
Corrientes de fases
Corrientes de líneas
Potencias aparentes de fases
Potencia aparente total
Potencia activa total
Potencia reactiva total
Medida de la potencia activa total absorbida por una carga trifásica
balanceada o desbalanceada
Método de los dos vatímetros
Medida de la potencia trifásica total por medio del método de los
Dos vatímetros monofásicos para el sistema Y balanceado en
secuencia directa
Medida de la potencia trifásica total por medio del método de los
dos vatímetros monofásicos para el sistema ∆ balanceado en
secuencia directa
Determinación del ángulo de la impedancia de la carga trifásica
Balanceada en Y o ∆
Análisis de circuitos bifásicos y trifásicos

4

Pag.
6
6
10
10
11
13
13
16
17
18
19
19
19
19
20
20
20
20
21
22
23
23
23
25
26
26
27
28
28
29
29
31
32

33

36
39
LEIV.
Pag.
Sistema trifásico tetrafilar balanceado en Y en secuencia inversa o negativa 40
Corrientes de líneas
41
Potencias aparentes de fases
42
Potencia aparente total trifásica
43
Potencia activa total
43
Potencia reactiva total
44
Medida de la potencia trifásica total por medio del método de los dos
vatímetros monofásicos para el sistema Y balanceado en secuencia
inversa o negativa
46
Sistema trifásico en ∆ en secuencia inversa o negativa
48
Tensiones de fase del generador
48
Corrientes de fases
49
Corrientes de líneas
50
Potencias aparentes de fases
51
Potencia aparente total
52
Potencia activa total
53
Potencia reactiva total
53
Medida de la potencia activa mediante el método de dos vatímetros
monofásicos
54
Medida de la potencia reactiva en un sistema Y o ∆ balanceado en
secuencia directa
57
Vatímetro en cuadratura en el sistema Y o ∆ balanceado en
Secuencia directa
57
Lectura del vatímetro monofásico en cuadratura para un sistema
trifásico en Y en secuencia directa
58
Lectura del vatímetro monofásico en cuadratura para un sistema
59
trifásico en ∆ en secuencia directa
Problemas resueltos de circuitos trifásicos balanceados
62 - 92
Problemas propuestos trifásicos balanceados
92 - 98
Circuitos trifásicos desbalanceados
99
Circuito trifásico trifilar en Y desbalanceado
99
Corrientes de mallas
99
Corrientes de líneas
99
Potencias aparentes de fases
100
Potencia aparente total
100
Circuito trifásico tetrafilar en Y desbalanceado
101
Corrientes de líneas
101
Corriente en el conductor neutro
101
Potencias aparentes de fases
101
Potencia aparente total
101
Circuito trifásico trifilar en ∆ desbalanceado
102
Corrientes de fases
102
Corrientes de líneas
102
Potencias aparentes de fases
103
Potencia aparente total
103
Problemas de circuitos trifásicos desbalanceados
104 - 137
Problemas propuestos desbalanceados
137 -140
Análisis de circuitos bifásicos y trifásicos

5

LEIV.
SISTEMAS POLIFASICOS
Un sistema polifásico es aquel que tiene dos o más sistemas monofásicos semejantes, en
que los valores máximos de las ondas alternas senoidales no se producen al mismo
tiempo.
El sistema bifásico es aquel que tiene dos sistemas monofásicos sinusoidales con
semejantes valores de amplitud y frecuencia, pero desfasadas entre si en 90 grados
eléctricos
El sistema trifásico es aquel que tiene tres sistemas monofásicos senoidales cuyas
tensiones con los mismos valores de amplitud y frecuencia, se encuentran desfasadas
entre si en 120 grados eléctricos
En instalaciones especiales como los convertidores rotativos, servicios electrolíticos y
otras es conveniente el uso de sistemas de seis, doce o más fases, que emplean tensiones
desfasadas de 60, 30 etc., grados eléctricos
Los equipos que se utilizan en los sistemas polifásicos presentan muchas ventajas con
respecto a los monofásicos. Los generadores polifásicos son de mayores potencias y
mas económicos, presentando mayores eficiencias.
Los motores polifásicos y equipos asociados como los conmutadores, transformadores y
elementos de control, son generalmente de altas potencias para ser usados
industrialmente.
En el sistema monofásico, los motores presentan un par electromagnético no uniforme,
debido a que el flujo de potencia es pulsante, en donde la potencia se anula cuatro veces
por ciclo y se hace negativa en dos intervalos de tiempo de cada ciclo, teniendo el motor
así la velocidad variable en cada instante.
Sin embargo los motores polifásicos operan con un par o torque electromagnético
uniforme, absorbiendo potencia a un ritmo constante y sin inversiones. Estos motores
polifásicos son más eficientes y económicos.

GENERADOR BIFASICO
En un generador bifásico se encuentran dos arrollamientos iguales e independientes o
interconectados en el estator, de forma tal que quedan desplazadas en el espacio en 90º
eléctricos, tal como se muestra en la Fig. 1. En el rotor para este caso de dos polos se
encuentra el arrollamiento de excitación que es alimentado por una fuente de tensión
continua, a través de un reóstato de campo, y que al girar el rotor a una velocidad
angular ω, se inducen en las bobinas de fuerza ( bobinas del estator) tensiones con
desfases de 90 grados eléctricos, tal como se muestra en la Fig.3
En la Fig. 2 se muestra un rotor de cuatro polos

Análisis de circuitos bifásicos y trifásicos

6

LEIV.
Fig. 2

Rotor de cuatro polos
salientes

Fig. 1 Generador Bifásico de dos polos

ˆ
E

ˆ
EA

ˆ
EB

ωt
α

ˆ
−E

Fig. 3

Tensiones inducidas de un generador bifásico en secuencia AB

Así:

ˆ
ˆ
ˆ
EA = EB = E

…( 1 )

En la fase A se induce:

ˆ
e Aa ( t ) = E Sen ωt V

…( 2 )

En la fase B se induce:

ˆ
e Bb ( t ) = E Sen ( ωt − 90 º ) V

…( 3 )

Análisis de circuitos bifásicos y trifásicos

7

LEIV.
El correspondiente diagrama fasorial de la Fig. 3 se muestra en la Fig. 4a y Fig. 4b
a

A

b

E Aa

E Aa

E Bb

B

E Bb

Fig. 4a

Fig. 4b

Estas tensiones tienen una secuencia AB, es decir que e Aa ( t ) ocurre primero y después
de 90º eléctricos ocurre e Bb ( t ) a esta secuencia de fases se le denomina secuencia
positiva o secuencia directa.
También puede darse la secuencia inversa BA o negativa, cuando ocurre primero
e Bb ( t ) , y después de 90º eléctricos ocurre e Aa ( t ) , tal como se muestra en la Fig. 5

ˆ
E

ˆ
EA

ˆ
EB

ωt
α

ˆ
−E

Fig. 5 Tensiones inducidas de un generador bifásico en secuencia BA
Siendo estas tensiones:

ˆ
e Bb ( t ) = E Sen ωt V

ˆ
e Aa ( t ) = E Sen (ωt − 90 º ) V

(4)
(5)

El correspondiente diagrama fasorial de la Fig. 5 se grafica en las Fig. 6a y Fig. 6b
Análisis de circuitos bifásicos y trifásicos

8

LEIV.
E Bb

E Bb

E Aa
E Aa

Fig. 6a

Fig. 6b

Los primeros sistemas polifásicos eran bifásicos o tetrafásicos, siendo completamente
desalojados por los sistemas trifásicos; pero en servicio subsisten cierta cantidad de
sistemas bifásicos, especialmente en los casos de equipos originales de alta calidad o en
los sistemas servomecánicos, donde las máquinas bifásicas tienen gran ventaja.
Un sistema bifásico se caracteriza por un desfase de 90 grados eléctricos entre las
tensiones; las que son generadas en un alternador de dos devanados independientes,
separados en el espacio por el mismo desplazamiento angular que el desplazamiento de
las tensiones en el tiempo
En la Fig. 7(a) , (b), y (c) se muestra un sistema bifásico en secuencia AB o secuencia
directa
Las tensiones de fases son E AN , E BN
En la tensión E AN , significa que el borne “A” es más positivo que el borne “N”
Las tensiones de líneas son E AB , E BA
En la tensión E AB , significa que el borne “A” es más positivo que el borne “B”
En la tensión E BA , significa que el borne “B” es más positivo que el borne “A”
N

a

A

b

E Aa

E Aa = E AN

N

EA

E AB

E Bb = E BN

E Bb

B

Fig. 7

EB

B

(a)

(b)

E BA

(c)

Siendo el valor eficaz de las tensiones de fase E f y el valor eficaz de las tensiones de
línea E l , se tiene:

Análisis de circuitos bifásicos y trifásicos

9

LEIV.
Tensiones de fases
Si: E AN = E f / 0 º V (referencia) en secuencia directa, entonces E BN = E f / − 90 º V

Tensiones de líneas

E AB = E AN − E BN = E f / 0 º − E f / − 90 º , E AB = 2 E f / 45º V

y

E BA = E BN − E AN = E f / − 90 º − E f / 0 º , E BA = 2 E f / − 135º V
En un sistema bifásico

El = 2 Ef
E Aa

E Aa

El circuito de la Fig. 7(a) pertenece a un generador bifásico
trifilar ( tres hilos).
E Bb

El circuito de la Fig. 8 pertenece a un generador

E Bb

bifásico tetrafilar (cuatro hilos) donde cada fase es utilizada
independientemente como dos sistemas monofásicos.

La Fig. 9 muestra un circuito eléctrico que constituye un generador tetrafásico
pentafilar (4 fases - 5 hilos) del que se tiene:

E Aa = E f / 0 º V

E Bb
2

E Aa
2

Ef
E
/ 0 º + f / − 90 º =
2
2
Ef
=
/ − 45º V
2

E Ab =
E Ab
E Aa
2

E Bb
2

Análisis de circuitos bifásicos y trifásicos

E BN =

10

E Bb = E f / − 90 º V

2 Ef
/ − 45º
2

Ef
/ − 90 º V
2

LEIV.
La Fig. 10 que constituye un sistema tetrafásico tetrafilar ( 4 fases - 4 hilos ), del
que se tiene:
E Aa

E QS = E Aa − E Bb
− E Bb

E QS = E f / 0 º − E f / − 90 º

E Bb

E QS = 2 E f / 45º V
− E Aa

GENERADOR TRIFASICO
Un generador trifásico es aquel que tiene tres arrollamientos independientes,
distribuidos en la periferia interna del estator, para los generadores de polos salientes; o
los tres arrollamientos independientes distribuidos en la periferia externa del rotor, para
los generadores de rotor cilíndrico. En la Fig. 11 se muestra un generador trifásico
bipolar, de rotor de polos salientes.

Fig. 11 Generador trifásico de dos polos salientes
Análisis de circuitos bifásicos y trifásicos

11

LEIV.
En estos tres arrollamientos independientes e iguales, se inducen tensiones alternas
senoidales desfasadas entre si en 120 grados eléctricos, debido a que la distribución de
los arrollamientos en la máquina rotativa, también están a 120 grados eléctricos Cada
arrollamiento independiente tiene un par de bornes denotados con las letras R-r, S-s, y
T-t, tal como se muestra en la Fig. 11. El estator y rotor está formado por chapas
laminadas de acero u otras aleaciones siliciosas.
Las tensiones inducidas en los arrollamientos de fuerza según Faraday, es debida a la
acción de una estructura de excitación en corriente continua, la que genera un campo
electromagnético y que al girar concéntricamente a una velocidad angular ωr , y las
espiras de los arrollamientos de fuerza cortan líneas de flujo, generando las tensiones de
las fases R, S, y T, tal como se muestra en la Fig. 12

ˆ
E

ˆ
ER

ˆ
ES

ˆ
ET

ωt

ˆ
E

Fig. 12 Tensiones inducidas en secuencia RST, de un generador trifásico

ˆ
ˆ
ˆ
ˆ
E R = ES = E T = E

(6)

ˆ
e Rr ( t ) = E Sen ωt

(7)

ˆ
e Ss ( t ) = E Sen (ωt − 120 º )

(8)

ˆ
e Tt ( t ) = E Sen ( ωt + 120 º )

Así

(9)

La secuencia de fases RST, STR, TRS, es denominada secuencia directa o positiva, y es
la que hace girar a los motores trifásicos de inducción en el sentido horario.
La secuencia de fases RTS, TSR, SRT, es denominada secuencia inversa o negativa, y
hace girar a los motores de inducción en el sentido antihorario.
Análisis de circuitos bifásicos y trifásicos

12

LEIV.
En la Fig. 13 se muestra el diagrama fasorial de las tensiones inducidas en los
arrollamientos de fuerza, tal que al girar a la velocidad angular ωr rad/s, se generan las
tensiones de la Fig. 12
E Tt

12
0º

120º

E Rr

0º
12

E Ss

Fig 13

Una vez establecida la secuencias de fases, debe determinarse el orden en que deben
conectarse los conductores de línea a la carga.
Un motor de inducción trifásico alimentado en secuencia directa o positiva, gira en un
sentido. Si se intercambian dos de los conductores cualesquiera, entonces se invierte la
secuencia de fases a la secuencia inversa o negativa, y el motor de inducción invierte el
sentido de giro.

CONEXIONES DE LOS ARROLLAMIENTOS DE FUERZA EN UN
GENERADOR TRIFÁSICO
Si cada una de las tres fases independientes del
generador trifásico de la Fig. 14, pueden actuar
independientemente, se tendría un sistema trifásico
exafilar

T

E Tt
t

s

r

R
E Rr

E Ss

En la práctica los generadores trifásicos se pueden
conectar en estrella (Y); o en triángulo o delta (∆)

Conexión del generador en Estrella (Y).

S
Para realizar este tipo de conexión los tres bornes r, s, y
t, se unen entre sí, formando un borne común denominado neutro, denotado con la letra
mayúscula N, tal como se muestra en la Fig. 15 conformando un sistema trifásico trifilar
( 3 fases - 3 hilos ); y Fig. 16 conformando un sistema trifásico tetrafilar ( 3 fases - 4
hilos ); ambas en secuencia de fases directa o positiva

Análisis de circuitos bifásicos y trifásicos

13

LEIV.
T

T

T

T
E TN

E TN

t
t N
r
s

N

r

s

R
E RN

E SN

E SN

R

E RN
N

S

S

S

S

Al invertir la secuencia de fases a inversa o negativa, se tendría las Fig. 17 y Fig. 18
S

S

S

S
E SN

E SN
s
t

N

N

R

r

R
E TN

E RN

E TN

E RN
N

T

T

T
Fig. 18.- Generador trifásico tetrafilar en
secuencia inversa o negativa

T

T

E TR
E TN

E RN
N

R

E RS
E SN
E ST

E TR o E RT

S

Análisis de circuitos bifásicos y trifásicos

Del circuito de la Fig. 15 que
corresponde a un generador trifásico
trifilar en secuencia directa, y que
por no tener el conductor neutro,
sólo se puede obtener tensiones de
líneas, que corresponden a las
tensiones entre las líneas R y S,
obteniendo la tensión E RS o E SR ;
entre las líneas S y T, para obtener la
tensión E ST o E TS ; y entre las
líneas T y R, para obtener la tensión

14

LEIV.
Del circuito de la Fig. 16 que corresponde a un generador trifásico tetrafilar en
secuencia directa, se pueden obtener tanto las tensiones de líneas como las tensiones de
fases.
Una tensión de línea se obtiene entre dos líneas cualesquiera, y una tensión de fase se
obtiene entre una línea y el conductor neutro.
Las tensiones de fase tienen el mismo valor eficaz E RN = E SN = E TN = E f

… (10)

y se encuentran desfasadas entre ellas en 120 grados eléctricos
También se determina que fasorialmente:
E RN = − E NR

…(11)

E SN = − E NS

…(12)

E TN = − E NT

…(13)

De la Fig. 19, las tensiones de líneas son.

E RS = E RN − E SN

…(14)

E ST = E SN − E TN

…(15)

E TR = E TN − E RN

…(16)

De la Fig. 19, se ha obtenido el triángulo SNR que es isósceles y que se muestra en la
Fig. 20 y Fig. 21
E RN

Ef

N

N

R

E RS

R

Ef

E SN

El

S

S

Aplicando ley de senos en el triángulo SNR de la Fig. 21 se tiene:
El
sen 120 º

=

Ef
E
E
, que al dar valores se tiene l = f ; relación de la que se
1
sen 30 º
3
2
2

determina que

El = 3 Ef

…(17).

La ecuación (17) indica que el valor eficaz de la tensión de línea es igual a raíz de tres
veces el valor eficaz de la tensión de fase
Análisis de circuitos bifásicos y trifásicos

15

LEIV.
Si la secuencia de fases es directa o positiva, según circuito de la Fig. 16, con
tensiones de fases:
…(18)
E RN = E f / 0 º V
E SN = E f / − 120 º V …(19)

E TN = E f / 120 º V

…(20)

Reemplazando las ecuaciones del (18) al (20) en las ecuaciones del (14) al (16), se tiene
las ecuaciones:

E RS = E f / 0 º − E f / − 120 º = 3 E f / 30 º V

…(21)

E ST = E f / − 120 º − E f / 120 º = 3 E f / − 90 º V

…(22)

E TR = E f / 120 º − E f / 0 º = 3 E f / 150 º V

…(23)

En la Fig. 22 se muestra el diagrama fasorial de tensiones de fases y tensiones de líneas
obtenidas de un generador trifásico tetrafilar en secuencia directa. Indicando también
que los valores eficaces de tensiones de líneas son iguales a raíz de tres veces el valor
eficaz de tensiones de fases
− E RN

30º

120
º

E TN

E RS

30º

E TR

30º
90º
90º

90º

30º

− E SN

0º
12
E RN

30º
E SN
S

120º

30º E
ST
− E TN

Fig. 22.- Diagrama fasorial de tensiones de fase y tensiones de
líneas en secuencia directa de un generador en Y

NOTA.- El ángulo formado entre una tensión de línea y una tensión de fase es de 30º
eléctricos (ver Fig. 19 y Fig. 22)
Análisis de circuitos bifásicos y trifásicos

16

LEIV.
Si la secuencia de fases es inversa o negativa, según Fig. 18, con tensiones de
fases:
…(24)
E RN = E f / 0 º V
E TN = E f / − 120 º V …(25)

E SN = E f / 120 º V
Las tensiones de líneas son:

…(26)

E RT = E RN − E TN

…(27)

E TS = E TN − E SN

…(28)

E SR = E SN − E RN

…(29)
S

E SR
E SN
S
S

N

E SN
N
R

E TN

E TS

N
T

0º
12

E RN

30º
E RT

R

E TN

E RN

T

120
º

º
30

T
Fig. 23.- Tensiones de fase y de línea de un
generador trifásico tetrafilar en
secuencia inversa o negativa

Reemplazando las ecuaciones del (24) al (26) en las ecuaciones del (27) al (29), se tiene
las ecuaciones:

E RT = E f / 0 º − E f / − 120 º = 3 E f / 30 º V

…(30)

E TS = E f / − 120 º − E f / 120 º = 3 E f / − 90 º V

…(31)

E SR = E f / 120 º − E f / 0 º = 3 E f / 150 º V

…(32)

También se obtiene las tensiones de líneas E RS , E ST y E TR , desfasando las
ecuaciones (32), (31) y (30) en 180 º eléctricos

E RS = − E SR = 3 E f / − 30 º V

…(33)

E ST = − E TS = 3 E f / 90 º V

…(34)

E TR = − E RT = 3 E f / − 150 º V

…(35)

NOTA.- El desfase en 180º eléctricos de un fasor se obtiene permutando los subíndices
correspondientes del fasor en referencia.
Análisis de circuitos bifásicos y trifásicos

17

LEIV.
E ST

− E RN
E RT

E SN

− E TN

E SR

E RN

E TN

E TR

E RS

E TS

− E SN

Conexión del generador en Delta (∆).
Los tres arrollamientos del generador trifásico se conectan en configuración ∆, como
se muestra en la Fig. 25. para secuencia positiva
T

T

E Tt = E TR

R

120º

E Tt

t
s

r

R

E Rr
E Ss
S

E Rr = E RS

0º
12

E Ss = E ST
S

Fig. 25.- Generador en delta en
secuencia de fases directa
Análisis de circuitos bifásicos y trifásicos

12
0º

Fig. 26.- Tensiones de fase y de línea de un generador
trifásico trifilar en , en secuencia directa o positiva

18

LEIV.
El borne “r” se conecta en cortocircuito con el borne “S”. De la misma forma el borne
“s” se conecta en cortocircuito con el borne “T”; y el borne “t” se conecta con el borne
“R” .
El la Fig. 26 se muestra las tensiones de fase en secuencia positiva, y que según en la
conexión ∆, las tensiones de fase son iguales a las tensiones de líneas.
Ef = El
…(36)
En las Fig. 27 y Fig. 28 se muestra la conexión ∆, y las tensiones de fase iguales a las
tensiones de líneas de un generador con tensiones en secuencia inversa o negativa

E Ss = E SR
S

S

E Ss

12
0º
s

t

r

R

E Rr = E RT

0º
12

R

E Rr

E Tt

E Tt = E TS

T

T

Fig. 27.- Generador en delta en
secuencia de fases inversa

CARGAS TRIFÁSICAS
Las cargas trifásicas son conectadas en estrella (Y), o en delta o triángulo (∆), y pueden
ser balanceadas (equilibradas) o desbalanceadas (desequilibradas)

Cargas trifásicas balanceadas.- Son Aquellas cargas constituidas por las tres
impedancias de fases: Z f = Z / ϕº Ω
…(37)
exactamente iguales, como se
muestran en las Fig. 29 y Fig. 30
R

R

Zf

Zf

Zf

Zf
Zf

T

T
S
Fig. 29. Carga trifásica
balanceada en estrella
Análisis de circuitos bifásicos y trifásicos

Zf

S

Fig. 30. Carga trifásica
balanceada en triángulo

19

LEIV.
Cargas trifásicas desbalanceadas.- Son Aquellas cargas constituidas por las tres
impedancias de fase desiguales, según se muestran en la Fig.31 y Fig. 32
R

Z1
Z1

Z2

Z2

Z3

T

S

Z3

Fig. 32. Carga trifásica
desbalanceada en triángulo
Z1 ≠ Z 2 ≠ Z3

Z1 ≠ Z 2 ≠ Z 3

CIRCUITOS TRIFÁSICOS BALANCEADOS
El sistema trifásico balanceado está conformado por un generador trifásico que
generalmente tiene las tensiones de fases balanceadas, el cual alimenta cargas trifásicas
balanceadas por medio de tres líneas (3 hilos) o cuatro líneas (4 hilos)

GENERADOR TRIFÁSICO EN Y, CON CARGA
BALANCEADA EN Y, EN SECUENCIA DIRECTA
IT

T

TRIFÁSICA

PT

T
E TN
t
N

E SN

s

r

Zf

PR

IR

Zf

R

E RN

Zf
N

S

I R + IS + I T

IS

S

Tensiones de fase del generador.
Para la Fig. 33 mostrada, sea:
E RN = E f / 0 º V ( referencia ) secuencia (+)

E SN = E f / − 120 º V

…(39)

E TN = E f / 120 º V

…(38)

…(40)

Análisis de circuitos bifásicos y trifásicos

20

LEIV.
Los potenciales de los bornes “r”, “s” y “t” son los mismos, denotados por “N”
(neutro). Los potenciales de los bornes “R” , “S” y “T” de las líneas y el potencial del
neutro son los mismos tanto en el generador como en la carga. Por lo tanto las tensiones
de fase del generador son también tensiones de fase de la carga.

Corrientes de líneas
Las corrientes de líneas I R , I S , I T , son denotadas con un subíndice, que corresponde al
borne o índice de la línea por donde circula. Estas corrientes de líneas, son también
corrientes de fase, tanto para el generador, como para la carga, debido a que circulan por
las línea, por las fases del generador en “Y” y las fases de la carga en “Y”.
Il = If
…(41)

IR =

IS =

IT =

E RN
Zf
E SN
Zf
E TN
Zf

=

E f / 0º E f
=
/ − ϕº A
Z / ϕº
Zf

…(42)

=

E f / − 120 º E f
=
/ − 120 º − ϕº A
Z / ϕº
Zf

…(43)

=

E f / 120 º E f
=
/ 120 º − ϕº A
Z / ϕº
Zf

…(44)

De las ecuaciones (42), (43) y (44), se observa que para el sistema trifásico
balanceado, los valores eficaces de las corrientes de líneas son de igual magnitud, es
E
decir: I R = I S = I T = I l = f
…(45)
Zf
También se observa que el desfase entre estas corrientes de líneas es de 120º eléctricos
Al sumar las tres corrientes de líneas, para determinar la corriente en el neutro se tiene
…(46) es decir que por conductor neutro no circula
que I R + IS + I T = 0
corriente (circuito abierto), por lo tanto se puede prescindir del conductor neutro.
Pero el conductor neutro, une el neutro del generador con el neutro de la carga, lo que
constituye un corto circuito.
De lo estudiado se concluye:
En el sistema trifásico balanceado, con generador en “Y” y carga en “Y”, las tres
corrientes de líneas son corrientes de fases, con igual magnitud eficaz es decir
E
I R = IS = IT = Il = If = f
…(47), y se encuentran desfasadas entre ellas en 120º
Zf
eléctricos; y cuya suma fasorial es cero
Análisis de circuitos bifásicos y trifásicos

21

LEIV.
En la Fig. 34 se observa el diagrama fasorial de tensiones de fases, tensiones de líneas y
de corrientes de líneas del sistema trifásico tetrafilar balanceado en estudio, cuando la
carga tiene factor de potencia en atraso
E TS

− E RN

30º
E TR

E TN

E RS

30º
IT
30º

º
30 − E SN

ϕº

30º
ϕº
90º
ϕº
30º

IS
E SN

E RN
IR

ϕº > 0

30º
E ST

− E TN

Fig. 34.- Diagrama fasorial de tensiones de fase, tensiones de línea y de
corrientes de línea, de un sistema trifásico tetrafilar balanceado en
secuencia (+), con factor de potencia en atraso

Potencias aparentes de fases
De la Fig. 33 se determina las potencias aparentes absorbidas por cada impedancia de
fase:

E2
Ef
/ ϕº = f / ϕº VA
Zf
Zf

Fase R-N

SRN = E RN I * = E f / 0 º
R

Fase S-N

*
SSN = E SN IS = E f / − 120 º

E2
Ef
/ 120 º + ϕº = f / ϕº VA
Zf
Zf

…(49)

Fase T-N

STN = E TN I * = E f / 120 º
T

E2
Ef
/ − 120 º + ϕº = f / ϕº VA
Zf
Zf

…(50)

Análisis de circuitos bifásicos y trifásicos

22

…(48)

LEIV.
De las ecuaciones (48), (49) y (50), se observa que las potencias absorbidas por cada
E2
impedancia de fase, son exactamente iguales: SRN = SSN = STN = f / ϕº VA …(50)
Zf
Potencia aparente total trifásica
La potencia aparente total entregada por la fuente es la suma de las tres potencias
aparentes absorbidas por cada impedancia de fase. Luego:
E2
Stot = SRN + SSN + STN = 3 x f / ϕº VA
…(51)
Zf
La ecuación (50) puede escribirse: Stot = 3 E f

De la ecuación (17), E f =

El
3

Ef
/ ϕ º VA
Zf

…(52)

…(53)

Ef
= Il
…(54)
Zf
Reemplazando las ecuaciones (53) y (54) en la ecuación (52) se tiene:
De la ecuación (45)

Stot = 3 x

El

x I l / ϕº VA …(55)
3
Efectuando operaciones en el segundo miembro de la ecuación (55) tenemos:
Stot = 3 E l I l / ϕ º VA

…(56)

El módulo de la potencia aparente total es: S tot = 3 E l I l VA …(57)
Pero

Stot = Ptot + J Q tot VA

…(58)

Potencia activa total
De la ecuación (56) la potencia activa total trifásica es:

Ptot = 3 E l I l cos ϕº vatios …(59)
Potencia reactiva total
De la ecuación (56) la potencia reactiva total trifásica es:

Q tot = 3 E l I l senϕº var es …(60)
Análisis de circuitos bifásicos y trifásicos

23

LEIV.
La potencia aparente total absorbida por una carga trifásica balanceada en “Y” son
funciones de los valores eficaces de la tensión y corriente de línea, como se analiza de la
ecuación (57)
La potencia activa total absorbida por una carga trifásica balanceada en “Y” son
funciones de los valores eficaces de tensión y corriente de línea y del factor de potencia
total (el factor de potencia total es igual al factor de potencia de cada impedancia de
fase) como indica la ecuación (59)
La potencia reactiva total absorbida por una carga trifásica balanceada en “Y” son
funciones de los valores eficaces de tensión y corriente de línea y del seno del ángulo de
la impedancia de fase, como indica la ecuación (60)
Los valores eficaces de tensión de línea y corriente de línea no determinan como está
conectado el generador ni la carga trifásica balanceada

E TS

− E RN

30º
E TR

30º

E RS

E TN

º
30 − E SN
IT

30º
30º
IR
IS

E RN

30º
ϕº = 0

E SN

30º
E ST
− E TN
Fig. 35.- Diagrama fasorial de tensiones de fase, tensiones de línea y de
corrientes de línea, de un sistema trifásico tetrafilar balanceado en
secuencia (+), con factor de potencia igual a uno

Análisis de circuitos bifásicos y trifásicos

24

LEIV.
E TS

− E RN
E RS

E TN

30º
E TR

º
30 − E SN

30º
IT

ϕº

30º
ϕº

IR

30º
E RN

ϕº

IS

E SN

ϕº < 0

30º

30º
− E TN

E ST

Fig. 36.- Diagrama fasorial de tensiones de fase, tensiones de línea y de
corrientes de línea, de un sistema trifásico tetrafilar balanceado en
secuencia (+), con factor de potencia en adelanto

GENERADOR TRIFÁSICO EN ∆, CON CARGA
BALANCEADA EN ∆, EN SECUENCIA DIRECTA
T

T

IT

TRIFÁSICA

PT
I TR

E Tt
t

s

r

Zf

I ST

IR

R

Zf

PR

R

E Rr

Zf

E Ss
S
S

Análisis de circuitos bifásicos y trifásicos

I RS

IS

25

LEIV.
Tensiones de fase del generador.

E RS = E f / 0 º V ( referencia ) secuencia (+)
E ST = E f / − 120 º V

…(62)

E TR = E f / 120 º V

…(61)

…(63)

El potencial de borne “r” es mismo que del borne “S”; el potencial del borne “s” es el
mismo que del borne “T”; y el potencial del borne “t” es el mismo que el potencial del
borne “R”, tal como se observa en la Fig.37, quedando el generador conectado en ∆,
Los potenciales “R”, “S” y “T” son los mismos que de la carga en conexión ∆, luego a
cada fase de la carga se a aplicado la tensión de línea, es decir que en conexión del
generador en ∆, las tensiones de fase son también tensiones de líneas
E f = E l , como indica la ecuación (36)

Corrientes de fases
Son aquellas corrientes que circulan por las fases del generador o fases de la carga. Las
corrientes de fases se especifican con doble subíndice como por ejemplo de la Fig. 38 se
dice:
I RS : que circula por la fase de la carga, del borne “R” al borne “S”

IST : que circula por la fase de la carga, del borne “S” al borne “T”
I TR : que circula por la fase de la carga, del borne “T” al borne “R”
Así también se puede determinar la corriente ISR que circula por la fase de la carga, del
borne “S” al borne “R”. Es decir I RS = − I SR …(64)
Cuando se evalúa las corrientes de fase del generador, generalmente es para determinar
las corrientes permisibles que circulan por las bobinas de fuerza del generador, con el
propósito de no deteriorar dichas fases generadoras de energía eléctrica.

I RS =

IST =

I TR =

E RS
Zf
E ST
Zf
E TR
Zf

=

E f / 0º E f
=
/ − ϕº A
Z f / ϕº Z f

=

E f / − 120 º E f
=
/ − 120 º − ϕº A
Z f / ϕº
Zf

=

E f / 120 º E f
=
/ 120 º − ϕº A
Z f / ϕº
Zf

I RS + IST + I TR = 0
Análisis de circuitos bifásicos y trifásicos

…(65)

…(66)

…(67)

…(68)
26

LEIV.
Se observa que en el sistema balanceado en ∆ en secuencia positiva, las tres corrientes
E
de fases tienen el mismo valor eficaz, es decir: I RS = I ST = I TR = I f = f A …(69),
Zf
y se encuentran desfasadas entre ellas en 120’ eléctricos, cuya suma fasorial es cero.

Corrientes de líneas
Son las corrientes que circulan por las líneas que unen el generador con la carga. Estas
corrientes circulan del generador hacia la carga, y se denotan con un solo subíndice
correspondiente al borne de la línea en referencia.
Así el circuito de la Fig. 37, las corrientes de líneas son: I R , I S e I T
Aplicando 1ra Ley de Kirchhoff a los bornes “R”, “S” y “T” de la carga en ∆, se tiene:
En el borne “R” I R = I RS − I TR

…(70)

En el borne “S” IS = I ST − I RS

…(71)

En el borne “T” I T = I TR − IST

…(72)

Reemplazando las corrientes de fases, en los segundos miembros en las ecuaciones (70),
(71) y (72) se tiene:

IR =

Ef
E
E
/ − ϕº − f / 120 º − ϕº = 3 f / − 30 º − ϕº A
Zf
Zf
Zf

…(73)

IS =

Ef
E
E
/ − 120 º − ϕº − f / − ϕº = 3 f / − 150 º − ϕº A
Zf
Zf
Zf

…(74)

IT =

Ef
E
E
/ 120 º − ϕº − f / − 120 º − ϕº = 3 f / 90 º − ϕº A …(75)
Zf
Zf
Zf

Sumando los primeros y segundos miembros de las ecuaciones (70), (71) y (72) se
tiene: I R + IS + I T = 0 A
Según la expresión (69) y las magnitudes eficaces de las corrientes de líneas
determinadas por los segundos miembros de las ecuaciones del (73) al (75), se
concluye: I R = I S = I T = I l = 3 I f A
…(76), y se encuentran desfasadas entre
ellas en 120° eléctricos, siendo la suma fasorial igual a cero

Análisis de circuitos bifásicos y trifásicos

27

LEIV.
Para este circuito el diagrama fasorial de tensiones y corrientes, con factor de potencia
en atraso se muestra en la Fig. 38.
E TR

E TS

− IST I T

(30 º − ϕº )

I TR

ϕº 30 º
60 º

IS

30º
ϕº

− I RS
IST

ϕº
30 º
90 º − ϕº

E RS
I RS

− I TR
IR

ϕº > 0
E ST

Fig. 38.- Diagrama fasorial de tensiones y corrientes de
un sistema trifásico trifilar en , balanceado en
secuencia (+), con factor de potencia en atraso

Potencias aparentes de fases
Determinando las potencias aparentes absorbidas por cada fase de la carga, se tiene:
En la fase R-S SRS = E RS I * = E f / 0 º
RS

E2
Ef
/ ϕº = f / ϕº VA
Zf
Zf

…(77)

*
En la fase S-T SST = E ST I ST = E f / − 120 º

E2
Ef
/ 120 º + ϕº = f / ϕº VA
Zf
Zf

En la fase T-R STR = E TR I * = E f / 120 º
TR

E2
Ef
/ − 120 º + ϕº = f / ϕº VA
Zf
Zf

…(78)

…(79)

Se observa que en el sistema balanceado en ∆, las potencias aparentes por fase son
E2
iguales, es decir SRS = SST = STR = Sf = f / ϕº VA
…(80)
Zf

Potencia aparente total
La potencia aparente total absorbida por la carga es igual a la suma fasorial de las tres
potencias aparentes de cada fase
Análisis de circuitos bifásicos y trifásicos

28

LEIV.
Stot = SRS + SST + STR = 3

2
Ef

Zf

La ecuación (81) puede escribirse: Stot = 3 E f
De la ecuación (36), E f = E l

/ ϕ º VA

Ef
/ ϕ º VA
Zf

…(81)

…(82)

…(83)

Ef
= If
…(84)
Zf
Reemplazando las ecuaciones (83) y (84) en la ecuación (82) se tiene:
De la ecuación (69)

Stot = 3 x E l x I f / ϕº VA

…(85)

Según la ecuación (76) se deduce: I f =

Il

A …(86)
3
Reemplazando (86) en (85) y efectuando operaciones en el segundo miembro de la
ecuación (85) tenemos:
Stot = 3 E l I l / ϕ º VA

…(87)

El módulo de la potencia aparente total es: S tot = 3 E l I l VA …(88)
Pero

Stot = Ptot + J Q tot VA

…(89)

Potencia activa total
De la ecuación (56) la potencia activa total trifásica es:

Ptot = 3 E l I l cos ϕº vatios …(90)
Potencia reactiva total
De la ecuación (56) la potencia reactiva total trifásica es:

Q tot = 3 E l I l senϕº var es …(91)

Análisis de circuitos bifásicos y trifásicos

29

LEIV.
A continuación se muestran en la Fig. 39 y Fig. 40, los diagramas fasoriales de
tensiones y corrientes de un sistema trifásico trifilar en ∆, con factor de potencia
unitario y factor de potencia en adelanto respectivamente
E TR

E TS

IT

− IST

I TR

30 º
I RS

60 º

IS

E RS

30 º

30º

− I TR

IST

IR

− I RS
ϕº = 0

E ST

Fig. 39.- Diagrama fasorial de tensiones y corrientes de
un sistema trifásico trifilar en , balanceado en
secuencia (+), con factor de potencia unitario
E TR

E TS

60 º

− IST

IT

ϕº

30 º − ϕº

I TR 30 º

I RS
30 º

ϕº

− I TR

E RS

IR

IST

IS

− I RS

ϕº

ϕº < 0

E ST

Análisis de circuitos bifásicos y trifásicos

30

LEIV.
La potencia aparente total depende únicamente de los valores de tensión y corriente
de línea, según se observa de la ecuación (88)
La potencia activa total absorbida por una carga trifásica balanceada en “∆” en
secuencia directa, son funciones de los valores eficaces de tensión y corriente de línea y
del factor de potencia total (el factor de potencia total es igual al factor de potencia de
cada impedancia de fase), como se observa de la ecuación (90)
La potencia reactiva total absorbida por una carga trifásica balanceada en “∆” en
secuencia directa, son funciones de los valores eficaces de tensión y corriente de línea y
del seno del ángulo de la impedancia de fase, deducido de la ecuación (91)
Los valores eficaces de tensión de línea y corriente de línea no determinan como está
conectado el generador ni la carga trifásica balanceada

MEDIDA DE LA POTENCIA ACTIVA TOTAL ABSORBIDA POR UNA
CARGA TRIFÁSICA BALANCEADA, O DESBALANCEADA
La medición de la potencia activa total trifásica en un sistema polifásico es posible,
instalando tantos vatímetros monofásicos como fases tiene el sistema
Para en sistema trifásico se utilizarían tres vatímetros monofásicos conectados en cada
fase; de manera tal que cada vatímetro monofásico mida la potencia activa de fase, esto
es que por cada bobina amperimétrica ingrese la corriente de fase y a la bobina
voltimétrica de cada vatímetro monofásico aplicar la tensión de fase (es decir entre línea
y neutro para la carga en estrella; o entre línea y línea para la carga en delta), tal como
se muestra en la Fig. 41 y Fig. 42, para una carga en Y o en ∆ respectivamente
R
PZ

3

PT
Z1 ≠ Z 2 ≠ Z3

IT
Z3

T
Z1

PZ

Z2

IS

PS
IR

Análisis de circuitos bifásicos y trifásicos

Z1

Z3

2

Z2

PZ

1

S
Z1 ≠ Z 2 ≠ Z3

PR

Fig. 42. Medida de la potencia trifásica
de una carga trifásica en triángulo,
mediante tres vatímetros monofásicos

31

LEIV.
El método de los tres vatímetros es adecuado para la medición de la potencia activa de
un sistema trifásico desbalanceado, generalmente si las tensiones entregadas por el
generador y el factor de potencia de la carga, varían constantemente, y la potencia total
es la suma algebraica de las tres lecturas indicadas de cada vatímetro
Así para la Fig. 41 la potencia total es: Ptot = PR + PS + PT
Y para la Fig. 42 la potencia total es: Ptot = PZ + PZ + PZ
1

2

…(92)
…(93)
3

Método de los dos vatímetros
Para la medición de la potencia trifásica, el método de los dos vatímetros es el que
comúnmente se utiliza. Para medir la potencia trifásica total por este método, la carga
puede estar conectado en estrella ( Y ) o delta ( ∆ ), y puede ser balanceada o
desbalanceada
Los dos vatímetros se deben conectar adecuadamente dos líneas, tal como se muestra en
la Fig. 43.
Cada vatímetro se conecta en dos líneas cualesquiera, de manera tal que en cada bobina
amperimétrica ingrese la corriente de línea; y a cada bobina voltimétrica se conecte la
tensión de línea determinada por la línea donde se ha conectado el vatímetro y la otra
línea donde no se ha conectado un vatímetro.
Si bien los vatímetros individuales ya no registran la potencia absorbida por cualquier
fase particular de la carga, la suma algebraica de las lecturas de los dos vatímetros es
igual a la potencia activa total tomada por las carga trifásica, sin que importe las
conexiones, tanto de generador como de la carga (estrella o delta).

T

R

S

IT

IR

PT

C arg a
trifásica
en Y o ∆

PR

balanceada
o
desbalanceada
IS

Análisis de circuitos bifásicos y trifásicos

32

LEIV.
Medida de potencia trifásica total por medio del método de los dos vatímetros
monofásicos para el sistema Y balanceado en secuencia directa
Analizaremos el circuito eléctrico ya estudiado de la Fig. 33, con Generador en “Y” y
carga en “Y”con su diagrama fasorial de tensiones y corrientes es el mostrado en la
Fig. 34
IT

T

PT

T

E TN
N

s

r

Zf

PR

IR

t

Zf

R

E RN

E SN

Zf
N

S

I R + IS + I T

IS

S

E TS

− E RN

30º
E TR

S

30º

E TN

E RS

º
30 − E SN

IT

30º

ϕº

30º
ϕº
IS

90º
ϕº
30º

E RN
IR

E SN

30º
− E TN

E ST

Fig. 34.- Diagrama fasorial de tensiones de fase, tensiones de línea y de
corrientes de línea, de un sistema trifásico tetrafilar balanceado en
secuencia (+), con factor de potencia en atraso
Análisis de circuitos bifásicos y trifásicos

33

LEIV.
Si :
Si :
Si :

0 º < ϕº < 90 º , el factor de potencia es en atraso
0 º = ϕº , el factor de potencia es unitario (máximo)
− 90 º < ϕº < 0 º , el factor de potencia es en adelanto

Para el circuito de la Fig. 33, las lecturas son:
E RS
IR

…(94)

E TS
IT

…(95)

PR = E RS I R cos 〈
PT = E TS I T cos 〈
Siendo E RS = E TS = E l
IR = IT = Il

…(96)
…(97)

De la Fig. 34 el ángulo formado entre E RS e I R es:
〈

y el ángulo formado entre E TS e I T es:

E TS
IT

〈

E RS
IR

= 30 º + ϕ º

= 30 º − ϕº

…(98)

…(99)

Reemplazando las ecuaciones (96), (97) y (98) en la ecuación (94) se obtiene en
valores de líneas PR = E l I l cos (30 º + ϕº ) vatios

…(100)

Reemplazando las ecuaciones (96), (97) y (99) en la ecuación (95) se obtiene en
valores de líneas PT = E l I l cos (30 º − ϕº ) vatios

…(101)

Sumando los primeros miembros de las ecuaciones (100) y (101) se tiene:
PR + PT = E l I l (cos (30 º + ϕ º ) + cos(30 º −ϕ º ) ) vatios

…(102)

Efectuando operaciones en el segundo miembro de la ecuación (102) se tiene:
PR + PT = E l I l 2 cos 30 º cos ϕ º vatios
siendo cos 30 º =

…(103)

3
…(104), que reemplazado en (103) se tiene:
2

PR + PT = 3 E l I l cos ϕº vatios
Análisis de circuitos bifásicos y trifásicos

…(104)

34

LEIV.
El segundo miembro de la expresión de la ecuación (104), es idéntica a la expresión del
segundo miembro de la ecuación (59), e indica que los dos vatímetros monofásicos dan
valores de lecturas, cuya suma es igual a la potencia activa total absorbida por la carga
trifásica balanceada en Y
La potencia total activa es función de los valores eficaces de tensión de línea, corriente
de línea y del factor de potencia total (igual al factor de potencia de cada impedancia de
fase)
En secuencia (+), los vatímetros se han colocado en las líneas “R” y “T” obteniendo
las lecturas:
PR = E l I l cos (30 º + ϕº ) vatios

…(100)

PT = E l I l cos (30 º − ϕº ) vatios

…(101)

PR + PT = 3 E l I l cos ϕº vatios

…(104)

Graficando las ecuaciones (100) y (101) y (104) se obtiene la Fig. 44, de la que se
deduce:
P
El Il

Ptot
El Il

3

PT
El Il

3/2
75º

− 75º

− ϕº
− 90 º

− 60 º − 45º − 30 º − 15º

15º 30 º

45º

60º

ϕº
90 º

PR
El Il

Fig. 44.- Gráfica de PR , PT , Ptot en secuencia directa, de un sistema trifásico balanceado en Y

1º. Para factor de potencia total en atraso, en secuencia directa R S T R S T, se ha
conectado dos vatímetros:
*

*

a. Un vatímetro conectado en la línea R y el otro en la línea T R S T R S T ,
quedando la línea “S” centrada entre las dos líneas donde se han conectado los
vatímetros
Así: PT ( vatímetro que se encuentra a la derecha ) tiene mayor lectura que PR
(vatímetro que se encuentra a la izquierda) es decir PT > PR

Análisis de circuitos bifásicos y trifásicos

35

LEIV.
b. Si se hubiera conectado los vatímetros en las líneas “S” y “T”, para factor de
*

*

potencia en atraso y en secuencia directa R S T R S T , la lectura PS > PT

c. Si se hubiera conectado los vatímetros en las líneas “R” y “S”, para factor de
*

*

potencia en atraso y en secuencia directa R S T R S T , la lectura PR > PS

2º. Si el factor de potencia total es unitario, las lecturas son iguales. PR = PT
3º. Para factor de potencia total en adelanto, en secuencia (+), se conecta dos
vatímetros:
*

*

a. Con los vatímetros en las líneas “R” y “T” R S T R S T , vatímetro conectado
en la línea “R” ( vatímetro que se encuentra a la izquierda) tiene mayor lectura
que PT (vatímetro que se encuentra a la derecha) es decir PR > PT
b. Si se hubiera conectado los vatímetros en las líneas “S” y “T”, para factor de
*

*

potencia en adelanto y en secuencia directa R S T R S T , la lectura PT > PS
c. Si se hubiera conectado los vatímetros en las líneas “R” y “S”, para factor de
*

*

potencia en adelanto y en secuencia directa R S T R S T , la lectura PS > PR

4º. En las ecuaciones (100) y (101), al cambiar ϕº por − ϕ º , se permutan las lecturas
Medida de potencia trifásica total por medio del método de los dos vatímetros
monofásicos para el sistema ∆ balanceado en secuencia directa
Analizaremos el circuito eléctrico ya estudiado de la Fig. 37, con Generador en “∆” y
carga en “∆”cuyo diagrama fasorial de tensiones y corrientes es el mostrado en la Fig.
38
T

T

IT

PT

I TR

E Tt

t

s

r

E Rr

Zf

E Ss

S

Zf

I ST

IR

R

Zf

PR

R

S

Análisis de circuitos bifásicos y trifásicos

I RS

IS

36

LEIV.
E TS

E TR
− IST I T

(30 º − ϕº )

I TR
ϕº 30 º

60 º

IS

30º
ϕº

− I RS
IST

ϕº
30 º
90 º − ϕº

E RS
I RS

− I TR

IR
ϕº > 0

E ST

Fig. 38.- Diagrama fasorial de tensiones y corrientes de
un sistema trifásico trifilar en , balanceado en
secuencia (+), con factor de potencia en atraso

Si :
Si :
Si :

0 º < ϕº < 90 º , el factor de potencia es en atraso
0 º = ϕº , el factor de potencia es unitario (máximo)
− 90 º < ϕº < 0 º , el factor de potencia es en adelanto

Para el circuito eléctrico de la Fig. 33, las lecturas son:
E RS
IR

…(105)

E TS
IT

…(106)

PR = E RS I R cos 〈
PT = E TS I T cos 〈
Siendo E RS = E TS = E l
IR = IT = Il

…(107)
…(108)

De la Fig. 38 el ángulo formado entre E RS e I R es:
〈

y el ángulo formado entre E TS e I T es:

E TS
IT

〈

E RS
IR

= 30 º − ϕº

= 30 º + ϕ º

…(109)

…(110)

Reemplazando las ecuaciones (107), (108) y (109) en la ecuación (105) se obtiene
en valores de líneas PR = E l I l cos (30 º + ϕº ) vatios
Análisis de circuitos bifásicos y trifásicos

37

…(111)
LEIV.
Reemplazando las ecuaciones (107), (108) y (110) en la ecuación (106) se obtiene
en
valores de líneas PT = E l I l cos (30 º − ϕº ) vatios

…(112)

Sumando los primeros miembros de las ecuaciones (111) y (112) se tiene:
PR + PT = E l I l (cos (30 º + ϕ º ) + cos(30 º −ϕ º ) ) vatios

…(113)

Efectuando operaciones en el segundo miembro de la ecuación (113) se tiene:
PR + PT = E l I l 2 cos 30 º cos ϕ º vatios
siendo cos 30 º =

…(114)

3
…(115), que reemplazado en (114) se tiene:
2

PR + PT = 3 E l I l cos ϕº vatios

…(116)

El segundo miembro de la ecuación (116), es idéntica al segundo miembro de la
ecuación (90), e indica que los dos vatímetros monofásicos dan valores de lecturas,
cuya suma es igual a la potencia activa total absorbida por la carga trifásica balanceada
en ∆.
La potencia total activa es función de los valores eficaces de tensión de línea, corriente
de línea y del factor de potencia total (igual al factor de potencia de cada impedancia de
fase)
En secuencia (+), los vatímetros se han colocado en las líneas “R” y “T” obteniendo
las lecturas:
PR = E l I l cos (30 º + ϕº ) vatios

…(111)

PT = E l I l cos (30 º − ϕº ) vatios

…(112)

PR + PT = 3 E l I l cos ϕº vatios

…(116)

Los valores de potencia indicados por las ecuaciones (111) , (112) , y (116), son
observados en la Fig. 45 (Exactamente igual a la Fig. 44)

Análisis de circuitos bifásicos y trifásicos

38

LEIV.
P
El Il
Ptot

3

El Il
PT
El Il

3/2
75º

− 75º

− ϕº
− 90 º

− 60 º − 45º − 30 º − 15º

15º 30 º

45º

60 º

ϕº
90 º

PR
El Il

Para el sistema Y o ∆ trifásico balanceado en secuencia directa:
La ecuación (100) es idéntica a la ecuación (111)
La ecuación (101) es idéntica a la ecuación (112)
La ecuación (104) es idéntica a la ecuación (116)
De lo que se concluye que la potencia indicada por cada vatímetro monofásico es
independiente de la conexión del generador y de la conexión de la carga, y que la suma
algebraica de las lecturas de cada vatímetro monofásico es la potencia activa total
absorbida por la carga trifásica.
La potencia activa total siempre positivo, sin embargo las lecturas de los vatímetros
peden tomar valores positivos y negativos, dependiendo de la secuencia de fases y del
factor de potencia de la carga trifásica balanceada.

Determinación del ángulo de impedancia de la carga trifásica balanceada en Y o ∆
Las lecturas de cada vatímetro monofásico de un circuito trifásico balanceado dependen
del factor de potencia, tal como las ecuaciones:
PR = E l I l cos (30 º + ϕº ) vatios

…(100) ó …(111)

PT = E l I l cos (30 º − ϕº ) vatios

…(101) ó …(112)

es lógico que estas ecuaciones nos permitan determinar una fórmula que determine el
ángulo de la impedancia de la carga trifásica balanceada.
Obteniendo la diferencia de lecturas de los vatímetros monofásicos
PT − PR = E l I l  cos(30 º − ϕ º ) − cos(30 º + ϕº ) 




Análisis de circuitos bifásicos y trifásicos

39

…(117)

LEIV.
Efectuando operaciones en el segundo miembro de la ecuación (117) se tiene:
PT − PR = E l I l  2 sen 30 º senϕº 




PT − PR = E l I l senϕº
Pero

siendo sen 30º = 1 / 2

…(118)

PR + PT = 3 E l I l cos ϕº vatios

…(104) ó …(116)

Dividiendo miembro a miembro las ecuaciones (118) y (104) se tiene:
PT − PR
=
PR + PT

3 E l I l senϕº
3 E l I l cos ϕ º

 P − PR 
de donde tgϕº = 3  T

 PT + PR 



…(119)

…(120)

En la aplicación de la ecuación (120) para evaluar tg ϕº es necesario aplicar los signos
correctos a los valores de PT y PR

SISTEMA TRIFASICO TETRAFILAR BALANCEADO EN Y, EN SECUENCIA
INVERSA O NEGATIVA
IS

S
S
E SN
s
N

t

r

Zf

PR

IR

Zf

R

E RN

Zf

E TN
N
T
T

I R + IS + I T

IT

PT

En la Fig. 46 se muestra el circuito eléctrico a estudiar, con los dos vatímetros
monofásicos conectados en las líneas “R” y “T”
Las tensiones de fase del generador en secuencia inversa son:
Análisis de circuitos bifásicos y trifásicos

40

LEIV.
E RN = E f / 0 º V

…( 121 )

E SN = E f / 120 º V

…(122)

E TN = E f / − 120 º V

…(123)

( referencia a 0º ) secuencia (-)

Corrientes de líneas
Las corrientes de líneas I R , IS , I T e son denotadas con un subíndice, que corresponde
al borne o índice de la línea por donde circula. Estas corrientes de líneas, son también
corrientes de fase, tanto para el generador, como para la carga, debido a que circulan por
las línea, por las fases del generador en “Y” y las fases de la carga en “Y”.
Il = If
…(41)
IR =

IS =

IT =

E RN
Zf
E SN
Zf
E TN
Zf

=

E f / 0º E f
=
/ − ϕº A
Z / ϕº
Zf

…(124)

=

E f / 120 º E f
=
/ 120 º − ϕº A
Z / ϕº
Zf

…(125)

=

E f / − 120 º E f
=
/ − 120 º − ϕº A
Z / ϕº
Zf

…(126)

De las ecuaciones (124), (125) y (126), se observa que para el sistema trifásico
balanceado, los valores eficaces de las corrientes de líneas son de igual magnitud, es
E
…(127)
decir: I R = I S = I T = I l = I f = f
Zf
También se observa que el desfase entre las corrientes de líneas es de 120º eléctricos
Al sumar las tres corrientes de líneas, para determinar la corriente en el neutro se tiene
que I R + IS + I T = 0
…(128) es decir que por conductor neutro no circula
corriente ( característica de un circuito abierto), por lo tanto se puede prescindir del
conductor neutro.
El conductor neutro, une el neutro del generador con el neutro de la carga, lo que
constituye un corto circuito.
De lo estudiado se concluye:
En el sistema trifásico balanceado, con generador en “Y” y carga en “Y”, las tres
corrientes de líneas son corrientes de fases, con igual magnitud eficaz es decir
E
I R = IS = IT = Il = I f = f
…(127), y se encuentran desfasadas entre ellas en
Zf
120º eléctricos; y cuya suma fasorial es cero
Análisis de circuitos bifásicos y trifásicos

41

LEIV.
En la Fig. 47 se observa el diagrama fasorial de tensiones de fases, tensiones de líneas y
de corrientes de líneas del sistema trifásico tetrafilar balanceado en secuencia negativa o
inversa, cuando la carga tiene factor de potencia en atraso

− E TN
E ST
30º

E RT

E SR
E SN

ϕº I
S

30º
ϕº > 0

IT

ϕº

30º
90

ϕº
E TR

E RN

30º
30º
º −ϕ

º

IR

30 − E SN
º

30º
30º
− E RN

E RS

E TN

E TS

Fig. 47.- Diagrama fasorial de tensiones de fase, tensiones de línea y de
corrientes de línea de un sistema trifásico tetrafilar en secuencia (-),
con factor de potencia en atraso

Potencias aparentes de fases
De la Fig. 47 se determina las potencias aparentes absorbidas por cada impedancia de
fase:

E2
Ef
/ ϕº = f / ϕº VA
Zf
Zf

Fase R-N

SRN = E RN I * = E f / 0 º
R

Fase S-N

*
SSN = E SN IS = E f / 120 º

Fase T-N

STN = E TN I * = E f / − 120 º
T

Análisis de circuitos bifásicos y trifásicos

…(129)

E2
Ef
/ − 120 º + ϕº = f / ϕº VA …(130)
Zf
Zf
E2
Ef
/ 120 º + ϕº = f / ϕº VA …(131)
Zf
Zf
42

LEIV.
De las ecuaciones (129), (130) y (131), se observa que las potencias absorbidas por
cada impedancia de fase, son exactamente iguales:

SRN = SSN = STN =

2
Ef

Zf

/ ϕº VA

…(132)

Potencia aparente total trifásica
La potencia aparente total entregada por la fuente es la suma de las tres potencias
aparentes absorbidas por cada impedancia de fase. Luego:
Stot = SRN + SSN + STN = 3 x

2
Ef

/ ϕº VA

…(133)

La ecuación (133) puede escribirse: Stot = 3 E f

Ef
/ ϕ º VA
Zf

Para el sistema estrella, E f =

El
3

Zf

…(134)

…(135)

Ef
= Il
…(136)
Zf
Reemplazando las ecuaciones (135) y (136) en la ecuación (134) se tiene:
De la ecuación (127)

Stot = 3 x

El

x I l / ϕº VA …(137)
3
Efectuando operaciones en el segundo miembro de la ecuación (137) tenemos:
Stot = 3 E l I l / ϕ º VA

…(138)

El módulo de la potencia aparente total es: S tot = 3 E l I l VA …(139)
Pero

Stot = Ptot + J Q tot VA

…(140)

Potencia activa total
De la ecuación (140) la potencia activa total trifásica es:

Ptot = 3 E l I l cos ϕº vatios …(141)

Análisis de circuitos bifásicos y trifásicos

43

LEIV.
Potencia reactiva total
De la ecuación (138) la potencia reactiva total trifásica es:

Q tot = 3 E l I l senϕº var es …(142)

− E TN

E ST

E RT

E SR
E SN

E RN

E TR

30 − E SN
º

E TN

ϕº = 0º

E RS

− E RN

E TS

La potencia aparente total absorbida por una carga trifásica balanceada en “Y” son
funciones de los valores eficaces de la tensión y corriente de línea, como se analiza de la
ecuación (139)
La potencia activa total absorbida por una carga trifásica balanceada en “Y” son
funciones de los valores eficaces de tensión y corriente de línea y del factor de potencia
total (el factor de potencia total es igual al factor de potencia de cada impedancia de
fase) como indica la ecuación (141)

Análisis de circuitos bifásicos y trifásicos

44

LEIV.
La potencia reactiva total absorbida por una carga trifásica balanceada en “Y” son
funciones de los valores eficaces de tensión y corriente de línea y del seno del ángulo de
la impedancia de fase, como indica la ecuación (142)
Los valores eficaces de tensión de línea y corriente de línea no determinan como está
conectado el generador ni la carga trifásica balanceada

En las Fig. 48 y Fig. 49 se muestran los diagramas fasoriales de tensiones y
corrientes en secuencia inversa, de los sistemas trifásicos tetrafilares en Y, con factor de
potencia unitario y factor de potencia en adelanto respectivamente

− E TN

E ST

E RT

E SR

E SN

ϕº

ϕº
E RN
ϕº

E TR

30 − E SN
º

E TN

ϕº < 0 º

E RS

− E RN

E TS

Análisis de circuitos bifásicos y trifásicos

45

LEIV.
Medida de potencia trifásica total por medio del método de los dos vatímetros
monofásicos para el sistema Y balanceado en secuencia inversa o negativa
Según el circuito eléctrico de la Fig. 46, los vatímetros están conectados en las líneas R
y T, luego las lecturas de los vatímetros serán:
E RS
IR

…(143)

E TS
IT

…(144)

PR = E RS I R cos〈
PT = E TS I T cos〈
La tensiones

E RS = E TS = E l

Las corrientes I R = I T = I l

…(145)
…(146)

− E TN

30º

E ST

E RT

E SR
E SN

ϕº I
S

30º
ϕº > 0

IT

9

− E RN

ϕ
0º −

º

IR

30 − E SN
º

30º
30º

ϕº

30º

ϕº
E TR

E RN

30º
30º

E RS

E TN

E TS

Fig. 47.- Diagrama fasorial de tensiones de fase, tensiones de línea y de
corrientes de línea de un sistema trifásico tetrafilar en secuencia (-),
con factor de potencia en atraso

Del diagrama de la Fig. 47 se obtienen los ángulos entre tensiones y corrientes, así
Análisis de circuitos bifásicos y trifásicos

46

LEIV.
〈

E RS
IR

= 30 º − ϕº

〈

…(147)

E TS
IT

= 30 º + ϕº

…(148)

Reemplazando las ecuaciones (145), (146) y (147) en la ecuación (143) se tiene:
PR = E l I l cos(30 º − ϕº ) vatios

…(149)

Reemplazando las ecuaciones (145), (146) y (148) en la ecuación (144) se tiene
PT = E l I l cos(30 º + ϕº ) vatios

…(150)

Sumando miembro a miembro las ecuaciones (149) y (150) se tiene:
…(151)
PR + PT = E l I l  cos(30º − ϕ º ) + cos(30 º + ϕº )  vatios




Efectuando operaciones en el segundo miembro de la ecuación (151) se tiene:
PR + PT = 3 E l I l cos ϕ º vatios

…(152)

El segundo miembro de la expresión de la ecuación (152), es idéntica a la expresión del
segundo miembro de la ecuación (141), e indica que los dos vatímetros monofásicos dan
valores de lecturas, cuya suma es igual a la potencia activa total absorbida por la carga
trifásica balanceada en Y
La potencia total activa es función de los valores eficaces de tensión de línea, corriente
de línea y del factor de potencia total (igual al factor de potencia de cada impedancia de
fase)
Graficando las potencias de las ecuaciones (149), (150) y (152) se obtiene la Fig. 50, de
la que se deduce:
Ptot
El Il

Ptot
El Il

3

PR
El Il

3/2

75º

− 75º

− ϕº
− 90º

− 60º − 45º − 30 º − 15º

Análisis de circuitos bifásicos y trifásicos

15º 30 º

47

45º

60 º

ϕº

90 º

PT
El Il

LEIV.
1º. Para factor de potencia total en atraso, en secuencia inversa R T S R T S, se han
conectado dos vatímetros:
*

*

a. Un vatímetro conectado en la línea R y el otro en la línea T R T S R T S ,
quedando la línea “S” centrada entre las dos líneas donde se han conectado los
vatímetros
Así: para factor de potencia en atraso y en secuencia inversa, PR ( vatímetro
que se encuentra a la derecha ) tiene mayor lectura que PT (vatímetro que se
encuentra a la izquierda) es decir PR > PT

b. Si se hubiera conectado los vatímetros en las líneas “S” y “T”, para factor de
*

*

potencia en atraso y en secuencia inversa R T S R T S , la lectura PT > PS
c. Si se hubiera conectado los vatímetros en las líneas “R” y “S”, para factor de
*

*

potencia en atraso y en secuencia inversa R T S R T S , la lectura PS > PR
2º. Si el factor de potencia total es unitario, las lecturas son iguales. PR = PT

3º. Para factor de potencia total en adelanto, en secuencia (-), se conecta dos
vatímetros:
*

*

a. Con los vatímetros en las líneas “R” y “T” R T S R T S , vatímetro
conectado en la línea “T” ( vatímetro que se encuentra a la izquierda) tiene
mayor lectura que el vatímetro conectado en la línea R (vatímetro que se
encuentra a la derecha) es decir PT > PR
b. Si se hubiera conectado los vatímetros en las líneas “S” y “T”, para factor de
*

*

potencia en adelanto y en secuencia inversa R T S R T S , la lectura PS > PT
c. Si se hubiera conectado los vatímetros en las líneas “R” y “S”, para factor de
*

*

potencia en adelanto y en secuencia inversa R T S R T S , la lectura PR > PS
4º. En las ecuaciones (149) y (150), al cambiar ϕº por − ϕ º , se permutan las
lecturas

SISTEMA TRIFÁSICO EN ∆ EN SECUENCIA INVERSA O NEGATIVA
Tensiones de fase del generador.
E RT = E f / 0 º V ( referencia ) secuencia (-)

E TS = E f / − 120 º V

…(154)

E SR = E f / 120 º V

…(153)

…(155)

Análisis de circuitos bifásicos y trifásicos

48

LEIV.
S

S

IS

ISR

E Ss = E SR
s

t

r

E Tt = E TS

Zf

PR

R

I TS

IR

R

Zf

E Rr = E RT

Zf

I RT
T

PT

IT

T

El potencial de borne “r” es mismo que del borne “T”; el potencial del borne “s” es el
mismo que del borne “R”; y el potencial del borne “t” es el mismo que el potencial del
borne “S”, tal como se observa en la Fig.51, quedando el generador conectado en ∆, en
secuencia inversa
Los potenciales “R”, “S” y “T” son los mismos que de la carga en conexión ∆, luego a
cada fase de la carga se a aplicado la tensión de línea, es decir que en conexión del
generador en ∆, las tensiones de fase son también tensiones de líneas
E RS = E ST = E TR = E f = E l
…(156)

E RS = E f / − 60 º V

…(157)

E ST = E f / 60 º V

…(158)

E TR = E f / 180 º V

…(159)

Corrientes de fase
Son aquellas corrientes que circulan por las fases del generador o fases de la carga. Las
corrientes de fases se especifican con doble subíndice como por ejemplo de la Fig. 51 se
dice:

ISR : Corriente que circula por la fase de la carga, del borne “S” al borne “R”
I TS : Corriente que circula por la fase de la carga, del borne “T” al borne “S”

I RT : Corriente que circula por la fase de la carga, del borne “R” al borne “T”
Así también se puede determinar la corriente I RS que circula por la fase de la carga, del
borne “R” al borne “S”. Es decir I RS = − ISR …(160)
Cuando se evalúa las corrientes de fase del generador, es generalmente para determinar
las corrientes permisibles que circulan por las bobinas de fuerza del generador, con el
propósito de no deteriorar dichas fases generadoras de energía eléctrica.
Análisis de circuitos bifásicos y trifásicos

49

LEIV.
ISR =

E SR
Zf
E TS

I TS =

Zf

I RT =

E RT
Zf

=

E f / 120 º E f
=
/ 120 − ϕº A …(161)
Z f / ϕº
Zf

=

E f / − 120 º E f
=
/ − 120 º − ϕº A
Z f / ϕº
Zf

=

E f / 0º E f
=
/ − ϕº A
Z f / ϕº Z f

I RS + IST + I TR = 0

…(162)

…(163)

…(164)

Se observa que en el sistema balanceado en ∆ en secuencia inversa, las tres corrientes de
E
fases tienen el mismo valor eficaz, es decir: I SR = I TS = I RT = I f = f A
…(165),
Zf
y se encuentran desfasadas entre ellas en 120º eléctricos, cuya suma fasorial es cero.

Corrientes de líneas
Son las corrientes que circulan por las líneas que unen el generador con la carga. Estas
corrientes circulan del generador hacia la carga, y se denotan con un solo subíndice
correspondiente al borne de la línea en referencia.
Así el circuito de la Fig. 51, las corrientes de líneas son: I R , IS e I T
Aplicando 1ra Ley de Kirchhoff a los bornes “R”, “S” y “T” de la carga en ∆, se tiene:
En el borne “R” I R = I RT − ISR

…(166)

En el borne “S” IS = ISR − I TS

…(167)

En el borne “T” I T = I TS − I RT

…(168)

Reemplazando las corrientes de fases, en los segundos miembros en las ecuaciones
(166), (167) y (168) se tiene:
IR =

Ef
E
E
/ − ϕº − f / 120 º − ϕº = 3 f / − 30 º − ϕº A
Zf
Zf
Zf

IS =

Ef
E
E
/ 120 º − ϕº − f / − 120 º − ϕº = 3 f / 90 º − ϕº A …(170)
Zf
Zf
Zf

Análisis de circuitos bifásicos y trifásicos

50

…(169)

LEIV.
IT =

Ef
E
E
/ − 120 º −ϕ º − f / − ϕº = 3 f / − 150 º − ϕº A …(171)
Zf
Zf
Zf

Sumando miembro a miembro las ecuaciones (169), (170)
I R + IS + I T = 0 A …(172)

y

(171) se tiene:

Las magnitudes eficaces de las corrientes de líneas determinadas por los segundos
miembros de las ecuaciones (169). (170) y (171), se concluye:
I R = I S = I T = I l = 3 I f A …(173), y se encuentran desfasadas entre ellas en 120°
eléctricos, siendo la suma fasorial igual a cero

El diagrama fasorial de tensiones y corrientes con factor de potencia en atraso en
secuencia inversa, se muestra en la Fig. 52.
E ST

E SR

− I TS

IS
(30 º − ϕº )

ISR
E RT

ϕº 30 º

60 º
IT
− I RT
I TS

E RT

ϕº
30 º
ϕº
90 º − ϕº

I RT

− ISR
IR
ϕº > 0

E TS

Potencias aparentes de fases
Determinando las potencias aparentes absorbidas por cada fase de la carga, se tiene:
*
En la fase S-R SSR = E SR ISR = E f / 120 º

E2
Ef
/ − 120 º + ϕº = f / ϕ º VA …(174)
Zf
Zf

En la fase T-S STS = E TS I * = E f / − 120 º
TS
Análisis de circuitos bifásicos y trifásicos

E2
Ef
/ 120 º + ϕº = f / ϕ º VA
Zf
Zf

51

…(175)

LEIV.
En la fase R-T SRT = E RT I * = E f / 0 º
RT

E2
Ef
/ ϕ º = f / ϕº VA
Zf
Zf

…(176)

Se observa que en el sistema balanceado en ∆ en secuencia inversa, las potencias
aparentes por fase son iguales, es decir:
E2
SSR = STS = SRT = Sf = f / ϕº VA
…(177)
Zf
Se deja como ejercicio al lector para que demuestre:

SSR = SRS = STS = SST = SRT = STR

…(178)

Potencia aparente total
La potencia aparente total absorbida por la carga es igual a la suma fasorial de las tres
potencias aparentes de cada fase

Stot = SSR + STS + SRT = 3

2
Ef

Zf

/ ϕ º VA

La ecuación (179) puede escribirse: Stot = 3 E f

…(179)

Ef
/ ϕ º VA
Zf

En delta la tensión de fase es tensión de línea E f = E l

…(180)

…(181)

Ef
= I f …(182)
Zf
Reemplazando las ecuaciones (181) y (182) en la ecuación (180) se tiene:

En delta la corriente de fase es

Stot = 3 x E l x I f / ϕº VA

…(183)

Por estar la carga en ∆ según la ecuación (173) se deduce: I f =

Il

A …(184)
3
Reemplazando (184) en (183) y efectuando operaciones en el segundo miembro de la se
obtiene:
Stot = 3 E l I l / ϕ º VA

…(185)

El módulo de la potencia aparente total es: S tot = 3 E l I l VA …(186)
Pero

Stot = Ptot + J Q tot VA

Análisis de circuitos bifásicos y trifásicos

…(187)
52

LEIV.
Potencia activa total
De la ecuación (185) la potencia activa total trifásica es:

Ptot = 3 E l I l cos ϕº vatios …(188)
Potencia reactiva total
De la ecuación (185) la potencia reactiva total trifásica es:

Q tot = 3 E l I l senϕº var es …(189)

E SR

E ST
− I TS

ISR

IS

30 º 30 º
60 º

I RT

E RT

30 º
IT

− ISR
I TS

IR

− I RT
ϕº = 0

E TS

Ejercicio
Dibujar el diagrama fasorial de tensiones y corrientes de un sistema trifásico trifilar en
∆, balanceado en secuencia inversa con factor de potencia en adelanto

Análisis de circuitos bifásicos y trifásicos

53

LEIV.
Medida de la potencia activa mediante el método de dos vatímetros monofásicos
S

S

IS

ISR

E Ss = E SR
s
r

t

Zf

PR

R

I TS

IR

R

E Rr = E RT

E Tt = E TS

Zf

Zf
I RT
T

PT

IT

T

Los vatímetros se han instalado en las líneas “R” y “T” según la Fig. 51 para la carga
trifásica balanceada en delta, en secuencia inversa, luego las lecturas de los vatímetros
serán:
E RS
IR

…(190)

E TS
IT

…(191)

PR = E RS I R cos〈
PT = E TS I T cos〈

La tensiones de líneas: E RS = E TS = E l

…(192)

Las corrientes de líneas: I R = I T = I l

…(193)

Del diagrama de la Fig. 52 se obtienen los ángulos entre tensiones y corrientes, así
〈

E RS
IR

= 30 º − ϕº

…(194)

〈

E TS
IT

= 30 º + ϕ º

…(195)

Reemplazando las ecuaciones (192), (193) y (194) en la ecuación (190) se tiene:
PR = E l I l cos(30 º − ϕº ) vatios

…(196)

Reemplazando las ecuaciones (192), (193) y (195) en la ecuación (191) se tiene
PT = E l I l cos(30 º + ϕº ) vatios
Análisis de circuitos bifásicos y trifásicos

…(197)

54

LEIV.
E SR

E ST
− I TS

IS
(30 º − ϕº )

ISR
E RT

ϕº 30 º
60 º
IT
− I RT
I TS

E RT

ϕº
I RT
30 º
ϕº
90 º − ϕº

ϕº > 0
− ISR
30 º − ϕº

IR

E RS

E TS

Sumando miembro a miembro las ecuaciones (196) y (197) se tiene:
PR + PT = E l I l  cos(30º − ϕ º ) + cos(30 º + ϕº )  vatios





…(198)

Efectuando operaciones en el segundo miembro de la ecuación (198) se tiene:
PR + PT = 3 E l I l cos ϕ º vatios

…(199)

El segundo miembro de la expresión de la ecuación (199), es idéntica a la expresión del
segundo miembro de la ecuación (188), e indica que los dos vatímetros monofásicos dan
valores de lecturas, cuya suma es igual a la potencia activa total absorbida por la carga
trifásica balanceada en ∆
La potencia total activa es función de los valores eficaces de tensión de línea, corriente
de línea y del factor de potencia total (igual al factor de potencia de cada impedancia de
fase)

Graficando las potencias de las ecuaciones (196), (197) y (199) se obtiene la Fig. 54, de
la que se deduce:

Análisis de circuitos bifásicos y trifásicos

55

LEIV.
Ptot
El Il
Ptot
El Il

3

PR
El Il

3/2
75º

− 75º

− ϕº
− 90 º

− 60 º − 45º − 30 º − 15º

15º 30 º

45º

ϕº

60 º

90 º

PT
El Il

Fig. 54.- Gráfica de PR , PT , Ptot en secuencia inversa de un sistema trifásico
balanceado en

1º. Para factor de potencia total en atraso, en secuencia inversa R T S R T S, se han
conectado dos vatímetros:
*

*

a. Un vatímetro conectado en la línea R y el otro en la línea T R T S R T S ,
quedando la línea “S” centrada entre las dos líneas donde se han conectado los
vatímetros
Así: para factor de potencia en atraso y en secuencia inversa, PR ( vatímetro
que se encuentra a la derecha ) tiene mayor lectura que PT (vatímetro que se
encuentra a la izquierda) es decir PR > PT

b. Si se hubiera conectado los vatímetros en las líneas “S” y “T”, para factor de
*

*

potencia en atraso y en secuencia inversa R T S R T S , la lectura PT > PS
c. Si se hubiera conectado los vatímetros en las líneas “R” y “S”, para factor de
*

*

potencia en atraso y en secuencia inversa R T S R T S , la lectura PS > PR
2º. Si el factor de potencia total es unitario, las lecturas son iguales. PR = PT

3º. Para factor de potencia total en adelanto, en secuencia (-), se conecta dos vatímetros:
*

*

a. Con los vatímetros en las líneas “R” y “T” R T S R T S , vatímetro
conectado en la línea “T” ( vatímetro que se encuentra a la izquierda) tiene
mayor lectura que el vatímetro conectado en la línea R (vatímetro que se
encuentra a la derecha) es decir PT > PR
b. Si se hubiera conectado los vatímetros en las líneas “S” y “T”, para factor de
*

*

potencia en adelanto y en secuencia inversa R T S R T S , la lectura PS > PT
c. Si se hubiera conectado los vatímetros en las líneas “R” y “S”, para factor de
*

*

potencia en adelanto y en secuencia inversa R T S R T S , la lectura PR > PS
Análisis de circuitos bifásicos y trifásicos

56

LEIV.
4º. En las ecuaciones (196) y (197), al cambiar ϕº por − ϕ º , se permutan las lecturas
NOTA.- La Fig. 50 que corresponde al circuito trifásico balanceado en Y en secuencia
negativa, y Fig. 53 que corresponde al circuito trifásico balanceado en ∆ en secuencia
negativa, ambas figuras son exactamente iguales.
Las ecuaciones de potencia indicada por cada vatímetro PR y PT y la potencia total Ptot
son las mismas; por lo que se deduce que el método de los dos vatímetros es válido para
cualquier circuito trifásico, balanceado y desbalanceado, y en cualquier secuencia;
debido que las lecturas de cada vatímetro dependen únicamente de los valores de líneas,
y del cos(30º ± ϕº ).

MEDIDA DE LA POTENCIA REACTIVA EN UN SISTEMA Y o ∆
BALANCEADO EN SECUENCIA DIRECTA
En todo sistema triásico balanceado en ∆ o Y, la potencia reactiva total es medida
mediante el método del vatímetro monofásico en cuadratura.

Vatímetro en cuadratura en el sistema Y o ∆ en secuencia directa
Es la conexión de un vatímetro monofásico en una línea en un sistema trifásico
balanceado, mediante el cual se demostrará que su lectura es directamente proporcional
a la potencia reactiva total, absorbida por una carga trifásica balanceada.
Para la secuencia de fases (+)

I
Si el vatímetro monofásico se conectara en la línea R,

E

←−
* *

, conectando el

RSTRST
vatímetro monofásico en la línea R, La corriente de línea I R ingresa por la marca
de polaridad de la bobina de corriente; y a la bobina de tensión se aplica el voltaje
E ST
I E
Si el vatímetro monofásico se conectara en la línea S,
la corriente de
* *←−
RSTRST
línea IS ingresa por la marca de polaridad de la bobina de corriente; y a la bobina
de tensión se le aplica el voltaje E TR
I

E

Si el vatímetro monofásico se conectara en la línea T,
de línea I T

la corriente
* *←−
RST RST
ingresa por la marca de polaridad de la bobina de corriente; y a la

bobina de tensión se le aplica el voltaje E RS

Análisis de circuitos bifásicos y trifásicos

57

LEIV.
Lectura del vatímetro monofásico en cuadratura para un sistema trifásico en “Y”
en secuencia directa
Conectando el vatímetro monofásico en la línea “R” cuyo circuito se muestra en la
Fig. 55.
IT

T
T

E TN
t
N

s

r

Zf

´
PR

IR

Zf

R
E RN

E SN

Zf

N
S

I R + IS + I T

IS

S

´
la lectura del vatímetro monofásico es: PR = E ST I R cos〈

E ST
IR

…(200)

Del diagrama fasorial correspondiente (Fig. 34) mostrado en la siguiente página, se
tiene:
…(201)
(tensión de línea)
E ST = E l
…(202)
(corriente de línea)
I R = Il
〈

E ST
IR

= 90 º − ϕ º

…(203)

(ángulo entre E ST e I R )

Reemplazando las ecuaciones (201), (202) y (203) en la ecuación (200) tenemos:
´
PR = E l I l cos( 90 º − ϕº )

siendo cos(90 º − ϕº ) = senϕº

…(204)
…(205)

´
Reemplazando la ecuación (205) en la (204) se tiene: PR = E l I l senϕº
Pero la potencia reactiva total es:

…(206)

Q tot = 3 E l I l senϕº var es …(207)
Reemplazando la ecuación (206) en (207) se tiene:
´
Q tot = 3 PR var es …(208)

Análisis de circuitos bifásicos y trifásicos

58

LEIV.
´
Luego PR =

Q tot
3

…(209)

E TS

− E RN

30º
E TR

S

30º

E TN

E RS

º
30 − E SN

IT

30º

ϕº

30º
ϕº
IS

90º
ϕº
30º

E RN

IR

E SN

30º
E ST
− E TN
Fig. 34.- Diagrama fasorial de tensiones de fase, tensiones de línea y de
corrientes de línea, de un sistema trifásico tetrafilar balanceado en
secuencia (+), con factor de potencia en atraso

Enunciado 1. La ecuación (209) indica que en un circuito trifásico balanceado en Y en
secuencia directa, cualquiera sea su factor de potencia, un vatímetro monofásico en
cuadratura indica que su lectura igual a la potencia reactiva total trifásica, entre raíz de
tres.

Lectura del vatímetro monofásico en cuadratura para un sistema trifásico en “∆”
en secuencia directa
Conectando el vatímetro monofásico en la línea “R” cuyo circuito se muestra en la
Fig. 56
.

Análisis de circuitos bifásicos y trifásicos

59

LEIV.
T

T

IT
I TR

E Tt
t
´
PR

R
r

s

Zf
I ST

IR

R

Zf

E Rr

Zf

E Ss
S
S

I RS

IS

´
La lectura del vatímetros es PR = E ST I R cos〈

E ST
IR

…(210)

Del diagrama fasorial correspondiente (Fig. 38) se tiene:
E ST = E l

…(211)

(tensión de línea)

I R = Il

…(212)

(corriente de línea)

…(213)

(ángulo entre E ST e I R )

〈

E ST
IR

= 90 º − ϕ º

Reemplazando las ecuaciones (211), (212) y (213) en la ecuación (210) tenemos:
´
PR = E l I l cos( 90 º − ϕº )

siendo cos(90 º − ϕº ) = senϕº

…(214)
…(215)

´
Reemplazando la ecuación (215) en la (214) Se tiene: PR = E l I l senϕº

…(216)

Pero la potencia reactiva total es

Q tot = 3 E l I l senϕº var es …(217)
reemplazando la ecuación (216) en (217) se tiene:
´
Q tot = 3 PR var es …(218)

´
Luego PR =

Q tot
3

…(219)

Análisis de circuitos bifásicos y trifásicos

60

LEIV.
E TR

E TS

− IST I T

(30 º − ϕº )

I TR
ϕº 30 º
60 º

IS

30º
ϕº

− I RS
IST

ϕº
30 º
90 º − ϕº

E RS
I RS

− I TR
IR

E ST

Fig. 38.- Diagrama fasorial de tensiones y corrientes de
un sistema trifásico trifilar en , balanceado en
secuencia (+), con factor de potencia en atraso

Enunciado 2. La ecuación (219) muestra que en un circuito trifásico balanceado en ∆
en secuencia directa, cualquiera sea su factor de potencia, un vatímetro monofásico en
cuadratura indica que su lectura igual a la potencia reactiva total trifásica, entre raíz de
tres.
De los enunciados 1 y 2 correspondientes a las ecuaciones (209) y (219) se deduce:

En todo circuito trifásico balanceado en secuencia directa, en Y o ∆, un vatímetro
conectado en cuadratura, indicará una lectura igual a la potencia reactiva trifásica total
absorbida, entre raíz de tres.
Las expresiones de las ecuaciones (206) y (216) es válida para todo circuito trifásico
balanceado en secuencia directa, cualquiera sea el factor de potencia de la carga
trifásica, debido a que la lectura depende de los valores de tensión de línea y corriente
de línea y del seno del ángulo ϕº

Análisis de circuitos bifásicos y trifásicos

61

LEIV.
PROBLEMAS RESUELTOS DE CIRCUITOS TRIFÁSICOS BALANCEADOS
Problema 1
Una carga trifásica balanceada conectada en estrella, tiene 16 Ω de resistencia y 12 Ω
de reactancia inductiva en serie en cada fase, y es alimentada por una línea trifásica de
230 V.
Determinar:
a. El valor eficaz de la corriente de línea.
b. La potencia activa total.
Solución
La carga equilibrada está conectada en estrella, luego las corrientes de fase y de línea
son las mismas, y están desfasadas en 120 grados eléctricos.
La impedancia de fase es:

Z f ,Y = 16 + J 12 Ω
Z f ,Y = 20 / 36.87 º Ω
El valor eficaz de la tensión de línea es: Vℓ = 230 voltios
El valor eficaz de la tensión de fase es:
V
230
Vf = l =
= 132.79 V
3
3
El valor eficaz de la corriente fase
If = Il =

Vf
132.79
=
= 6.64 A Rpta. a.
Z f ,Y
20

La potencia activa total trifásica es:
2
Ptot = 3I f R f
R f = 16 Ω
Ptot = 3 x 6.64 2 x 16 = 2116.30 vatios Rpta. b.
Otra forma de calcular la potencia activa total es mediante la fórmula:
Ptot = 3 E l I l Cos ϕ

Ptot = 3 x 230 x 6.64 x 0.8 = 2116.15 vatios

Rpta. b.

Problema 2
Para el problema 1; si las tres impedancias se conectan en triángulo y si se colocan a
través de los mismos voltajes de línea, determinar:

a. El valor eficaz de las corrientes de líneas.
b. El valor eficaz de las corrientes de fase.
Análisis de circuitos bifásicos y trifásicos

62

LEIV.
c. La potencia activa total.
Solución
La carga está conectada en delta, luego la impedancia por fase en delta es:
Z f ,∆ = 16 + J12 Ω

Z f ,∆ = 20 / 36.87º Ω
Con la carga conectada en triángulo o delta, la tensión de línea es también la tensión de
fase.
Luego: E l = Ef
El valor eficaz de la corriente de fase de la carga en delta es:
If =

Ef
230
=
= 11.50 A. Rpta. b.
Z f ,∆
20

El valor eficaz de la corriente de línea es:

Il = 3 If
I l = 3 x 11.50 = 19.92 A Rpta. a.
2
La potencia activa total Ptot = 3Pf = 3I f R f

Ptot = 3 x 11.50 2 x 16 = 6348 vatios

Rpta. c.

Otra forma de calcular la potencia es a través de la fórmula:

Ptot =

3 E l I l Cos ϕ

Cos ϕ = Cos 36.87 º = 0.8

Ptot =

3 x 230 x 19.92 x 0.8 = 6348.45 vatios

Rpta. c.

Problema 3

Dos cargas en paralelo están alimentadas por una línea trifásica a una tensión de 240
voltios, 60 Hz. Una de las impedancias de fase de la carga en delta es 12 / – 60º Ω. Y
la otra impedancia de fase de la carga en estrella es 10 / 25º Ω. Determinar:
a. La corriente de línea.
b. El factor de potencia total.
Solución
Sea:
Vl : Tensión de línea: Vl = 240 voltios
f = 60 Hz.

Z f ,∆ : Impedancia de fase de la carga trifásica equilibrada conectada en triángulo.
Análisis de circuitos bifásicos y trifásicos

63

LEIV.
Z f ,Y : Impedancia de fase de la carga trifásica equilibrada en estrella.
Z f ,∆ = 12 / –60º Ω ,

Z f ,Y = 10 / 25º Ω

Como Z f ,∆ y Z f .Y son cargas trifásicas equilibradas, el sistema trifásico total es
equilibrado. Luego para la solución es conveniente desarrollar mediante el equivalente
monofásico; para lo cual, se pasa la carga delta a estrella.
'

Sea Z f ,Y = Z f ,∆ / 3 la impedancia por fase equivalente en estrella, de la carga delta
'

I1 : Corriente en Z f ,Y
E RN

I 2 : Corriente en Z f ,Y

I1
'
Zf , Y

I2
Zf ,Y

E RN : Tensión de fase a 0º (referencia)
E RN =

I1 =

I2 =

240
/ 0º V
3

E RN
'
Z f ,Y

E RN
Z f ,Y

=

=

E RN = 138.564 / 0º V

138.56 / 0º
4 / − 60º

= 34.64 / 60º A

138.56 / 0º
= 13.85 / − 25º A
1 0 / 25º

a. La corriente de línea I R = I1 + I 2

I R = 34.64 / 60º + 13.85 / − 25º A

I R = 29.87 + J 24.15 = 38.41 / 38.94º

A Rpta. a.

Otro método es determinar la impedancia por fase total en estrella
Z f ,Y ,tot =

'
Z f ,Y Z f ,Y
'
Z f ,Y + Z f ,Y

=

4 / − 60º x 10 / 25º
= 3.607 / − 38.941º Ω
4 / − 60º + 10 / 25º

La corriente de línea es: I R =

138.56 / 0º
E RN
=
= 38.413/ 38.941º A OK.
Z f ,Y ,tot 3.607 / − 38.941

b. El factor de potencia total es Cos -38.94º = 0.7777 Rpta.b.
Análisis de circuitos bifásicos y trifásicos

64

LEIV.
Problema 4

Un motor de inducción de 50 HP trifásico, conectado en estrella a una línea trifásica de
440 voltios y 60 Hz está trabajando al 80% de su potencia nominal. El motor tiene el
factor de potencia de 0.76 y eficiencia de 72%.
Determinar la capacidad por fase en una conexión en triángulo requerida a través de la
línea para corregir el factor de potencia a 0.92 en atraso.
Solución

Motor: 50HP conectado en estrella.
Eℓ = 440 voltios

f = 60 Hz.

η = 72% = 0.72

Cos ϕ m = 0.76

%PC = 0.8

La potencia aparente total del motor trifásico es:
Sm =

50 x 746 x 0.8
/ Cos −1 0.76 VA
0.76 x 0.72

S m = 54532.16 / 40.54º VA
La potencia aparente del motor por fase es la tercera parte de la potencia aparente total

Sf ,m = 18177.39 / 40.54º VA

ϕ1 = ϕ m = 40.54º

Sf ,m = 13813.95 + J 11814.92 VA = Pf ,m + J Q f ,m
El circuito equivalente monofásico es:
C f ,Y : Capacidad por fase en conexión en
estrella, del banco 3φ de condensadores.
Sf ,m

ϕ 2 = Cos −1 0.92
Como Eℓ = 440 V, se tiene que E f =

440
V.
3

→ ϕ 2 = 23.07 º

Ef = 254.03 V.

Así la capacidad por fase en conexión estrella C f ,Y para mejorar el factor de potencia
es: C f ,Y =

Pf ,m
E RN 2 ω

(Tg ϕ1 − tg ϕ 2 ) F , y que al reemplazar datos se tiene:

Análisis de circuitos bifásicos y trifásicos

65

LEIV.
C f ,Y =

13813.95
254.03 2 x 377

(Tg 40.54º − tg 23.07 º )

C f ,Y = 243.8 uF
Siendo X Cf , ∆ la reactancia capacitiva por fase en conexión delta
como: X Cf ,Y =

X Cf ,∆

entonces

3
C f ,Y
3

de donde: C f ,∆ =

1
1
=
2 π f C f , y 2 π f C f ,∆ (3)

→ C f ,∆ =

243.8
= 81.27 uF
3

Rpta.

Problema 5

Un generador trifásico de 200 voltios de tensión de línea, alimenta a una carga inductiva
conectada en estrella, que absorbe 10 amperios a través de una línea aérea trifásica de
1 Ω de resistencia y 5 Ω de reactancia inductiva por conductor. Determinar la tensión de
fase en bornes de la carga si:
a. El factor de potencia en los terminales de la carga es 0.6 (-)
b. El factor de potencia en terminales del generador es 0.6 (-)
Solución

Eℓ= 200 voltios
a. Z f ,Y = Z f ,Y

Z f ,Y : impedancia por fase de la carga en estrella
/ Cos −1 0.6

=

/ 53.13º Ω

Z f ,Y

Z f , Y = 0. 6 Z f , Y + J 0. 8 Z f , Y Ω
R
2

Zl = 1 + J 5

2

IR

Z f ,tot = (1 + 0.6 Z f ,Y ) + (5 + 0.8Z f ,Y ) Ω

VZf ,Y

Zf , Y

Il =

E RN =

Ef
Z f ,tot

,

Ef =

El
200
=
V
3
3

N

200
/ 0º V (referencia a 0º )
3
Luego:

200 / 3

10 =

2

(1 + 0.6 Z f ,Y ) + (5 + 0.8Z f ,Y )
Análisis de circuitos bifásicos y trifásicos

66

2

… (1)
LEIV.
Simplificando la ecuación (1) se tiene:
2
Z f ,Y + 9.2 Z f ,Y − 107.33 = 0

…(2)

Resolviendo la ecuación (2) se tiene:
Z f ,Y = 6.74 Ω así Z f ,Y = 6.74 / 53.13º Ω
Tomando

E RN = 115.47 / 0º V

(referencia)

Por divisor de tensión tenemos que la caída de tensión en la impedancia Z f ,Y es

VZf ,Y = E RN

Z f ,Y
Z f ,Y + (1 + J5)

Z f ,Y = R + J X L

Reemplazando valores y efectuando operaciones se tiene:

V Zf ,Y = 67.4 / − 11º V

Rpta. a

b. Factor de potencia 0.6 en bornes del generador.
IR

Zl = 1 + J 5

Tg 53.13º =

E RN

5 + XL
1+ R

luego:
1 + R = 3.75 + 0.75 XL

Z f ,tot = (1 + R ) 2 + (5 + X L ) 2

Ω

Z f ,tot = (3.75 + 0.75X L ) 2 + (5 + X L ) 2
Pero: I R = I f = I l =

Por lo tanto: 10 =

E RN
,
Z f ,tot

….(A)

Ω

E RN = 115.47 / 0º

115.47
(3.75 + 0.75 X L ) 2 + (5 + X L ) 2

V

…. (B)

Resolviendo la ecuación (B), se tiene: XL = 4.237 Ω
Análisis de circuitos bifásicos y trifásicos

67

LEIV.
Luego por la ecuación (A) se tiene que:

R = 5.928 Ω

La impedancia equivalente total por fase en bornes del generador es:
Z f ,tot = 6.928 + J 9.237 = 11.547 / 53.13º Ω
La corriente de línea I R es:
IR =

115.47 / 0º
E RN
=
= 10 / − 53.13º A
Z f ,tot 11.547 / 53.13º

Siendo la impedancia por fase Z f ,Y = R + J X L Ω , entonces:
Z f ,Y = 5.928 + J 4.237 Ω
La caída de tensión por fase en la impedancia es:
V Zf ,Y = I Z = 10 / − 53.13º x 7.287 / 35.55º
V Zf ,Y = 72.87 / − 17.57º

V

Rpta.b.

Problema 6
Una carga trifásica balanceada inductiva absorbe 5 KW y 17.32 KVAR. Determinar las
lecturas de los dos vatímetros conectados para medir la potencia total.
Solución
Sea la secuencia de fases positiva, con los vatímetros conectados en las líneas R y T,
luego las lecturas de los vatímetros será:
PR = E I Cos (30º + ϕ)

... (1)

PT = E I Cos (30º - ϕ)

... (2)

(1)/(2) da:

PR Cos (30º + ϕ)
=
PT Cos (30º − ϕ)

Empleando propiedades de las proporciones:

Análisis de circuitos bifásicos y trifásicos

68

LEIV.
PR + PT
Cos (30º + ϕ) + Cos (30º − ϕ)
=
PR
Cos (30º + ϕ)
PR + PT
3 Cos ϕ
=
PR
Cos (30º + ϕ)

De (3) : PR = Ptot

ϕ = Tg −1

...(3)

Cos (30º + ϕ)
3 Cosϕ

...(4)

Q tot
17.32
= tg −1
= 73.9º
Ptot
5

Reemplazando el valor de ϕ en (4) se tiene:
PR = -2.5 KW Rpta.
Rpta.

Así PT = Ptot – PR = 5 – (-2.5) = 7.5KW
Problema 7

Una carga balanceada conectada en delta, tiene como impedancia de fase un circuito
serie de 12 Ω. de resistencia y 16 Ω. de reactancia capacitiva. Las tensiones de línea
sonde 115 voltios, determinar las corrientes de línea y de fase.
Solución
La carga está conectada en delta. La impedancia de fase es:

Z f ,∆ = 12 − J 16 ohms. = 20 / − 53.13º

Ω

La tensión de línea es: Eℓ = 115 voltios
Luego la corriente de fase es: If = Eℓ / Zf,∆

If =

115
= 5.75 A
20

Rpta.

la corriente de línea es: I l = 3 I f

I l = 3 x 5.75 = 9.96 amp.

Análisis de circuitos bifásicos y trifásicos

Rpta.

69

LEIV.
Problema 8

Una carga balanceada en delta tiene 18 Ω. de resistencia y 24 Ω de reactancia capacitiva
en serie en cada fase, siendo alimentada por líneas con impedancia cada una de 1 Ω de
reactancia y 2 Ω. de reactancia inductiva por conductor. El generador que alimenta al
circuito total tiene tensión entre líneas de 250 voltios. determinar.
Solución

Carga trifásica conectada en delta, con impedancia por fase:
Z f ,∆ = 18 − J 24 = 30 / − 53.13º Ω
La impedancia de línea Z l = 1 + J 2

Ω.

Tensión de línea proporcionada por el generador es: Eℓ = 250 voltios.
Como el circuito total es balanceado, se resuelve con un equivalente monofásico.

E f = E RN =
Sea:

250
= 144.34 V
3

E RN = 144.34 / 0º

Zl = 1 + J 2

V (referencia a 0º)

E RN

Zf , Y

La impedancia por fase en conexión estrella:
Z f ,Y =

Z f ,∆
= 10 / − 53.13º
3

Ω

la impedancia equivalente total por fase es:
Z f ,tot ,Y = 1 + J 2 + 10 / − 53.13º Ω
Z f ,tot ,Y = 1 + J 2 + 6 − J 8 Ω
Z f ,tot ,Y = 7 − J 6 = 9.22 / − 40.6º Ω
La corriente de línea I R es:

IR =

E RN
Z f ,tot

=

144.34 / 0º
A
9.22 / − 40.6º

I R = 15.66 / 40.6º A
Análisis de circuitos bifásicos y trifásicos

70

LEIV.
La caída de tensión por fase en la carga en conexión estrella es:

V f ,Y = I Z f ,Y = 15.66 / 40.6º x 10 / − 53.13º V
V f ,Y = 156.6 / − 12.53º V
a.

El voltaje entre líneas en los terminales de carga es:

Vl = 3 Vf ,Y = 3 x 156.6 = 271.24 V Rpta. a
b.

La potencia total consumida por la carga es:

Ptot = 3 I 2 R f = 3 x 15.66 2 x 6 = 4414.24

vatios

Rpta. b

Problema 9

Una carga inductiva balanceada conectada en estrella toma 5.4KW a 0.6 de factor de
potencia, a 200 voltios de tensión de líneas. Esta carga se encuentra conectada en
paralelo con otra carga en estrella balanceada, puramente resistiva, la que toma 5 KW.
Determinar la corriente total de línea suministrada a las dos cargas.
Solución
La primera carga trifásica en estrella, que toma una potencia de 5.4KW a Cos ϕ1 = 0.6

Luego ϕ1 = Cos −1 0.6 = 53.13º , así: P1 = 5.4 KW

Q1 = P1 tg ϕ1 = 5.4 tg 53.13 = 7.2 KVAR
La potencia aparente total que toma la primera carga trifásica
En estrella es:
S1 = 5.4 + J 7.2 = 9 / 53.13º KVA
La segunda carga trifásica en estrella toma una potencia aparente S 2 = 5 / 0º KVA
Así las dos cargas trifásicas en conjunto (ambas conectadas en paralelo) absorben una
potencia aparente total:
S tot = S1 + S 2 KVA

Análisis de circuitos bifásicos y trifásicos

71

LEIV.
S tot = 5.4 + J 7.2 + 5 = 10.4 + J 7.2 KVA
S tot = 12.65 / 34.7 º KVA
Pero

S tot = 3 E l I l . Luego:

Il =

S tot

Il =

3 El

, siendo E l = 200 V

12,650
= 36.52 A Rpta.
3 x 200

Problema 10

Una conexión delta balanceada tiene una impedancia de fase de 12 / 70º Ω y es
alimentada por una línea trifásica de 240 voltios, 60 Hz. Se mide la potencia del circuito
por el método de dos vatímetros. Determinar la lectura indicada por cada vatímetro y la
potencia trifásica total
Solución:
Carga balanceada en delta con impedancia de fase:
Z ∆ ,f = 12 / 70º Ω

, ϕ = 70º (factor de potencia en atraso)

Tensión de línea: Ef = Eℓ = 240 voltios, 60 Hz (Por tener la carga en delta)
Sea la secuencia de fase positiva, con los vatímetros conectados en las líneas S y T,
R S T R S T, teniendo la carga factor de potencia en atraso, se tiene: PS > PT
La corriente de fase es:

If =

E
Ef
240
= l =
= 20 A
Z ∆ ,f Z ∆ ,f
12

La corriente de línea es: I l = 3 I f = 3 x 20 = 34.64 A
La potencia indicada por cada vatímetro es:
PS = E l I l Cos (30º − ϕ) vatios, y que al reemplazar valores se tiene :
PS = 240 x 34.64 x Cos (30º − 70º ) = 6368.59 vatios Rpta.
PT = E l I l Cos (30º + ϕ) vatios, y que al reemplazar valores se tiene :
PS = 240 x 34.64 x Cos (30º + 70º ) = − 1443.60 vatios Rpta.
Análisis de circuitos bifásicos y trifásicos

72

LEIV.
La potencia total es: Ptot = PS + PT
Ptot = 6368.59 + (-1443.68) = 4924.91 vatios Rpta
Problema 11

Una delta balanceada toma 14.5 / 30º KVA de una línea trifásica, con tensión
E RS = 440 / 0º V a 60 Hz y en secuencia negativa.
Determinar:
a. La impedancia de fase de la carga en delta Z ∆ ,f
b. Las corrientes de fase en forma polar.
c. Las corrientes de líneas en forma rectangular.
d. La bancada trifásica de condensadores conectados en estrella, para corregir el factor
de potencia total a 0.93 en atraso.
Solución

Graficando la delta en secuencia negativa de forma tal que la tensión ERS se esté a cero
grados.
S

R
N

Así en el triángulo graficado, las tensiones de fase y de líneas
indican sus ángulos de fases.
Del diagrama fasorial de tensiones de fase y de líneas quedan
definidas en módulo y ángulo, tal como se muestra en la
figura.

T

EST

El = 440 voltios
f = 60 Hz.
La potencia aparente total absorbida por la carga
trifásica en delta es S tot = 14500 / 30º KVA

ESN

E RN

30
º

30º

E RS

30

º

Resolviendo mediante el equivalente 1 φ en bornes
R-N, se tiene:

E TN

E RN

E
=
= 254.03 / 30º V
3

E TR

Sea Z Y ,f la impedancia por fase de la carga en conexión estrella. Para el equivalente
monofásico, la potencia aparente por fase tomada de la línea es la tercera parte de la
potencia aparente total, así:

Análisis de circuitos bifásicos y trifásicos

73

LEIV.
S Y ,f =
Pero:

S tot 14500
=
= 4833.33 / 30º VA
3
3
IR

S Y ,f = E RN I R
E RN

*

Luego: I R = S Y ,f / E RN
IR =

Zf ,Y

4833.33 / 30º
= 19.03 / 0º A
254.03 / 30º

Z Y ,f =

E RN
IR

=

254.03 / 30º
= 13.35 / 30º Ω
19.03 / 0º

La impedancia por fase en conexión en estrella es:
254.03 / 30º
E RN
Z Y ,f =
=
= 13.35 / 30º Ω.
19.03 / 0º
IR
a. La impedancia por fase en conexión delta será:
Z ∆ ,f = 3 Z Y ,f = 3 x 13.35 / 30º = 40.05 / 30º Ω Rpta.
b. Corrientes de fase en módulo y ángulo

E RS = 440 / 0º V

E ST = 440 / 120º V

E TR = 440 / − 120º V

I RS =

I ST =

I TR =

E RS
Z ∆ ,f
E ST
Z ∆ ,f
E TR
Z ∆ ,f

=

440 / 0º
= 10.99 / − 30º = 9.52 − J 5.50 A Rpta.
40.05 / 30º

=

440 / 120º
= 10.99 / − 90º = J 10.99 A Rpta.
40.05 / 30º

=

440 / − 12 0º
40.05 / 30º

Análisis de circuitos bifásicos y trifásicos

= 10.99 / − 150º = − 9.52 − J 5.50 A Rpta.

74

LEIV.
c. Las corrientes de línea en forma rectangular:

I R = I RS − I TR
I R = 10.99 / − 30º −10.99 / − 150º

IR

I R = 19.03 A Rpta.

IS

I S = I ST − I RS
I S = − 9.52 + J 16.49

Rpta.

A

IST

Z ∆,

I S = J 10.99 − (9.52 − J 5.50) =

IRS

Z ∆ ,f

f

IT

Z∆

,f

ITR

I T = I TR − I ST
I T = − 9.52 − J 5.50 − J 10.99
I T = − 9.52 − J 16.49 A Rpta
d. Bancada trifásica de condensadores en estrella para obtener el factor de potencia total
a 0.93 en atraso.
Del circuito equivalente monofásico, la potencia aparente por fase es:
S Y ,f = 4833.33 / 30º = 4185.79 + J 2416.64 VA
La potencia activa que toma cada fase es:

PY,f = 4185.79 vatios.

En condiciones iniciales ϕ1 = 30º
En condiciones finales ϕ2 = Cos −1 0.93 = 21.57 º
Luego: C Y ,f =

PY ,f
2
Ef V

C Y ,f =

( tg ϕ1 − tg ϕ 2 ) F.
4185.79

254.03 2 x 377

C Y ,f = 31.32uF

( tg 30º − tg 21.57 º )

Rpta.

Problema 12
Una delta balanceada absorbe 14.4 /-68º KVA de una línea trifásica de 600 voltios a
60 Hz. Determinar los vatios y vares, cuando las impedancias de fase se conectan en
estrella a una línea de 440 voltios y 25 Hz.
Solución:
Inicialmente la carga está conectada en delta, la cual absorbe en total 14400 /-68º VA; a
la tensión de línea de 600 voltios, 60 Hz.
Análisis de circuitos bifásicos y trifásicos

75

LEIV.
E RN =

Sea

600
/ 0º = 346.41 / 0º V
3

Así el primer equivalente monofásico es:
I R1
E RN

Z Y ,f : Es la impedancia equivalente por fase
en conexión estrella, de la carga balanceada en
delta.

ZY ,f

Sea S Y ,f : potencia aparente que absorbe Z Y ,f

S Y ,f =

14400
/ − 68º VA = 4800 / − 68º VA
3

Pero: S Y ,f = E RN I * 1
R
*

I R1 =

*

de donde I R1 =

S Y ,f
E RN

4800 / − 68º
= 13.86 / − 68º A
346.41 / 0º

Así: I R1 = 13.86 / 68º A
Por lo tanto: Z Y ,f =

E RN 346.41 / 0º
=
= 25 / − 68º Ω
I R1 13.86 / 68º

Luego, la impedancia de fase de la carga en conexión en delta es:
Z ∆ ,f = 3 Z Y,f = 75 / − 68º = 28.1 − J 69.54 Ω a 60 Hz
La disminución de frecuencia de 60 Hz a 25 Hz, tiene el efecto de aumentar la reactancia
capacitiva, por ser inversamente proporcional a la frecuencia. Luego la impedancia de
fase a 25 Hz en conexión delta es:
60
Ω a 25 Hz
25

'

Z ∆ ,f = 28.1 − J 69.54 x
'

Z ∆,f = 28.1 − J 166.89 = 169.24 / − 80.44º Ω a 25 Hz
'

Las impedancias de fase Z ∆,f se conectan en estrella, a la tensión de línea de 440
voltios a 25 Hz. El segundo equivalente monofásico es:

Análisis de circuitos bifásicos y trifásicos

76

LEIV.
Sistemas bifásicos y trifásicos LEIV
Sistemas bifásicos y trifásicos LEIV
Sistemas bifásicos y trifásicos LEIV
Sistemas bifásicos y trifásicos LEIV
Sistemas bifásicos y trifásicos LEIV
Sistemas bifásicos y trifásicos LEIV
Sistemas bifásicos y trifásicos LEIV
Sistemas bifásicos y trifásicos LEIV
Sistemas bifásicos y trifásicos LEIV
Sistemas bifásicos y trifásicos LEIV
Sistemas bifásicos y trifásicos LEIV
Sistemas bifásicos y trifásicos LEIV
Sistemas bifásicos y trifásicos LEIV
Sistemas bifásicos y trifásicos LEIV
Sistemas bifásicos y trifásicos LEIV
Sistemas bifásicos y trifásicos LEIV
Sistemas bifásicos y trifásicos LEIV
Sistemas bifásicos y trifásicos LEIV
Sistemas bifásicos y trifásicos LEIV
Sistemas bifásicos y trifásicos LEIV
Sistemas bifásicos y trifásicos LEIV
Sistemas bifásicos y trifásicos LEIV
Sistemas bifásicos y trifásicos LEIV
Sistemas bifásicos y trifásicos LEIV
Sistemas bifásicos y trifásicos LEIV
Sistemas bifásicos y trifásicos LEIV
Sistemas bifásicos y trifásicos LEIV
Sistemas bifásicos y trifásicos LEIV
Sistemas bifásicos y trifásicos LEIV
Sistemas bifásicos y trifásicos LEIV
Sistemas bifásicos y trifásicos LEIV
Sistemas bifásicos y trifásicos LEIV
Sistemas bifásicos y trifásicos LEIV
Sistemas bifásicos y trifásicos LEIV
Sistemas bifásicos y trifásicos LEIV
Sistemas bifásicos y trifásicos LEIV
Sistemas bifásicos y trifásicos LEIV
Sistemas bifásicos y trifásicos LEIV
Sistemas bifásicos y trifásicos LEIV
Sistemas bifásicos y trifásicos LEIV
Sistemas bifásicos y trifásicos LEIV
Sistemas bifásicos y trifásicos LEIV
Sistemas bifásicos y trifásicos LEIV
Sistemas bifásicos y trifásicos LEIV
Sistemas bifásicos y trifásicos LEIV
Sistemas bifásicos y trifásicos LEIV
Sistemas bifásicos y trifásicos LEIV
Sistemas bifásicos y trifásicos LEIV
Sistemas bifásicos y trifásicos LEIV
Sistemas bifásicos y trifásicos LEIV
Sistemas bifásicos y trifásicos LEIV
Sistemas bifásicos y trifásicos LEIV
Sistemas bifásicos y trifásicos LEIV
Sistemas bifásicos y trifásicos LEIV
Sistemas bifásicos y trifásicos LEIV
Sistemas bifásicos y trifásicos LEIV
Sistemas bifásicos y trifásicos LEIV
Sistemas bifásicos y trifásicos LEIV
Sistemas bifásicos y trifásicos LEIV
Sistemas bifásicos y trifásicos LEIV
Sistemas bifásicos y trifásicos LEIV
Sistemas bifásicos y trifásicos LEIV
Sistemas bifásicos y trifásicos LEIV
Sistemas bifásicos y trifásicos LEIV
Sistemas bifásicos y trifásicos LEIV
Sistemas bifásicos y trifásicos LEIV

Más contenido relacionado

La actualidad más candente

Tipos de conexiones de los transformadores
Tipos de conexiones de los transformadoresTipos de conexiones de los transformadores
Tipos de conexiones de los transformadoresJosef Alexander
 
1.3.1 polarizacion del jfet
1.3.1 polarizacion del jfet1.3.1 polarizacion del jfet
1.3.1 polarizacion del jfetjosefer28051989
 
Capitulo I completo (Chapman Electric Machinery Fundamentals 5th)
Capitulo I completo (Chapman Electric Machinery Fundamentals 5th)Capitulo I completo (Chapman Electric Machinery Fundamentals 5th)
Capitulo I completo (Chapman Electric Machinery Fundamentals 5th)Edgar Francisco Lozado Campoverde
 
Problemas de acoplamiento magnetico
Problemas de  acoplamiento magneticoProblemas de  acoplamiento magnetico
Problemas de acoplamiento magneticoJefferson Duran
 
Clase 2a analisis de circuitos Circuitos en Serie
Clase 2a analisis de circuitos Circuitos en SerieClase 2a analisis de circuitos Circuitos en Serie
Clase 2a analisis de circuitos Circuitos en SerieTensor
 
Clase 9 teorema de la maxima transferencia de potencia
Clase 9 teorema de la maxima transferencia de potenciaClase 9 teorema de la maxima transferencia de potencia
Clase 9 teorema de la maxima transferencia de potenciaTensor
 
Resistencia estática y dinamica de diodos
Resistencia estática y dinamica de diodosResistencia estática y dinamica de diodos
Resistencia estática y dinamica de diodosDrykers García Say
 
Seaparat elect pote
Seaparat elect poteSeaparat elect pote
Seaparat elect poteLuis Sanchez
 
Problema nuestra del calculo de factor de rizo E1
Problema nuestra del calculo de factor de rizo E1Problema nuestra del calculo de factor de rizo E1
Problema nuestra del calculo de factor de rizo E1Tensor
 
Recortadores Serie Y Paralelo
Recortadores Serie Y ParaleloRecortadores Serie Y Paralelo
Recortadores Serie Y ParaleloUisraelCircuitos
 
Codigo De Colores Para Resistencia Electrica
Codigo De Colores Para Resistencia ElectricaCodigo De Colores Para Resistencia Electrica
Codigo De Colores Para Resistencia ElectricaArnulfo Vega
 
Informe IEEE - conexio estrella - delta
Informe IEEE -   conexio estrella - deltaInforme IEEE -   conexio estrella - delta
Informe IEEE - conexio estrella - deltaHarold Medina
 
practica circutos RL en serie y paralelo
practica circutos RL en serie y paralelopractica circutos RL en serie y paralelo
practica circutos RL en serie y paraleloMiguel Cajiga
 

La actualidad más candente (20)

Tipos de conexiones de los transformadores
Tipos de conexiones de los transformadoresTipos de conexiones de los transformadores
Tipos de conexiones de los transformadores
 
1.3.1 polarizacion del jfet
1.3.1 polarizacion del jfet1.3.1 polarizacion del jfet
1.3.1 polarizacion del jfet
 
Capitulo I completo (Chapman Electric Machinery Fundamentals 5th)
Capitulo I completo (Chapman Electric Machinery Fundamentals 5th)Capitulo I completo (Chapman Electric Machinery Fundamentals 5th)
Capitulo I completo (Chapman Electric Machinery Fundamentals 5th)
 
Valores eficaces
Valores eficacesValores eficaces
Valores eficaces
 
Problemas de acoplamiento magnetico
Problemas de  acoplamiento magneticoProblemas de  acoplamiento magnetico
Problemas de acoplamiento magnetico
 
Clase 2a analisis de circuitos Circuitos en Serie
Clase 2a analisis de circuitos Circuitos en SerieClase 2a analisis de circuitos Circuitos en Serie
Clase 2a analisis de circuitos Circuitos en Serie
 
Clase 9 teorema de la maxima transferencia de potencia
Clase 9 teorema de la maxima transferencia de potenciaClase 9 teorema de la maxima transferencia de potencia
Clase 9 teorema de la maxima transferencia de potencia
 
Resistencia estática y dinamica de diodos
Resistencia estática y dinamica de diodosResistencia estática y dinamica de diodos
Resistencia estática y dinamica de diodos
 
Tema 4 -_ejercicios_resueltos
Tema 4 -_ejercicios_resueltosTema 4 -_ejercicios_resueltos
Tema 4 -_ejercicios_resueltos
 
Seaparat elect pote
Seaparat elect poteSeaparat elect pote
Seaparat elect pote
 
Problema nuestra del calculo de factor de rizo E1
Problema nuestra del calculo de factor de rizo E1Problema nuestra del calculo de factor de rizo E1
Problema nuestra del calculo de factor de rizo E1
 
Recortadores Serie Y Paralelo
Recortadores Serie Y ParaleloRecortadores Serie Y Paralelo
Recortadores Serie Y Paralelo
 
Maquinas de corriente continua (CC)
Maquinas de corriente continua (CC)Maquinas de corriente continua (CC)
Maquinas de corriente continua (CC)
 
ejercicios desarrollados de Lineas de transmision
ejercicios desarrollados de Lineas de transmisionejercicios desarrollados de Lineas de transmision
ejercicios desarrollados de Lineas de transmision
 
Redes de dos puertos
Redes de dos puertosRedes de dos puertos
Redes de dos puertos
 
Voltaje de rizado
Voltaje de rizadoVoltaje de rizado
Voltaje de rizado
 
Codigo De Colores Para Resistencia Electrica
Codigo De Colores Para Resistencia ElectricaCodigo De Colores Para Resistencia Electrica
Codigo De Colores Para Resistencia Electrica
 
Informe IEEE - conexio estrella - delta
Informe IEEE -   conexio estrella - deltaInforme IEEE -   conexio estrella - delta
Informe IEEE - conexio estrella - delta
 
practica circutos RL en serie y paralelo
practica circutos RL en serie y paralelopractica circutos RL en serie y paralelo
practica circutos RL en serie y paralelo
 
Rectificadores no controlados
Rectificadores no controladosRectificadores no controlados
Rectificadores no controlados
 

Destacado

Sistemas monofasicos
Sistemas monofasicos Sistemas monofasicos
Sistemas monofasicos Omar Gabaldon
 
Exercicios circuitos trifasicos 2010
Exercicios circuitos trifasicos 2010Exercicios circuitos trifasicos 2010
Exercicios circuitos trifasicos 2010AlcidesSousa
 
5 mediciones electricas
5 mediciones electricas5 mediciones electricas
5 mediciones electricassercastelli30
 
Puentes de medicion
Puentes de medicionPuentes de medicion
Puentes de medicionRuber Pachon
 

Destacado (6)

Sistemas monofasicos
Sistemas monofasicos Sistemas monofasicos
Sistemas monofasicos
 
Exercicios circuitos trifasicos 2010
Exercicios circuitos trifasicos 2010Exercicios circuitos trifasicos 2010
Exercicios circuitos trifasicos 2010
 
5 mediciones electricas
5 mediciones electricas5 mediciones electricas
5 mediciones electricas
 
Medicion potencia
Medicion potenciaMedicion potencia
Medicion potencia
 
Puentes de medicion
Puentes de medicionPuentes de medicion
Puentes de medicion
 
Circuitos trifasicos
Circuitos trifasicosCircuitos trifasicos
Circuitos trifasicos
 

Similar a Sistemas bifásicos y trifásicos LEIV

Sistemas de alimentación eléctrica
Sistemas de alimentación eléctricaSistemas de alimentación eléctrica
Sistemas de alimentación eléctricamikeimperio
 
Control de motores
Control de motoresControl de motores
Control de motoresAimee Aceves
 
sistemas electricos de potencia tesco .pdf
sistemas electricos de potencia tesco  .pdfsistemas electricos de potencia tesco  .pdf
sistemas electricos de potencia tesco .pdfBrianYaelSoriaChavez
 
Circuitos Trifasicos
Circuitos TrifasicosCircuitos Trifasicos
Circuitos Trifasicosmijari
 
3laboratorio electricos (gisela milagros%2c chuco garcia)
3laboratorio electricos (gisela milagros%2c chuco garcia)3laboratorio electricos (gisela milagros%2c chuco garcia)
3laboratorio electricos (gisela milagros%2c chuco garcia)Gisela Milagros
 
Sistemas TrifáSicos
Sistemas TrifáSicosSistemas TrifáSicos
Sistemas TrifáSicosguest5b41fb
 
CONSEPTOS GENERALES DE CONEXIONES TRIFASICOS
CONSEPTOS GENERALES DE CONEXIONES TRIFASICOSCONSEPTOS GENERALES DE CONEXIONES TRIFASICOS
CONSEPTOS GENERALES DE CONEXIONES TRIFASICOSpelucas9
 
Eligheor cohilasig2
Eligheor cohilasig2Eligheor cohilasig2
Eligheor cohilasig2nickjeorly
 
PRESENTACION DIPLOMADO OFICIAL.pptx
PRESENTACION DIPLOMADO OFICIAL.pptxPRESENTACION DIPLOMADO OFICIAL.pptx
PRESENTACION DIPLOMADO OFICIAL.pptxLuisIvanSalgueiro1
 

Similar a Sistemas bifásicos y trifásicos LEIV (20)

GonzalezDiegoAsig2
GonzalezDiegoAsig2GonzalezDiegoAsig2
GonzalezDiegoAsig2
 
Sistemas polifásicos
Sistemas polifásicosSistemas polifásicos
Sistemas polifásicos
 
Uth
UthUth
Uth
 
Investigacion
InvestigacionInvestigacion
Investigacion
 
Sistemas de alimentación eléctrica
Sistemas de alimentación eléctricaSistemas de alimentación eléctrica
Sistemas de alimentación eléctrica
 
E im 8_circuitos_trifasicos
E im 8_circuitos_trifasicosE im 8_circuitos_trifasicos
E im 8_circuitos_trifasicos
 
S11.s1-Material.pdf
S11.s1-Material.pdfS11.s1-Material.pdf
S11.s1-Material.pdf
 
Control de motores
Control de motoresControl de motores
Control de motores
 
sistemas electricos de potencia tesco .pdf
sistemas electricos de potencia tesco  .pdfsistemas electricos de potencia tesco  .pdf
sistemas electricos de potencia tesco .pdf
 
Circuitos Trifasicos
Circuitos TrifasicosCircuitos Trifasicos
Circuitos Trifasicos
 
Laboratorios 2-de-redes-presentar (1)
Laboratorios 2-de-redes-presentar (1)Laboratorios 2-de-redes-presentar (1)
Laboratorios 2-de-redes-presentar (1)
 
3laboratorio electricos (gisela milagros%2c chuco garcia)
3laboratorio electricos (gisela milagros%2c chuco garcia)3laboratorio electricos (gisela milagros%2c chuco garcia)
3laboratorio electricos (gisela milagros%2c chuco garcia)
 
Sistemas TrifáSicos
Sistemas TrifáSicosSistemas TrifáSicos
Sistemas TrifáSicos
 
Sistemas TrifáSicos
Sistemas TrifáSicosSistemas TrifáSicos
Sistemas TrifáSicos
 
Present.sistemas trifásicos
Present.sistemas trifásicosPresent.sistemas trifásicos
Present.sistemas trifásicos
 
CONSEPTOS GENERALES DE CONEXIONES TRIFASICOS
CONSEPTOS GENERALES DE CONEXIONES TRIFASICOSCONSEPTOS GENERALES DE CONEXIONES TRIFASICOS
CONSEPTOS GENERALES DE CONEXIONES TRIFASICOS
 
Fundamento Electrico
Fundamento ElectricoFundamento Electrico
Fundamento Electrico
 
Eligheor cohilasig2
Eligheor cohilasig2Eligheor cohilasig2
Eligheor cohilasig2
 
T11 motores ca
T11 motores caT11 motores ca
T11 motores ca
 
PRESENTACION DIPLOMADO OFICIAL.pptx
PRESENTACION DIPLOMADO OFICIAL.pptxPRESENTACION DIPLOMADO OFICIAL.pptx
PRESENTACION DIPLOMADO OFICIAL.pptx
 

Sistemas bifásicos y trifásicos LEIV

  • 1. ANÁLISIS DE CIRCUITOS BIFÁSICOS Y TRIFÁSICOS Luis E. Iparraguirre Vásquez R Estator N s t Rotor r T S S r ˆ E ˆ ER ˆ ES ˆ ET ωt ˆ E 2008
  • 2. Luis E. Iparraguirre Vásquez Profesor Asociado de la Escuela de Ingeniería Electrónica de la Universidad Privada Antenor Orrego Trujillo - Perú ANÁLISIS DE CIRCUITOS BIFÁSICOS Y TRIFÁSICOS Reservados todos los derechos Esta obra es propiedad intelectual del autor Prohibida su reproducción parcial o total por cualquier medio, sin permiso por escrito del autor Análisis de circuitos bifásicos y trifásicos 1 LEIV.
  • 3. INTRODUCCIÓN Para el consumo de grandes cantidades de energía eléctrica demanda generación, transmisión y distribución de la energía eléctrica, que se logra mediante los circuitos eléctricos trifásicos. El estudio de los circuitos polifásicos constituye un análisis exhaustivo de los sistemas bifásicos y trifásicos, balanceados o desbalanceados, que se estudian en el presente texto. La generación, transmisión y distribución de la energía eléctrica es el gran negocio de las grandes compañías eléctricas, utilizando diversos combustibles como el carbón, el gas natural, petróleo para generar energía eléctrica en centrales térmicas; o utilizar la energía potencial gravitatoria de grandes masas de agua y que por medio de turbinas acopladas con los ejes de los generadores conforman las centrales hidráulicas. El voltaje del generador en las centrales, se eleva mediante una transformador para la transmisión a grandes distancias, debido a que las centrales, generalmente no están ubicadas ceca de los centros de consumo, y también para tener un mínimo de pérdidas en las líneas de transmisión. La energía eléctrica se genera, transmite y distribuye en sistema trifásico, y sólo cuando esta cerca de los centros de consumo se reduce su tensión a los valores de consumo ( baja tensión), y también se cambia de trifásico a monofásico de acuerdo a lo solicitado por los usuarios. Las líneas de transmisión generalmente están conformadas por ternas, siendo soportadas por torres de madera o metálicos, debidamente aisladas, y en baja tensión puede estar conformada por tres hilos o cuatro hilos; uno de los cuales está conectado a tierra y es el de menor calibre. El caso ideal es que los sistemas polifásicos son diseñados para operar en estado balanceado o equilibrado, pero en condiciones normales de operación de los circuitos polifásicos, operan cerca del equilibrio, es decir que las corrientes de cada fase deben tener los mismos valores eficaces, para que todas las fases del sistema polifásico distribuyan su potencia aproximadamente iguales. La estructura básica de un sistema polifásico consiste en fuentes de voltaje conectadas a cargas a través de transformadores y líneas de transmisión. La omisión del transformador en el sistema polifásico, simplifica el análisis sin afectar la comprensión básica de los cálculos implicados. En el texto se describe el análisis de los circuitos bifásicos y trifásicos de corriente alterna, los que son conformados por dos y tres fases respectivamente, con tensiones de la misma frecuencia. Las corrientes alternas fueron adoptadas notablemente por Nikola Tesla (1856 – 1943), quien obtuvo diez patentes para motores de inducción de corriente alterna en 1895 Análisis de circuitos bifásicos y trifásicos 2 LEIV.
  • 4. 1 Hz, frecuencia muy alta 3 para el funcionamiento de los motores eléctricos. Gradualmente se estandarizó en Estados Unidos la frecuencia de 60 Hz, por el cual el ojo humano no puede detectar el parpadeo que efectúa la corriente alterna senoidal en una lámpara de incandescencia, pero si es notorio a frecuencias menores. Las frecuencias mayores tienen como efecto aumento de pérdida de energía en el circuito magnético de las máquinas eléctricas Los primeros alternadores de Westinghouse fueron de 133 Fundamentalmente la ventaja que presentan los circuitos trifásicos frente a los monofásicos es que a igualdad de potencia a transmitir y pérdidas en las líneas o conductores, las líneas trifásicas son mas económicas que los monofásicos, permitiendo un ahorro del 25 % en el peso de los conductores (líneas). Otra ventaja es que la potencia instantánea de un sistema trifásico balanceado es constante, independiente del tiempo, permitiendo que los motores trifásicos tengan un par uniforme, evitando vibraciones y esfuerzos en el rotor del motor. Los motores trifásicos pueden arrancar por sí mismos, sin embargo los motores monofásicos necesitan un dispositivo para conseguir el arranque. Luis E. Iparraguirre Vásquez Análisis de circuitos bifásicos y trifásicos 3 LEIV.
  • 5. CONTENIDO Sistemas Polifásicos Generador Bifásico Tensiones de fase Tensiones de líneas Generador trifásico Conexiones de los arrollamientos de fuerza en un generador trifásico Conexión estrella ( Y ) • Secuencia de fases directa o positiva • Secuencia de fases inversa o negativa Conexión en delta ( ∆ ) • Secuencia de fases directa o positiva • Secuencia de fases inversa o negativa Cargas trifásicas Cargas trifásicas balanceadas Cargas trifásicas desbalanceadas Circuitos trifásicos balanceados Generador trifásico en Y, con carga trifásica balanceada en Y en secuencia directa Tensiones de fase del generador Corrientes de líneas Potencias aparentes de fases Potencia aparente total trifásica Potencia activa total Potencia reactiva total Generador trifásico en ∆, con carga trifásica balanceada en ∆ en secuencia directa Tensiones de fase del generador Corrientes de fases Corrientes de líneas Potencias aparentes de fases Potencia aparente total Potencia activa total Potencia reactiva total Medida de la potencia activa total absorbida por una carga trifásica balanceada o desbalanceada Método de los dos vatímetros Medida de la potencia trifásica total por medio del método de los Dos vatímetros monofásicos para el sistema Y balanceado en secuencia directa Medida de la potencia trifásica total por medio del método de los dos vatímetros monofásicos para el sistema ∆ balanceado en secuencia directa Determinación del ángulo de la impedancia de la carga trifásica Balanceada en Y o ∆ Análisis de circuitos bifásicos y trifásicos 4 Pag. 6 6 10 10 11 13 13 16 17 18 19 19 19 19 20 20 20 20 21 22 23 23 23 25 26 26 27 28 28 29 29 31 32 33 36 39 LEIV.
  • 6. Pag. Sistema trifásico tetrafilar balanceado en Y en secuencia inversa o negativa 40 Corrientes de líneas 41 Potencias aparentes de fases 42 Potencia aparente total trifásica 43 Potencia activa total 43 Potencia reactiva total 44 Medida de la potencia trifásica total por medio del método de los dos vatímetros monofásicos para el sistema Y balanceado en secuencia inversa o negativa 46 Sistema trifásico en ∆ en secuencia inversa o negativa 48 Tensiones de fase del generador 48 Corrientes de fases 49 Corrientes de líneas 50 Potencias aparentes de fases 51 Potencia aparente total 52 Potencia activa total 53 Potencia reactiva total 53 Medida de la potencia activa mediante el método de dos vatímetros monofásicos 54 Medida de la potencia reactiva en un sistema Y o ∆ balanceado en secuencia directa 57 Vatímetro en cuadratura en el sistema Y o ∆ balanceado en Secuencia directa 57 Lectura del vatímetro monofásico en cuadratura para un sistema trifásico en Y en secuencia directa 58 Lectura del vatímetro monofásico en cuadratura para un sistema 59 trifásico en ∆ en secuencia directa Problemas resueltos de circuitos trifásicos balanceados 62 - 92 Problemas propuestos trifásicos balanceados 92 - 98 Circuitos trifásicos desbalanceados 99 Circuito trifásico trifilar en Y desbalanceado 99 Corrientes de mallas 99 Corrientes de líneas 99 Potencias aparentes de fases 100 Potencia aparente total 100 Circuito trifásico tetrafilar en Y desbalanceado 101 Corrientes de líneas 101 Corriente en el conductor neutro 101 Potencias aparentes de fases 101 Potencia aparente total 101 Circuito trifásico trifilar en ∆ desbalanceado 102 Corrientes de fases 102 Corrientes de líneas 102 Potencias aparentes de fases 103 Potencia aparente total 103 Problemas de circuitos trifásicos desbalanceados 104 - 137 Problemas propuestos desbalanceados 137 -140 Análisis de circuitos bifásicos y trifásicos 5 LEIV.
  • 7. SISTEMAS POLIFASICOS Un sistema polifásico es aquel que tiene dos o más sistemas monofásicos semejantes, en que los valores máximos de las ondas alternas senoidales no se producen al mismo tiempo. El sistema bifásico es aquel que tiene dos sistemas monofásicos sinusoidales con semejantes valores de amplitud y frecuencia, pero desfasadas entre si en 90 grados eléctricos El sistema trifásico es aquel que tiene tres sistemas monofásicos senoidales cuyas tensiones con los mismos valores de amplitud y frecuencia, se encuentran desfasadas entre si en 120 grados eléctricos En instalaciones especiales como los convertidores rotativos, servicios electrolíticos y otras es conveniente el uso de sistemas de seis, doce o más fases, que emplean tensiones desfasadas de 60, 30 etc., grados eléctricos Los equipos que se utilizan en los sistemas polifásicos presentan muchas ventajas con respecto a los monofásicos. Los generadores polifásicos son de mayores potencias y mas económicos, presentando mayores eficiencias. Los motores polifásicos y equipos asociados como los conmutadores, transformadores y elementos de control, son generalmente de altas potencias para ser usados industrialmente. En el sistema monofásico, los motores presentan un par electromagnético no uniforme, debido a que el flujo de potencia es pulsante, en donde la potencia se anula cuatro veces por ciclo y se hace negativa en dos intervalos de tiempo de cada ciclo, teniendo el motor así la velocidad variable en cada instante. Sin embargo los motores polifásicos operan con un par o torque electromagnético uniforme, absorbiendo potencia a un ritmo constante y sin inversiones. Estos motores polifásicos son más eficientes y económicos. GENERADOR BIFASICO En un generador bifásico se encuentran dos arrollamientos iguales e independientes o interconectados en el estator, de forma tal que quedan desplazadas en el espacio en 90º eléctricos, tal como se muestra en la Fig. 1. En el rotor para este caso de dos polos se encuentra el arrollamiento de excitación que es alimentado por una fuente de tensión continua, a través de un reóstato de campo, y que al girar el rotor a una velocidad angular ω, se inducen en las bobinas de fuerza ( bobinas del estator) tensiones con desfases de 90 grados eléctricos, tal como se muestra en la Fig.3 En la Fig. 2 se muestra un rotor de cuatro polos Análisis de circuitos bifásicos y trifásicos 6 LEIV.
  • 8. Fig. 2 Rotor de cuatro polos salientes Fig. 1 Generador Bifásico de dos polos ˆ E ˆ EA ˆ EB ωt α ˆ −E Fig. 3 Tensiones inducidas de un generador bifásico en secuencia AB Así: ˆ ˆ ˆ EA = EB = E …( 1 ) En la fase A se induce: ˆ e Aa ( t ) = E Sen ωt V …( 2 ) En la fase B se induce: ˆ e Bb ( t ) = E Sen ( ωt − 90 º ) V …( 3 ) Análisis de circuitos bifásicos y trifásicos 7 LEIV.
  • 9. El correspondiente diagrama fasorial de la Fig. 3 se muestra en la Fig. 4a y Fig. 4b a A b E Aa E Aa E Bb B E Bb Fig. 4a Fig. 4b Estas tensiones tienen una secuencia AB, es decir que e Aa ( t ) ocurre primero y después de 90º eléctricos ocurre e Bb ( t ) a esta secuencia de fases se le denomina secuencia positiva o secuencia directa. También puede darse la secuencia inversa BA o negativa, cuando ocurre primero e Bb ( t ) , y después de 90º eléctricos ocurre e Aa ( t ) , tal como se muestra en la Fig. 5 ˆ E ˆ EA ˆ EB ωt α ˆ −E Fig. 5 Tensiones inducidas de un generador bifásico en secuencia BA Siendo estas tensiones: ˆ e Bb ( t ) = E Sen ωt V ˆ e Aa ( t ) = E Sen (ωt − 90 º ) V (4) (5) El correspondiente diagrama fasorial de la Fig. 5 se grafica en las Fig. 6a y Fig. 6b Análisis de circuitos bifásicos y trifásicos 8 LEIV.
  • 10. E Bb E Bb E Aa E Aa Fig. 6a Fig. 6b Los primeros sistemas polifásicos eran bifásicos o tetrafásicos, siendo completamente desalojados por los sistemas trifásicos; pero en servicio subsisten cierta cantidad de sistemas bifásicos, especialmente en los casos de equipos originales de alta calidad o en los sistemas servomecánicos, donde las máquinas bifásicas tienen gran ventaja. Un sistema bifásico se caracteriza por un desfase de 90 grados eléctricos entre las tensiones; las que son generadas en un alternador de dos devanados independientes, separados en el espacio por el mismo desplazamiento angular que el desplazamiento de las tensiones en el tiempo En la Fig. 7(a) , (b), y (c) se muestra un sistema bifásico en secuencia AB o secuencia directa Las tensiones de fases son E AN , E BN En la tensión E AN , significa que el borne “A” es más positivo que el borne “N” Las tensiones de líneas son E AB , E BA En la tensión E AB , significa que el borne “A” es más positivo que el borne “B” En la tensión E BA , significa que el borne “B” es más positivo que el borne “A” N a A b E Aa E Aa = E AN N EA E AB E Bb = E BN E Bb B Fig. 7 EB B (a) (b) E BA (c) Siendo el valor eficaz de las tensiones de fase E f y el valor eficaz de las tensiones de línea E l , se tiene: Análisis de circuitos bifásicos y trifásicos 9 LEIV.
  • 11. Tensiones de fases Si: E AN = E f / 0 º V (referencia) en secuencia directa, entonces E BN = E f / − 90 º V Tensiones de líneas E AB = E AN − E BN = E f / 0 º − E f / − 90 º , E AB = 2 E f / 45º V y E BA = E BN − E AN = E f / − 90 º − E f / 0 º , E BA = 2 E f / − 135º V En un sistema bifásico El = 2 Ef E Aa E Aa El circuito de la Fig. 7(a) pertenece a un generador bifásico trifilar ( tres hilos). E Bb El circuito de la Fig. 8 pertenece a un generador E Bb bifásico tetrafilar (cuatro hilos) donde cada fase es utilizada independientemente como dos sistemas monofásicos. La Fig. 9 muestra un circuito eléctrico que constituye un generador tetrafásico pentafilar (4 fases - 5 hilos) del que se tiene: E Aa = E f / 0 º V E Bb 2 E Aa 2 Ef E / 0 º + f / − 90 º = 2 2 Ef = / − 45º V 2 E Ab = E Ab E Aa 2 E Bb 2 Análisis de circuitos bifásicos y trifásicos E BN = 10 E Bb = E f / − 90 º V 2 Ef / − 45º 2 Ef / − 90 º V 2 LEIV.
  • 12. La Fig. 10 que constituye un sistema tetrafásico tetrafilar ( 4 fases - 4 hilos ), del que se tiene: E Aa E QS = E Aa − E Bb − E Bb E QS = E f / 0 º − E f / − 90 º E Bb E QS = 2 E f / 45º V − E Aa GENERADOR TRIFASICO Un generador trifásico es aquel que tiene tres arrollamientos independientes, distribuidos en la periferia interna del estator, para los generadores de polos salientes; o los tres arrollamientos independientes distribuidos en la periferia externa del rotor, para los generadores de rotor cilíndrico. En la Fig. 11 se muestra un generador trifásico bipolar, de rotor de polos salientes. Fig. 11 Generador trifásico de dos polos salientes Análisis de circuitos bifásicos y trifásicos 11 LEIV.
  • 13. En estos tres arrollamientos independientes e iguales, se inducen tensiones alternas senoidales desfasadas entre si en 120 grados eléctricos, debido a que la distribución de los arrollamientos en la máquina rotativa, también están a 120 grados eléctricos Cada arrollamiento independiente tiene un par de bornes denotados con las letras R-r, S-s, y T-t, tal como se muestra en la Fig. 11. El estator y rotor está formado por chapas laminadas de acero u otras aleaciones siliciosas. Las tensiones inducidas en los arrollamientos de fuerza según Faraday, es debida a la acción de una estructura de excitación en corriente continua, la que genera un campo electromagnético y que al girar concéntricamente a una velocidad angular ωr , y las espiras de los arrollamientos de fuerza cortan líneas de flujo, generando las tensiones de las fases R, S, y T, tal como se muestra en la Fig. 12 ˆ E ˆ ER ˆ ES ˆ ET ωt ˆ E Fig. 12 Tensiones inducidas en secuencia RST, de un generador trifásico ˆ ˆ ˆ ˆ E R = ES = E T = E (6) ˆ e Rr ( t ) = E Sen ωt (7) ˆ e Ss ( t ) = E Sen (ωt − 120 º ) (8) ˆ e Tt ( t ) = E Sen ( ωt + 120 º ) Así (9) La secuencia de fases RST, STR, TRS, es denominada secuencia directa o positiva, y es la que hace girar a los motores trifásicos de inducción en el sentido horario. La secuencia de fases RTS, TSR, SRT, es denominada secuencia inversa o negativa, y hace girar a los motores de inducción en el sentido antihorario. Análisis de circuitos bifásicos y trifásicos 12 LEIV.
  • 14. En la Fig. 13 se muestra el diagrama fasorial de las tensiones inducidas en los arrollamientos de fuerza, tal que al girar a la velocidad angular ωr rad/s, se generan las tensiones de la Fig. 12 E Tt 12 0º 120º E Rr 0º 12 E Ss Fig 13 Una vez establecida la secuencias de fases, debe determinarse el orden en que deben conectarse los conductores de línea a la carga. Un motor de inducción trifásico alimentado en secuencia directa o positiva, gira en un sentido. Si se intercambian dos de los conductores cualesquiera, entonces se invierte la secuencia de fases a la secuencia inversa o negativa, y el motor de inducción invierte el sentido de giro. CONEXIONES DE LOS ARROLLAMIENTOS DE FUERZA EN UN GENERADOR TRIFÁSICO Si cada una de las tres fases independientes del generador trifásico de la Fig. 14, pueden actuar independientemente, se tendría un sistema trifásico exafilar T E Tt t s r R E Rr E Ss En la práctica los generadores trifásicos se pueden conectar en estrella (Y); o en triángulo o delta (∆) Conexión del generador en Estrella (Y). S Para realizar este tipo de conexión los tres bornes r, s, y t, se unen entre sí, formando un borne común denominado neutro, denotado con la letra mayúscula N, tal como se muestra en la Fig. 15 conformando un sistema trifásico trifilar ( 3 fases - 3 hilos ); y Fig. 16 conformando un sistema trifásico tetrafilar ( 3 fases - 4 hilos ); ambas en secuencia de fases directa o positiva Análisis de circuitos bifásicos y trifásicos 13 LEIV.
  • 15. T T T T E TN E TN t t N r s N r s R E RN E SN E SN R E RN N S S S S Al invertir la secuencia de fases a inversa o negativa, se tendría las Fig. 17 y Fig. 18 S S S S E SN E SN s t N N R r R E TN E RN E TN E RN N T T T Fig. 18.- Generador trifásico tetrafilar en secuencia inversa o negativa T T E TR E TN E RN N R E RS E SN E ST E TR o E RT S Análisis de circuitos bifásicos y trifásicos Del circuito de la Fig. 15 que corresponde a un generador trifásico trifilar en secuencia directa, y que por no tener el conductor neutro, sólo se puede obtener tensiones de líneas, que corresponden a las tensiones entre las líneas R y S, obteniendo la tensión E RS o E SR ; entre las líneas S y T, para obtener la tensión E ST o E TS ; y entre las líneas T y R, para obtener la tensión 14 LEIV.
  • 16. Del circuito de la Fig. 16 que corresponde a un generador trifásico tetrafilar en secuencia directa, se pueden obtener tanto las tensiones de líneas como las tensiones de fases. Una tensión de línea se obtiene entre dos líneas cualesquiera, y una tensión de fase se obtiene entre una línea y el conductor neutro. Las tensiones de fase tienen el mismo valor eficaz E RN = E SN = E TN = E f … (10) y se encuentran desfasadas entre ellas en 120 grados eléctricos También se determina que fasorialmente: E RN = − E NR …(11) E SN = − E NS …(12) E TN = − E NT …(13) De la Fig. 19, las tensiones de líneas son. E RS = E RN − E SN …(14) E ST = E SN − E TN …(15) E TR = E TN − E RN …(16) De la Fig. 19, se ha obtenido el triángulo SNR que es isósceles y que se muestra en la Fig. 20 y Fig. 21 E RN Ef N N R E RS R Ef E SN El S S Aplicando ley de senos en el triángulo SNR de la Fig. 21 se tiene: El sen 120 º = Ef E E , que al dar valores se tiene l = f ; relación de la que se 1 sen 30 º 3 2 2 determina que El = 3 Ef …(17). La ecuación (17) indica que el valor eficaz de la tensión de línea es igual a raíz de tres veces el valor eficaz de la tensión de fase Análisis de circuitos bifásicos y trifásicos 15 LEIV.
  • 17. Si la secuencia de fases es directa o positiva, según circuito de la Fig. 16, con tensiones de fases: …(18) E RN = E f / 0 º V E SN = E f / − 120 º V …(19) E TN = E f / 120 º V …(20) Reemplazando las ecuaciones del (18) al (20) en las ecuaciones del (14) al (16), se tiene las ecuaciones: E RS = E f / 0 º − E f / − 120 º = 3 E f / 30 º V …(21) E ST = E f / − 120 º − E f / 120 º = 3 E f / − 90 º V …(22) E TR = E f / 120 º − E f / 0 º = 3 E f / 150 º V …(23) En la Fig. 22 se muestra el diagrama fasorial de tensiones de fases y tensiones de líneas obtenidas de un generador trifásico tetrafilar en secuencia directa. Indicando también que los valores eficaces de tensiones de líneas son iguales a raíz de tres veces el valor eficaz de tensiones de fases − E RN 30º 120 º E TN E RS 30º E TR 30º 90º 90º 90º 30º − E SN 0º 12 E RN 30º E SN S 120º 30º E ST − E TN Fig. 22.- Diagrama fasorial de tensiones de fase y tensiones de líneas en secuencia directa de un generador en Y NOTA.- El ángulo formado entre una tensión de línea y una tensión de fase es de 30º eléctricos (ver Fig. 19 y Fig. 22) Análisis de circuitos bifásicos y trifásicos 16 LEIV.
  • 18. Si la secuencia de fases es inversa o negativa, según Fig. 18, con tensiones de fases: …(24) E RN = E f / 0 º V E TN = E f / − 120 º V …(25) E SN = E f / 120 º V Las tensiones de líneas son: …(26) E RT = E RN − E TN …(27) E TS = E TN − E SN …(28) E SR = E SN − E RN …(29) S E SR E SN S S N E SN N R E TN E TS N T 0º 12 E RN 30º E RT R E TN E RN T 120 º º 30 T Fig. 23.- Tensiones de fase y de línea de un generador trifásico tetrafilar en secuencia inversa o negativa Reemplazando las ecuaciones del (24) al (26) en las ecuaciones del (27) al (29), se tiene las ecuaciones: E RT = E f / 0 º − E f / − 120 º = 3 E f / 30 º V …(30) E TS = E f / − 120 º − E f / 120 º = 3 E f / − 90 º V …(31) E SR = E f / 120 º − E f / 0 º = 3 E f / 150 º V …(32) También se obtiene las tensiones de líneas E RS , E ST y E TR , desfasando las ecuaciones (32), (31) y (30) en 180 º eléctricos E RS = − E SR = 3 E f / − 30 º V …(33) E ST = − E TS = 3 E f / 90 º V …(34) E TR = − E RT = 3 E f / − 150 º V …(35) NOTA.- El desfase en 180º eléctricos de un fasor se obtiene permutando los subíndices correspondientes del fasor en referencia. Análisis de circuitos bifásicos y trifásicos 17 LEIV.
  • 19. E ST − E RN E RT E SN − E TN E SR E RN E TN E TR E RS E TS − E SN Conexión del generador en Delta (∆). Los tres arrollamientos del generador trifásico se conectan en configuración ∆, como se muestra en la Fig. 25. para secuencia positiva T T E Tt = E TR R 120º E Tt t s r R E Rr E Ss S E Rr = E RS 0º 12 E Ss = E ST S Fig. 25.- Generador en delta en secuencia de fases directa Análisis de circuitos bifásicos y trifásicos 12 0º Fig. 26.- Tensiones de fase y de línea de un generador trifásico trifilar en , en secuencia directa o positiva 18 LEIV.
  • 20. El borne “r” se conecta en cortocircuito con el borne “S”. De la misma forma el borne “s” se conecta en cortocircuito con el borne “T”; y el borne “t” se conecta con el borne “R” . El la Fig. 26 se muestra las tensiones de fase en secuencia positiva, y que según en la conexión ∆, las tensiones de fase son iguales a las tensiones de líneas. Ef = El …(36) En las Fig. 27 y Fig. 28 se muestra la conexión ∆, y las tensiones de fase iguales a las tensiones de líneas de un generador con tensiones en secuencia inversa o negativa E Ss = E SR S S E Ss 12 0º s t r R E Rr = E RT 0º 12 R E Rr E Tt E Tt = E TS T T Fig. 27.- Generador en delta en secuencia de fases inversa CARGAS TRIFÁSICAS Las cargas trifásicas son conectadas en estrella (Y), o en delta o triángulo (∆), y pueden ser balanceadas (equilibradas) o desbalanceadas (desequilibradas) Cargas trifásicas balanceadas.- Son Aquellas cargas constituidas por las tres impedancias de fases: Z f = Z / ϕº Ω …(37) exactamente iguales, como se muestran en las Fig. 29 y Fig. 30 R R Zf Zf Zf Zf Zf T T S Fig. 29. Carga trifásica balanceada en estrella Análisis de circuitos bifásicos y trifásicos Zf S Fig. 30. Carga trifásica balanceada en triángulo 19 LEIV.
  • 21. Cargas trifásicas desbalanceadas.- Son Aquellas cargas constituidas por las tres impedancias de fase desiguales, según se muestran en la Fig.31 y Fig. 32 R Z1 Z1 Z2 Z2 Z3 T S Z3 Fig. 32. Carga trifásica desbalanceada en triángulo Z1 ≠ Z 2 ≠ Z3 Z1 ≠ Z 2 ≠ Z 3 CIRCUITOS TRIFÁSICOS BALANCEADOS El sistema trifásico balanceado está conformado por un generador trifásico que generalmente tiene las tensiones de fases balanceadas, el cual alimenta cargas trifásicas balanceadas por medio de tres líneas (3 hilos) o cuatro líneas (4 hilos) GENERADOR TRIFÁSICO EN Y, CON CARGA BALANCEADA EN Y, EN SECUENCIA DIRECTA IT T TRIFÁSICA PT T E TN t N E SN s r Zf PR IR Zf R E RN Zf N S I R + IS + I T IS S Tensiones de fase del generador. Para la Fig. 33 mostrada, sea: E RN = E f / 0 º V ( referencia ) secuencia (+) E SN = E f / − 120 º V …(39) E TN = E f / 120 º V …(38) …(40) Análisis de circuitos bifásicos y trifásicos 20 LEIV.
  • 22. Los potenciales de los bornes “r”, “s” y “t” son los mismos, denotados por “N” (neutro). Los potenciales de los bornes “R” , “S” y “T” de las líneas y el potencial del neutro son los mismos tanto en el generador como en la carga. Por lo tanto las tensiones de fase del generador son también tensiones de fase de la carga. Corrientes de líneas Las corrientes de líneas I R , I S , I T , son denotadas con un subíndice, que corresponde al borne o índice de la línea por donde circula. Estas corrientes de líneas, son también corrientes de fase, tanto para el generador, como para la carga, debido a que circulan por las línea, por las fases del generador en “Y” y las fases de la carga en “Y”. Il = If …(41) IR = IS = IT = E RN Zf E SN Zf E TN Zf = E f / 0º E f = / − ϕº A Z / ϕº Zf …(42) = E f / − 120 º E f = / − 120 º − ϕº A Z / ϕº Zf …(43) = E f / 120 º E f = / 120 º − ϕº A Z / ϕº Zf …(44) De las ecuaciones (42), (43) y (44), se observa que para el sistema trifásico balanceado, los valores eficaces de las corrientes de líneas son de igual magnitud, es E decir: I R = I S = I T = I l = f …(45) Zf También se observa que el desfase entre estas corrientes de líneas es de 120º eléctricos Al sumar las tres corrientes de líneas, para determinar la corriente en el neutro se tiene …(46) es decir que por conductor neutro no circula que I R + IS + I T = 0 corriente (circuito abierto), por lo tanto se puede prescindir del conductor neutro. Pero el conductor neutro, une el neutro del generador con el neutro de la carga, lo que constituye un corto circuito. De lo estudiado se concluye: En el sistema trifásico balanceado, con generador en “Y” y carga en “Y”, las tres corrientes de líneas son corrientes de fases, con igual magnitud eficaz es decir E I R = IS = IT = Il = If = f …(47), y se encuentran desfasadas entre ellas en 120º Zf eléctricos; y cuya suma fasorial es cero Análisis de circuitos bifásicos y trifásicos 21 LEIV.
  • 23. En la Fig. 34 se observa el diagrama fasorial de tensiones de fases, tensiones de líneas y de corrientes de líneas del sistema trifásico tetrafilar balanceado en estudio, cuando la carga tiene factor de potencia en atraso E TS − E RN 30º E TR E TN E RS 30º IT 30º º 30 − E SN ϕº 30º ϕº 90º ϕº 30º IS E SN E RN IR ϕº > 0 30º E ST − E TN Fig. 34.- Diagrama fasorial de tensiones de fase, tensiones de línea y de corrientes de línea, de un sistema trifásico tetrafilar balanceado en secuencia (+), con factor de potencia en atraso Potencias aparentes de fases De la Fig. 33 se determina las potencias aparentes absorbidas por cada impedancia de fase: E2 Ef / ϕº = f / ϕº VA Zf Zf Fase R-N SRN = E RN I * = E f / 0 º R Fase S-N * SSN = E SN IS = E f / − 120 º E2 Ef / 120 º + ϕº = f / ϕº VA Zf Zf …(49) Fase T-N STN = E TN I * = E f / 120 º T E2 Ef / − 120 º + ϕº = f / ϕº VA Zf Zf …(50) Análisis de circuitos bifásicos y trifásicos 22 …(48) LEIV.
  • 24. De las ecuaciones (48), (49) y (50), se observa que las potencias absorbidas por cada E2 impedancia de fase, son exactamente iguales: SRN = SSN = STN = f / ϕº VA …(50) Zf Potencia aparente total trifásica La potencia aparente total entregada por la fuente es la suma de las tres potencias aparentes absorbidas por cada impedancia de fase. Luego: E2 Stot = SRN + SSN + STN = 3 x f / ϕº VA …(51) Zf La ecuación (50) puede escribirse: Stot = 3 E f De la ecuación (17), E f = El 3 Ef / ϕ º VA Zf …(52) …(53) Ef = Il …(54) Zf Reemplazando las ecuaciones (53) y (54) en la ecuación (52) se tiene: De la ecuación (45) Stot = 3 x El x I l / ϕº VA …(55) 3 Efectuando operaciones en el segundo miembro de la ecuación (55) tenemos: Stot = 3 E l I l / ϕ º VA …(56) El módulo de la potencia aparente total es: S tot = 3 E l I l VA …(57) Pero Stot = Ptot + J Q tot VA …(58) Potencia activa total De la ecuación (56) la potencia activa total trifásica es: Ptot = 3 E l I l cos ϕº vatios …(59) Potencia reactiva total De la ecuación (56) la potencia reactiva total trifásica es: Q tot = 3 E l I l senϕº var es …(60) Análisis de circuitos bifásicos y trifásicos 23 LEIV.
  • 25. La potencia aparente total absorbida por una carga trifásica balanceada en “Y” son funciones de los valores eficaces de la tensión y corriente de línea, como se analiza de la ecuación (57) La potencia activa total absorbida por una carga trifásica balanceada en “Y” son funciones de los valores eficaces de tensión y corriente de línea y del factor de potencia total (el factor de potencia total es igual al factor de potencia de cada impedancia de fase) como indica la ecuación (59) La potencia reactiva total absorbida por una carga trifásica balanceada en “Y” son funciones de los valores eficaces de tensión y corriente de línea y del seno del ángulo de la impedancia de fase, como indica la ecuación (60) Los valores eficaces de tensión de línea y corriente de línea no determinan como está conectado el generador ni la carga trifásica balanceada E TS − E RN 30º E TR 30º E RS E TN º 30 − E SN IT 30º 30º IR IS E RN 30º ϕº = 0 E SN 30º E ST − E TN Fig. 35.- Diagrama fasorial de tensiones de fase, tensiones de línea y de corrientes de línea, de un sistema trifásico tetrafilar balanceado en secuencia (+), con factor de potencia igual a uno Análisis de circuitos bifásicos y trifásicos 24 LEIV.
  • 26. E TS − E RN E RS E TN 30º E TR º 30 − E SN 30º IT ϕº 30º ϕº IR 30º E RN ϕº IS E SN ϕº < 0 30º 30º − E TN E ST Fig. 36.- Diagrama fasorial de tensiones de fase, tensiones de línea y de corrientes de línea, de un sistema trifásico tetrafilar balanceado en secuencia (+), con factor de potencia en adelanto GENERADOR TRIFÁSICO EN ∆, CON CARGA BALANCEADA EN ∆, EN SECUENCIA DIRECTA T T IT TRIFÁSICA PT I TR E Tt t s r Zf I ST IR R Zf PR R E Rr Zf E Ss S S Análisis de circuitos bifásicos y trifásicos I RS IS 25 LEIV.
  • 27. Tensiones de fase del generador. E RS = E f / 0 º V ( referencia ) secuencia (+) E ST = E f / − 120 º V …(62) E TR = E f / 120 º V …(61) …(63) El potencial de borne “r” es mismo que del borne “S”; el potencial del borne “s” es el mismo que del borne “T”; y el potencial del borne “t” es el mismo que el potencial del borne “R”, tal como se observa en la Fig.37, quedando el generador conectado en ∆, Los potenciales “R”, “S” y “T” son los mismos que de la carga en conexión ∆, luego a cada fase de la carga se a aplicado la tensión de línea, es decir que en conexión del generador en ∆, las tensiones de fase son también tensiones de líneas E f = E l , como indica la ecuación (36) Corrientes de fases Son aquellas corrientes que circulan por las fases del generador o fases de la carga. Las corrientes de fases se especifican con doble subíndice como por ejemplo de la Fig. 38 se dice: I RS : que circula por la fase de la carga, del borne “R” al borne “S” IST : que circula por la fase de la carga, del borne “S” al borne “T” I TR : que circula por la fase de la carga, del borne “T” al borne “R” Así también se puede determinar la corriente ISR que circula por la fase de la carga, del borne “S” al borne “R”. Es decir I RS = − I SR …(64) Cuando se evalúa las corrientes de fase del generador, generalmente es para determinar las corrientes permisibles que circulan por las bobinas de fuerza del generador, con el propósito de no deteriorar dichas fases generadoras de energía eléctrica. I RS = IST = I TR = E RS Zf E ST Zf E TR Zf = E f / 0º E f = / − ϕº A Z f / ϕº Z f = E f / − 120 º E f = / − 120 º − ϕº A Z f / ϕº Zf = E f / 120 º E f = / 120 º − ϕº A Z f / ϕº Zf I RS + IST + I TR = 0 Análisis de circuitos bifásicos y trifásicos …(65) …(66) …(67) …(68) 26 LEIV.
  • 28. Se observa que en el sistema balanceado en ∆ en secuencia positiva, las tres corrientes E de fases tienen el mismo valor eficaz, es decir: I RS = I ST = I TR = I f = f A …(69), Zf y se encuentran desfasadas entre ellas en 120’ eléctricos, cuya suma fasorial es cero. Corrientes de líneas Son las corrientes que circulan por las líneas que unen el generador con la carga. Estas corrientes circulan del generador hacia la carga, y se denotan con un solo subíndice correspondiente al borne de la línea en referencia. Así el circuito de la Fig. 37, las corrientes de líneas son: I R , I S e I T Aplicando 1ra Ley de Kirchhoff a los bornes “R”, “S” y “T” de la carga en ∆, se tiene: En el borne “R” I R = I RS − I TR …(70) En el borne “S” IS = I ST − I RS …(71) En el borne “T” I T = I TR − IST …(72) Reemplazando las corrientes de fases, en los segundos miembros en las ecuaciones (70), (71) y (72) se tiene: IR = Ef E E / − ϕº − f / 120 º − ϕº = 3 f / − 30 º − ϕº A Zf Zf Zf …(73) IS = Ef E E / − 120 º − ϕº − f / − ϕº = 3 f / − 150 º − ϕº A Zf Zf Zf …(74) IT = Ef E E / 120 º − ϕº − f / − 120 º − ϕº = 3 f / 90 º − ϕº A …(75) Zf Zf Zf Sumando los primeros y segundos miembros de las ecuaciones (70), (71) y (72) se tiene: I R + IS + I T = 0 A Según la expresión (69) y las magnitudes eficaces de las corrientes de líneas determinadas por los segundos miembros de las ecuaciones del (73) al (75), se concluye: I R = I S = I T = I l = 3 I f A …(76), y se encuentran desfasadas entre ellas en 120° eléctricos, siendo la suma fasorial igual a cero Análisis de circuitos bifásicos y trifásicos 27 LEIV.
  • 29. Para este circuito el diagrama fasorial de tensiones y corrientes, con factor de potencia en atraso se muestra en la Fig. 38. E TR E TS − IST I T (30 º − ϕº ) I TR ϕº 30 º 60 º IS 30º ϕº − I RS IST ϕº 30 º 90 º − ϕº E RS I RS − I TR IR ϕº > 0 E ST Fig. 38.- Diagrama fasorial de tensiones y corrientes de un sistema trifásico trifilar en , balanceado en secuencia (+), con factor de potencia en atraso Potencias aparentes de fases Determinando las potencias aparentes absorbidas por cada fase de la carga, se tiene: En la fase R-S SRS = E RS I * = E f / 0 º RS E2 Ef / ϕº = f / ϕº VA Zf Zf …(77) * En la fase S-T SST = E ST I ST = E f / − 120 º E2 Ef / 120 º + ϕº = f / ϕº VA Zf Zf En la fase T-R STR = E TR I * = E f / 120 º TR E2 Ef / − 120 º + ϕº = f / ϕº VA Zf Zf …(78) …(79) Se observa que en el sistema balanceado en ∆, las potencias aparentes por fase son E2 iguales, es decir SRS = SST = STR = Sf = f / ϕº VA …(80) Zf Potencia aparente total La potencia aparente total absorbida por la carga es igual a la suma fasorial de las tres potencias aparentes de cada fase Análisis de circuitos bifásicos y trifásicos 28 LEIV.
  • 30. Stot = SRS + SST + STR = 3 2 Ef Zf La ecuación (81) puede escribirse: Stot = 3 E f De la ecuación (36), E f = E l / ϕ º VA Ef / ϕ º VA Zf …(81) …(82) …(83) Ef = If …(84) Zf Reemplazando las ecuaciones (83) y (84) en la ecuación (82) se tiene: De la ecuación (69) Stot = 3 x E l x I f / ϕº VA …(85) Según la ecuación (76) se deduce: I f = Il A …(86) 3 Reemplazando (86) en (85) y efectuando operaciones en el segundo miembro de la ecuación (85) tenemos: Stot = 3 E l I l / ϕ º VA …(87) El módulo de la potencia aparente total es: S tot = 3 E l I l VA …(88) Pero Stot = Ptot + J Q tot VA …(89) Potencia activa total De la ecuación (56) la potencia activa total trifásica es: Ptot = 3 E l I l cos ϕº vatios …(90) Potencia reactiva total De la ecuación (56) la potencia reactiva total trifásica es: Q tot = 3 E l I l senϕº var es …(91) Análisis de circuitos bifásicos y trifásicos 29 LEIV.
  • 31. A continuación se muestran en la Fig. 39 y Fig. 40, los diagramas fasoriales de tensiones y corrientes de un sistema trifásico trifilar en ∆, con factor de potencia unitario y factor de potencia en adelanto respectivamente E TR E TS IT − IST I TR 30 º I RS 60 º IS E RS 30 º 30º − I TR IST IR − I RS ϕº = 0 E ST Fig. 39.- Diagrama fasorial de tensiones y corrientes de un sistema trifásico trifilar en , balanceado en secuencia (+), con factor de potencia unitario E TR E TS 60 º − IST IT ϕº 30 º − ϕº I TR 30 º I RS 30 º ϕº − I TR E RS IR IST IS − I RS ϕº ϕº < 0 E ST Análisis de circuitos bifásicos y trifásicos 30 LEIV.
  • 32. La potencia aparente total depende únicamente de los valores de tensión y corriente de línea, según se observa de la ecuación (88) La potencia activa total absorbida por una carga trifásica balanceada en “∆” en secuencia directa, son funciones de los valores eficaces de tensión y corriente de línea y del factor de potencia total (el factor de potencia total es igual al factor de potencia de cada impedancia de fase), como se observa de la ecuación (90) La potencia reactiva total absorbida por una carga trifásica balanceada en “∆” en secuencia directa, son funciones de los valores eficaces de tensión y corriente de línea y del seno del ángulo de la impedancia de fase, deducido de la ecuación (91) Los valores eficaces de tensión de línea y corriente de línea no determinan como está conectado el generador ni la carga trifásica balanceada MEDIDA DE LA POTENCIA ACTIVA TOTAL ABSORBIDA POR UNA CARGA TRIFÁSICA BALANCEADA, O DESBALANCEADA La medición de la potencia activa total trifásica en un sistema polifásico es posible, instalando tantos vatímetros monofásicos como fases tiene el sistema Para en sistema trifásico se utilizarían tres vatímetros monofásicos conectados en cada fase; de manera tal que cada vatímetro monofásico mida la potencia activa de fase, esto es que por cada bobina amperimétrica ingrese la corriente de fase y a la bobina voltimétrica de cada vatímetro monofásico aplicar la tensión de fase (es decir entre línea y neutro para la carga en estrella; o entre línea y línea para la carga en delta), tal como se muestra en la Fig. 41 y Fig. 42, para una carga en Y o en ∆ respectivamente R PZ 3 PT Z1 ≠ Z 2 ≠ Z3 IT Z3 T Z1 PZ Z2 IS PS IR Análisis de circuitos bifásicos y trifásicos Z1 Z3 2 Z2 PZ 1 S Z1 ≠ Z 2 ≠ Z3 PR Fig. 42. Medida de la potencia trifásica de una carga trifásica en triángulo, mediante tres vatímetros monofásicos 31 LEIV.
  • 33. El método de los tres vatímetros es adecuado para la medición de la potencia activa de un sistema trifásico desbalanceado, generalmente si las tensiones entregadas por el generador y el factor de potencia de la carga, varían constantemente, y la potencia total es la suma algebraica de las tres lecturas indicadas de cada vatímetro Así para la Fig. 41 la potencia total es: Ptot = PR + PS + PT Y para la Fig. 42 la potencia total es: Ptot = PZ + PZ + PZ 1 2 …(92) …(93) 3 Método de los dos vatímetros Para la medición de la potencia trifásica, el método de los dos vatímetros es el que comúnmente se utiliza. Para medir la potencia trifásica total por este método, la carga puede estar conectado en estrella ( Y ) o delta ( ∆ ), y puede ser balanceada o desbalanceada Los dos vatímetros se deben conectar adecuadamente dos líneas, tal como se muestra en la Fig. 43. Cada vatímetro se conecta en dos líneas cualesquiera, de manera tal que en cada bobina amperimétrica ingrese la corriente de línea; y a cada bobina voltimétrica se conecte la tensión de línea determinada por la línea donde se ha conectado el vatímetro y la otra línea donde no se ha conectado un vatímetro. Si bien los vatímetros individuales ya no registran la potencia absorbida por cualquier fase particular de la carga, la suma algebraica de las lecturas de los dos vatímetros es igual a la potencia activa total tomada por las carga trifásica, sin que importe las conexiones, tanto de generador como de la carga (estrella o delta). T R S IT IR PT C arg a trifásica en Y o ∆ PR balanceada o desbalanceada IS Análisis de circuitos bifásicos y trifásicos 32 LEIV.
  • 34. Medida de potencia trifásica total por medio del método de los dos vatímetros monofásicos para el sistema Y balanceado en secuencia directa Analizaremos el circuito eléctrico ya estudiado de la Fig. 33, con Generador en “Y” y carga en “Y”con su diagrama fasorial de tensiones y corrientes es el mostrado en la Fig. 34 IT T PT T E TN N s r Zf PR IR t Zf R E RN E SN Zf N S I R + IS + I T IS S E TS − E RN 30º E TR S 30º E TN E RS º 30 − E SN IT 30º ϕº 30º ϕº IS 90º ϕº 30º E RN IR E SN 30º − E TN E ST Fig. 34.- Diagrama fasorial de tensiones de fase, tensiones de línea y de corrientes de línea, de un sistema trifásico tetrafilar balanceado en secuencia (+), con factor de potencia en atraso Análisis de circuitos bifásicos y trifásicos 33 LEIV.
  • 35. Si : Si : Si : 0 º < ϕº < 90 º , el factor de potencia es en atraso 0 º = ϕº , el factor de potencia es unitario (máximo) − 90 º < ϕº < 0 º , el factor de potencia es en adelanto Para el circuito de la Fig. 33, las lecturas son: E RS IR …(94) E TS IT …(95) PR = E RS I R cos 〈 PT = E TS I T cos 〈 Siendo E RS = E TS = E l IR = IT = Il …(96) …(97) De la Fig. 34 el ángulo formado entre E RS e I R es: 〈 y el ángulo formado entre E TS e I T es: E TS IT 〈 E RS IR = 30 º + ϕ º = 30 º − ϕº …(98) …(99) Reemplazando las ecuaciones (96), (97) y (98) en la ecuación (94) se obtiene en valores de líneas PR = E l I l cos (30 º + ϕº ) vatios …(100) Reemplazando las ecuaciones (96), (97) y (99) en la ecuación (95) se obtiene en valores de líneas PT = E l I l cos (30 º − ϕº ) vatios …(101) Sumando los primeros miembros de las ecuaciones (100) y (101) se tiene: PR + PT = E l I l (cos (30 º + ϕ º ) + cos(30 º −ϕ º ) ) vatios …(102) Efectuando operaciones en el segundo miembro de la ecuación (102) se tiene: PR + PT = E l I l 2 cos 30 º cos ϕ º vatios siendo cos 30 º = …(103) 3 …(104), que reemplazado en (103) se tiene: 2 PR + PT = 3 E l I l cos ϕº vatios Análisis de circuitos bifásicos y trifásicos …(104) 34 LEIV.
  • 36. El segundo miembro de la expresión de la ecuación (104), es idéntica a la expresión del segundo miembro de la ecuación (59), e indica que los dos vatímetros monofásicos dan valores de lecturas, cuya suma es igual a la potencia activa total absorbida por la carga trifásica balanceada en Y La potencia total activa es función de los valores eficaces de tensión de línea, corriente de línea y del factor de potencia total (igual al factor de potencia de cada impedancia de fase) En secuencia (+), los vatímetros se han colocado en las líneas “R” y “T” obteniendo las lecturas: PR = E l I l cos (30 º + ϕº ) vatios …(100) PT = E l I l cos (30 º − ϕº ) vatios …(101) PR + PT = 3 E l I l cos ϕº vatios …(104) Graficando las ecuaciones (100) y (101) y (104) se obtiene la Fig. 44, de la que se deduce: P El Il Ptot El Il 3 PT El Il 3/2 75º − 75º − ϕº − 90 º − 60 º − 45º − 30 º − 15º 15º 30 º 45º 60º ϕº 90 º PR El Il Fig. 44.- Gráfica de PR , PT , Ptot en secuencia directa, de un sistema trifásico balanceado en Y 1º. Para factor de potencia total en atraso, en secuencia directa R S T R S T, se ha conectado dos vatímetros: * * a. Un vatímetro conectado en la línea R y el otro en la línea T R S T R S T , quedando la línea “S” centrada entre las dos líneas donde se han conectado los vatímetros Así: PT ( vatímetro que se encuentra a la derecha ) tiene mayor lectura que PR (vatímetro que se encuentra a la izquierda) es decir PT > PR Análisis de circuitos bifásicos y trifásicos 35 LEIV.
  • 37. b. Si se hubiera conectado los vatímetros en las líneas “S” y “T”, para factor de * * potencia en atraso y en secuencia directa R S T R S T , la lectura PS > PT c. Si se hubiera conectado los vatímetros en las líneas “R” y “S”, para factor de * * potencia en atraso y en secuencia directa R S T R S T , la lectura PR > PS 2º. Si el factor de potencia total es unitario, las lecturas son iguales. PR = PT 3º. Para factor de potencia total en adelanto, en secuencia (+), se conecta dos vatímetros: * * a. Con los vatímetros en las líneas “R” y “T” R S T R S T , vatímetro conectado en la línea “R” ( vatímetro que se encuentra a la izquierda) tiene mayor lectura que PT (vatímetro que se encuentra a la derecha) es decir PR > PT b. Si se hubiera conectado los vatímetros en las líneas “S” y “T”, para factor de * * potencia en adelanto y en secuencia directa R S T R S T , la lectura PT > PS c. Si se hubiera conectado los vatímetros en las líneas “R” y “S”, para factor de * * potencia en adelanto y en secuencia directa R S T R S T , la lectura PS > PR 4º. En las ecuaciones (100) y (101), al cambiar ϕº por − ϕ º , se permutan las lecturas Medida de potencia trifásica total por medio del método de los dos vatímetros monofásicos para el sistema ∆ balanceado en secuencia directa Analizaremos el circuito eléctrico ya estudiado de la Fig. 37, con Generador en “∆” y carga en “∆”cuyo diagrama fasorial de tensiones y corrientes es el mostrado en la Fig. 38 T T IT PT I TR E Tt t s r E Rr Zf E Ss S Zf I ST IR R Zf PR R S Análisis de circuitos bifásicos y trifásicos I RS IS 36 LEIV.
  • 38. E TS E TR − IST I T (30 º − ϕº ) I TR ϕº 30 º 60 º IS 30º ϕº − I RS IST ϕº 30 º 90 º − ϕº E RS I RS − I TR IR ϕº > 0 E ST Fig. 38.- Diagrama fasorial de tensiones y corrientes de un sistema trifásico trifilar en , balanceado en secuencia (+), con factor de potencia en atraso Si : Si : Si : 0 º < ϕº < 90 º , el factor de potencia es en atraso 0 º = ϕº , el factor de potencia es unitario (máximo) − 90 º < ϕº < 0 º , el factor de potencia es en adelanto Para el circuito eléctrico de la Fig. 33, las lecturas son: E RS IR …(105) E TS IT …(106) PR = E RS I R cos 〈 PT = E TS I T cos 〈 Siendo E RS = E TS = E l IR = IT = Il …(107) …(108) De la Fig. 38 el ángulo formado entre E RS e I R es: 〈 y el ángulo formado entre E TS e I T es: E TS IT 〈 E RS IR = 30 º − ϕº = 30 º + ϕ º …(109) …(110) Reemplazando las ecuaciones (107), (108) y (109) en la ecuación (105) se obtiene en valores de líneas PR = E l I l cos (30 º + ϕº ) vatios Análisis de circuitos bifásicos y trifásicos 37 …(111) LEIV.
  • 39. Reemplazando las ecuaciones (107), (108) y (110) en la ecuación (106) se obtiene en valores de líneas PT = E l I l cos (30 º − ϕº ) vatios …(112) Sumando los primeros miembros de las ecuaciones (111) y (112) se tiene: PR + PT = E l I l (cos (30 º + ϕ º ) + cos(30 º −ϕ º ) ) vatios …(113) Efectuando operaciones en el segundo miembro de la ecuación (113) se tiene: PR + PT = E l I l 2 cos 30 º cos ϕ º vatios siendo cos 30 º = …(114) 3 …(115), que reemplazado en (114) se tiene: 2 PR + PT = 3 E l I l cos ϕº vatios …(116) El segundo miembro de la ecuación (116), es idéntica al segundo miembro de la ecuación (90), e indica que los dos vatímetros monofásicos dan valores de lecturas, cuya suma es igual a la potencia activa total absorbida por la carga trifásica balanceada en ∆. La potencia total activa es función de los valores eficaces de tensión de línea, corriente de línea y del factor de potencia total (igual al factor de potencia de cada impedancia de fase) En secuencia (+), los vatímetros se han colocado en las líneas “R” y “T” obteniendo las lecturas: PR = E l I l cos (30 º + ϕº ) vatios …(111) PT = E l I l cos (30 º − ϕº ) vatios …(112) PR + PT = 3 E l I l cos ϕº vatios …(116) Los valores de potencia indicados por las ecuaciones (111) , (112) , y (116), son observados en la Fig. 45 (Exactamente igual a la Fig. 44) Análisis de circuitos bifásicos y trifásicos 38 LEIV.
  • 40. P El Il Ptot 3 El Il PT El Il 3/2 75º − 75º − ϕº − 90 º − 60 º − 45º − 30 º − 15º 15º 30 º 45º 60 º ϕº 90 º PR El Il Para el sistema Y o ∆ trifásico balanceado en secuencia directa: La ecuación (100) es idéntica a la ecuación (111) La ecuación (101) es idéntica a la ecuación (112) La ecuación (104) es idéntica a la ecuación (116) De lo que se concluye que la potencia indicada por cada vatímetro monofásico es independiente de la conexión del generador y de la conexión de la carga, y que la suma algebraica de las lecturas de cada vatímetro monofásico es la potencia activa total absorbida por la carga trifásica. La potencia activa total siempre positivo, sin embargo las lecturas de los vatímetros peden tomar valores positivos y negativos, dependiendo de la secuencia de fases y del factor de potencia de la carga trifásica balanceada. Determinación del ángulo de impedancia de la carga trifásica balanceada en Y o ∆ Las lecturas de cada vatímetro monofásico de un circuito trifásico balanceado dependen del factor de potencia, tal como las ecuaciones: PR = E l I l cos (30 º + ϕº ) vatios …(100) ó …(111) PT = E l I l cos (30 º − ϕº ) vatios …(101) ó …(112) es lógico que estas ecuaciones nos permitan determinar una fórmula que determine el ángulo de la impedancia de la carga trifásica balanceada. Obteniendo la diferencia de lecturas de los vatímetros monofásicos PT − PR = E l I l  cos(30 º − ϕ º ) − cos(30 º + ϕº )      Análisis de circuitos bifásicos y trifásicos 39 …(117) LEIV.
  • 41. Efectuando operaciones en el segundo miembro de la ecuación (117) se tiene: PT − PR = E l I l  2 sen 30 º senϕº      PT − PR = E l I l senϕº Pero siendo sen 30º = 1 / 2 …(118) PR + PT = 3 E l I l cos ϕº vatios …(104) ó …(116) Dividiendo miembro a miembro las ecuaciones (118) y (104) se tiene: PT − PR = PR + PT 3 E l I l senϕº 3 E l I l cos ϕ º  P − PR  de donde tgϕº = 3  T   PT + PR    …(119) …(120) En la aplicación de la ecuación (120) para evaluar tg ϕº es necesario aplicar los signos correctos a los valores de PT y PR SISTEMA TRIFASICO TETRAFILAR BALANCEADO EN Y, EN SECUENCIA INVERSA O NEGATIVA IS S S E SN s N t r Zf PR IR Zf R E RN Zf E TN N T T I R + IS + I T IT PT En la Fig. 46 se muestra el circuito eléctrico a estudiar, con los dos vatímetros monofásicos conectados en las líneas “R” y “T” Las tensiones de fase del generador en secuencia inversa son: Análisis de circuitos bifásicos y trifásicos 40 LEIV.
  • 42. E RN = E f / 0 º V …( 121 ) E SN = E f / 120 º V …(122) E TN = E f / − 120 º V …(123) ( referencia a 0º ) secuencia (-) Corrientes de líneas Las corrientes de líneas I R , IS , I T e son denotadas con un subíndice, que corresponde al borne o índice de la línea por donde circula. Estas corrientes de líneas, son también corrientes de fase, tanto para el generador, como para la carga, debido a que circulan por las línea, por las fases del generador en “Y” y las fases de la carga en “Y”. Il = If …(41) IR = IS = IT = E RN Zf E SN Zf E TN Zf = E f / 0º E f = / − ϕº A Z / ϕº Zf …(124) = E f / 120 º E f = / 120 º − ϕº A Z / ϕº Zf …(125) = E f / − 120 º E f = / − 120 º − ϕº A Z / ϕº Zf …(126) De las ecuaciones (124), (125) y (126), se observa que para el sistema trifásico balanceado, los valores eficaces de las corrientes de líneas son de igual magnitud, es E …(127) decir: I R = I S = I T = I l = I f = f Zf También se observa que el desfase entre las corrientes de líneas es de 120º eléctricos Al sumar las tres corrientes de líneas, para determinar la corriente en el neutro se tiene que I R + IS + I T = 0 …(128) es decir que por conductor neutro no circula corriente ( característica de un circuito abierto), por lo tanto se puede prescindir del conductor neutro. El conductor neutro, une el neutro del generador con el neutro de la carga, lo que constituye un corto circuito. De lo estudiado se concluye: En el sistema trifásico balanceado, con generador en “Y” y carga en “Y”, las tres corrientes de líneas son corrientes de fases, con igual magnitud eficaz es decir E I R = IS = IT = Il = I f = f …(127), y se encuentran desfasadas entre ellas en Zf 120º eléctricos; y cuya suma fasorial es cero Análisis de circuitos bifásicos y trifásicos 41 LEIV.
  • 43. En la Fig. 47 se observa el diagrama fasorial de tensiones de fases, tensiones de líneas y de corrientes de líneas del sistema trifásico tetrafilar balanceado en secuencia negativa o inversa, cuando la carga tiene factor de potencia en atraso − E TN E ST 30º E RT E SR E SN ϕº I S 30º ϕº > 0 IT ϕº 30º 90 ϕº E TR E RN 30º 30º º −ϕ º IR 30 − E SN º 30º 30º − E RN E RS E TN E TS Fig. 47.- Diagrama fasorial de tensiones de fase, tensiones de línea y de corrientes de línea de un sistema trifásico tetrafilar en secuencia (-), con factor de potencia en atraso Potencias aparentes de fases De la Fig. 47 se determina las potencias aparentes absorbidas por cada impedancia de fase: E2 Ef / ϕº = f / ϕº VA Zf Zf Fase R-N SRN = E RN I * = E f / 0 º R Fase S-N * SSN = E SN IS = E f / 120 º Fase T-N STN = E TN I * = E f / − 120 º T Análisis de circuitos bifásicos y trifásicos …(129) E2 Ef / − 120 º + ϕº = f / ϕº VA …(130) Zf Zf E2 Ef / 120 º + ϕº = f / ϕº VA …(131) Zf Zf 42 LEIV.
  • 44. De las ecuaciones (129), (130) y (131), se observa que las potencias absorbidas por cada impedancia de fase, son exactamente iguales: SRN = SSN = STN = 2 Ef Zf / ϕº VA …(132) Potencia aparente total trifásica La potencia aparente total entregada por la fuente es la suma de las tres potencias aparentes absorbidas por cada impedancia de fase. Luego: Stot = SRN + SSN + STN = 3 x 2 Ef / ϕº VA …(133) La ecuación (133) puede escribirse: Stot = 3 E f Ef / ϕ º VA Zf Para el sistema estrella, E f = El 3 Zf …(134) …(135) Ef = Il …(136) Zf Reemplazando las ecuaciones (135) y (136) en la ecuación (134) se tiene: De la ecuación (127) Stot = 3 x El x I l / ϕº VA …(137) 3 Efectuando operaciones en el segundo miembro de la ecuación (137) tenemos: Stot = 3 E l I l / ϕ º VA …(138) El módulo de la potencia aparente total es: S tot = 3 E l I l VA …(139) Pero Stot = Ptot + J Q tot VA …(140) Potencia activa total De la ecuación (140) la potencia activa total trifásica es: Ptot = 3 E l I l cos ϕº vatios …(141) Análisis de circuitos bifásicos y trifásicos 43 LEIV.
  • 45. Potencia reactiva total De la ecuación (138) la potencia reactiva total trifásica es: Q tot = 3 E l I l senϕº var es …(142) − E TN E ST E RT E SR E SN E RN E TR 30 − E SN º E TN ϕº = 0º E RS − E RN E TS La potencia aparente total absorbida por una carga trifásica balanceada en “Y” son funciones de los valores eficaces de la tensión y corriente de línea, como se analiza de la ecuación (139) La potencia activa total absorbida por una carga trifásica balanceada en “Y” son funciones de los valores eficaces de tensión y corriente de línea y del factor de potencia total (el factor de potencia total es igual al factor de potencia de cada impedancia de fase) como indica la ecuación (141) Análisis de circuitos bifásicos y trifásicos 44 LEIV.
  • 46. La potencia reactiva total absorbida por una carga trifásica balanceada en “Y” son funciones de los valores eficaces de tensión y corriente de línea y del seno del ángulo de la impedancia de fase, como indica la ecuación (142) Los valores eficaces de tensión de línea y corriente de línea no determinan como está conectado el generador ni la carga trifásica balanceada En las Fig. 48 y Fig. 49 se muestran los diagramas fasoriales de tensiones y corrientes en secuencia inversa, de los sistemas trifásicos tetrafilares en Y, con factor de potencia unitario y factor de potencia en adelanto respectivamente − E TN E ST E RT E SR E SN ϕº ϕº E RN ϕº E TR 30 − E SN º E TN ϕº < 0 º E RS − E RN E TS Análisis de circuitos bifásicos y trifásicos 45 LEIV.
  • 47. Medida de potencia trifásica total por medio del método de los dos vatímetros monofásicos para el sistema Y balanceado en secuencia inversa o negativa Según el circuito eléctrico de la Fig. 46, los vatímetros están conectados en las líneas R y T, luego las lecturas de los vatímetros serán: E RS IR …(143) E TS IT …(144) PR = E RS I R cos〈 PT = E TS I T cos〈 La tensiones E RS = E TS = E l Las corrientes I R = I T = I l …(145) …(146) − E TN 30º E ST E RT E SR E SN ϕº I S 30º ϕº > 0 IT 9 − E RN ϕ 0º − º IR 30 − E SN º 30º 30º ϕº 30º ϕº E TR E RN 30º 30º E RS E TN E TS Fig. 47.- Diagrama fasorial de tensiones de fase, tensiones de línea y de corrientes de línea de un sistema trifásico tetrafilar en secuencia (-), con factor de potencia en atraso Del diagrama de la Fig. 47 se obtienen los ángulos entre tensiones y corrientes, así Análisis de circuitos bifásicos y trifásicos 46 LEIV.
  • 48. 〈 E RS IR = 30 º − ϕº 〈 …(147) E TS IT = 30 º + ϕº …(148) Reemplazando las ecuaciones (145), (146) y (147) en la ecuación (143) se tiene: PR = E l I l cos(30 º − ϕº ) vatios …(149) Reemplazando las ecuaciones (145), (146) y (148) en la ecuación (144) se tiene PT = E l I l cos(30 º + ϕº ) vatios …(150) Sumando miembro a miembro las ecuaciones (149) y (150) se tiene: …(151) PR + PT = E l I l  cos(30º − ϕ º ) + cos(30 º + ϕº )  vatios     Efectuando operaciones en el segundo miembro de la ecuación (151) se tiene: PR + PT = 3 E l I l cos ϕ º vatios …(152) El segundo miembro de la expresión de la ecuación (152), es idéntica a la expresión del segundo miembro de la ecuación (141), e indica que los dos vatímetros monofásicos dan valores de lecturas, cuya suma es igual a la potencia activa total absorbida por la carga trifásica balanceada en Y La potencia total activa es función de los valores eficaces de tensión de línea, corriente de línea y del factor de potencia total (igual al factor de potencia de cada impedancia de fase) Graficando las potencias de las ecuaciones (149), (150) y (152) se obtiene la Fig. 50, de la que se deduce: Ptot El Il Ptot El Il 3 PR El Il 3/2 75º − 75º − ϕº − 90º − 60º − 45º − 30 º − 15º Análisis de circuitos bifásicos y trifásicos 15º 30 º 47 45º 60 º ϕº 90 º PT El Il LEIV.
  • 49. 1º. Para factor de potencia total en atraso, en secuencia inversa R T S R T S, se han conectado dos vatímetros: * * a. Un vatímetro conectado en la línea R y el otro en la línea T R T S R T S , quedando la línea “S” centrada entre las dos líneas donde se han conectado los vatímetros Así: para factor de potencia en atraso y en secuencia inversa, PR ( vatímetro que se encuentra a la derecha ) tiene mayor lectura que PT (vatímetro que se encuentra a la izquierda) es decir PR > PT b. Si se hubiera conectado los vatímetros en las líneas “S” y “T”, para factor de * * potencia en atraso y en secuencia inversa R T S R T S , la lectura PT > PS c. Si se hubiera conectado los vatímetros en las líneas “R” y “S”, para factor de * * potencia en atraso y en secuencia inversa R T S R T S , la lectura PS > PR 2º. Si el factor de potencia total es unitario, las lecturas son iguales. PR = PT 3º. Para factor de potencia total en adelanto, en secuencia (-), se conecta dos vatímetros: * * a. Con los vatímetros en las líneas “R” y “T” R T S R T S , vatímetro conectado en la línea “T” ( vatímetro que se encuentra a la izquierda) tiene mayor lectura que el vatímetro conectado en la línea R (vatímetro que se encuentra a la derecha) es decir PT > PR b. Si se hubiera conectado los vatímetros en las líneas “S” y “T”, para factor de * * potencia en adelanto y en secuencia inversa R T S R T S , la lectura PS > PT c. Si se hubiera conectado los vatímetros en las líneas “R” y “S”, para factor de * * potencia en adelanto y en secuencia inversa R T S R T S , la lectura PR > PS 4º. En las ecuaciones (149) y (150), al cambiar ϕº por − ϕ º , se permutan las lecturas SISTEMA TRIFÁSICO EN ∆ EN SECUENCIA INVERSA O NEGATIVA Tensiones de fase del generador. E RT = E f / 0 º V ( referencia ) secuencia (-) E TS = E f / − 120 º V …(154) E SR = E f / 120 º V …(153) …(155) Análisis de circuitos bifásicos y trifásicos 48 LEIV.
  • 50. S S IS ISR E Ss = E SR s t r E Tt = E TS Zf PR R I TS IR R Zf E Rr = E RT Zf I RT T PT IT T El potencial de borne “r” es mismo que del borne “T”; el potencial del borne “s” es el mismo que del borne “R”; y el potencial del borne “t” es el mismo que el potencial del borne “S”, tal como se observa en la Fig.51, quedando el generador conectado en ∆, en secuencia inversa Los potenciales “R”, “S” y “T” son los mismos que de la carga en conexión ∆, luego a cada fase de la carga se a aplicado la tensión de línea, es decir que en conexión del generador en ∆, las tensiones de fase son también tensiones de líneas E RS = E ST = E TR = E f = E l …(156) E RS = E f / − 60 º V …(157) E ST = E f / 60 º V …(158) E TR = E f / 180 º V …(159) Corrientes de fase Son aquellas corrientes que circulan por las fases del generador o fases de la carga. Las corrientes de fases se especifican con doble subíndice como por ejemplo de la Fig. 51 se dice: ISR : Corriente que circula por la fase de la carga, del borne “S” al borne “R” I TS : Corriente que circula por la fase de la carga, del borne “T” al borne “S” I RT : Corriente que circula por la fase de la carga, del borne “R” al borne “T” Así también se puede determinar la corriente I RS que circula por la fase de la carga, del borne “R” al borne “S”. Es decir I RS = − ISR …(160) Cuando se evalúa las corrientes de fase del generador, es generalmente para determinar las corrientes permisibles que circulan por las bobinas de fuerza del generador, con el propósito de no deteriorar dichas fases generadoras de energía eléctrica. Análisis de circuitos bifásicos y trifásicos 49 LEIV.
  • 51. ISR = E SR Zf E TS I TS = Zf I RT = E RT Zf = E f / 120 º E f = / 120 − ϕº A …(161) Z f / ϕº Zf = E f / − 120 º E f = / − 120 º − ϕº A Z f / ϕº Zf = E f / 0º E f = / − ϕº A Z f / ϕº Z f I RS + IST + I TR = 0 …(162) …(163) …(164) Se observa que en el sistema balanceado en ∆ en secuencia inversa, las tres corrientes de E fases tienen el mismo valor eficaz, es decir: I SR = I TS = I RT = I f = f A …(165), Zf y se encuentran desfasadas entre ellas en 120º eléctricos, cuya suma fasorial es cero. Corrientes de líneas Son las corrientes que circulan por las líneas que unen el generador con la carga. Estas corrientes circulan del generador hacia la carga, y se denotan con un solo subíndice correspondiente al borne de la línea en referencia. Así el circuito de la Fig. 51, las corrientes de líneas son: I R , IS e I T Aplicando 1ra Ley de Kirchhoff a los bornes “R”, “S” y “T” de la carga en ∆, se tiene: En el borne “R” I R = I RT − ISR …(166) En el borne “S” IS = ISR − I TS …(167) En el borne “T” I T = I TS − I RT …(168) Reemplazando las corrientes de fases, en los segundos miembros en las ecuaciones (166), (167) y (168) se tiene: IR = Ef E E / − ϕº − f / 120 º − ϕº = 3 f / − 30 º − ϕº A Zf Zf Zf IS = Ef E E / 120 º − ϕº − f / − 120 º − ϕº = 3 f / 90 º − ϕº A …(170) Zf Zf Zf Análisis de circuitos bifásicos y trifásicos 50 …(169) LEIV.
  • 52. IT = Ef E E / − 120 º −ϕ º − f / − ϕº = 3 f / − 150 º − ϕº A …(171) Zf Zf Zf Sumando miembro a miembro las ecuaciones (169), (170) I R + IS + I T = 0 A …(172) y (171) se tiene: Las magnitudes eficaces de las corrientes de líneas determinadas por los segundos miembros de las ecuaciones (169). (170) y (171), se concluye: I R = I S = I T = I l = 3 I f A …(173), y se encuentran desfasadas entre ellas en 120° eléctricos, siendo la suma fasorial igual a cero El diagrama fasorial de tensiones y corrientes con factor de potencia en atraso en secuencia inversa, se muestra en la Fig. 52. E ST E SR − I TS IS (30 º − ϕº ) ISR E RT ϕº 30 º 60 º IT − I RT I TS E RT ϕº 30 º ϕº 90 º − ϕº I RT − ISR IR ϕº > 0 E TS Potencias aparentes de fases Determinando las potencias aparentes absorbidas por cada fase de la carga, se tiene: * En la fase S-R SSR = E SR ISR = E f / 120 º E2 Ef / − 120 º + ϕº = f / ϕ º VA …(174) Zf Zf En la fase T-S STS = E TS I * = E f / − 120 º TS Análisis de circuitos bifásicos y trifásicos E2 Ef / 120 º + ϕº = f / ϕ º VA Zf Zf 51 …(175) LEIV.
  • 53. En la fase R-T SRT = E RT I * = E f / 0 º RT E2 Ef / ϕ º = f / ϕº VA Zf Zf …(176) Se observa que en el sistema balanceado en ∆ en secuencia inversa, las potencias aparentes por fase son iguales, es decir: E2 SSR = STS = SRT = Sf = f / ϕº VA …(177) Zf Se deja como ejercicio al lector para que demuestre: SSR = SRS = STS = SST = SRT = STR …(178) Potencia aparente total La potencia aparente total absorbida por la carga es igual a la suma fasorial de las tres potencias aparentes de cada fase Stot = SSR + STS + SRT = 3 2 Ef Zf / ϕ º VA La ecuación (179) puede escribirse: Stot = 3 E f …(179) Ef / ϕ º VA Zf En delta la tensión de fase es tensión de línea E f = E l …(180) …(181) Ef = I f …(182) Zf Reemplazando las ecuaciones (181) y (182) en la ecuación (180) se tiene: En delta la corriente de fase es Stot = 3 x E l x I f / ϕº VA …(183) Por estar la carga en ∆ según la ecuación (173) se deduce: I f = Il A …(184) 3 Reemplazando (184) en (183) y efectuando operaciones en el segundo miembro de la se obtiene: Stot = 3 E l I l / ϕ º VA …(185) El módulo de la potencia aparente total es: S tot = 3 E l I l VA …(186) Pero Stot = Ptot + J Q tot VA Análisis de circuitos bifásicos y trifásicos …(187) 52 LEIV.
  • 54. Potencia activa total De la ecuación (185) la potencia activa total trifásica es: Ptot = 3 E l I l cos ϕº vatios …(188) Potencia reactiva total De la ecuación (185) la potencia reactiva total trifásica es: Q tot = 3 E l I l senϕº var es …(189) E SR E ST − I TS ISR IS 30 º 30 º 60 º I RT E RT 30 º IT − ISR I TS IR − I RT ϕº = 0 E TS Ejercicio Dibujar el diagrama fasorial de tensiones y corrientes de un sistema trifásico trifilar en ∆, balanceado en secuencia inversa con factor de potencia en adelanto Análisis de circuitos bifásicos y trifásicos 53 LEIV.
  • 55. Medida de la potencia activa mediante el método de dos vatímetros monofásicos S S IS ISR E Ss = E SR s r t Zf PR R I TS IR R E Rr = E RT E Tt = E TS Zf Zf I RT T PT IT T Los vatímetros se han instalado en las líneas “R” y “T” según la Fig. 51 para la carga trifásica balanceada en delta, en secuencia inversa, luego las lecturas de los vatímetros serán: E RS IR …(190) E TS IT …(191) PR = E RS I R cos〈 PT = E TS I T cos〈 La tensiones de líneas: E RS = E TS = E l …(192) Las corrientes de líneas: I R = I T = I l …(193) Del diagrama de la Fig. 52 se obtienen los ángulos entre tensiones y corrientes, así 〈 E RS IR = 30 º − ϕº …(194) 〈 E TS IT = 30 º + ϕ º …(195) Reemplazando las ecuaciones (192), (193) y (194) en la ecuación (190) se tiene: PR = E l I l cos(30 º − ϕº ) vatios …(196) Reemplazando las ecuaciones (192), (193) y (195) en la ecuación (191) se tiene PT = E l I l cos(30 º + ϕº ) vatios Análisis de circuitos bifásicos y trifásicos …(197) 54 LEIV.
  • 56. E SR E ST − I TS IS (30 º − ϕº ) ISR E RT ϕº 30 º 60 º IT − I RT I TS E RT ϕº I RT 30 º ϕº 90 º − ϕº ϕº > 0 − ISR 30 º − ϕº IR E RS E TS Sumando miembro a miembro las ecuaciones (196) y (197) se tiene: PR + PT = E l I l  cos(30º − ϕ º ) + cos(30 º + ϕº )  vatios     …(198) Efectuando operaciones en el segundo miembro de la ecuación (198) se tiene: PR + PT = 3 E l I l cos ϕ º vatios …(199) El segundo miembro de la expresión de la ecuación (199), es idéntica a la expresión del segundo miembro de la ecuación (188), e indica que los dos vatímetros monofásicos dan valores de lecturas, cuya suma es igual a la potencia activa total absorbida por la carga trifásica balanceada en ∆ La potencia total activa es función de los valores eficaces de tensión de línea, corriente de línea y del factor de potencia total (igual al factor de potencia de cada impedancia de fase) Graficando las potencias de las ecuaciones (196), (197) y (199) se obtiene la Fig. 54, de la que se deduce: Análisis de circuitos bifásicos y trifásicos 55 LEIV.
  • 57. Ptot El Il Ptot El Il 3 PR El Il 3/2 75º − 75º − ϕº − 90 º − 60 º − 45º − 30 º − 15º 15º 30 º 45º ϕº 60 º 90 º PT El Il Fig. 54.- Gráfica de PR , PT , Ptot en secuencia inversa de un sistema trifásico balanceado en 1º. Para factor de potencia total en atraso, en secuencia inversa R T S R T S, se han conectado dos vatímetros: * * a. Un vatímetro conectado en la línea R y el otro en la línea T R T S R T S , quedando la línea “S” centrada entre las dos líneas donde se han conectado los vatímetros Así: para factor de potencia en atraso y en secuencia inversa, PR ( vatímetro que se encuentra a la derecha ) tiene mayor lectura que PT (vatímetro que se encuentra a la izquierda) es decir PR > PT b. Si se hubiera conectado los vatímetros en las líneas “S” y “T”, para factor de * * potencia en atraso y en secuencia inversa R T S R T S , la lectura PT > PS c. Si se hubiera conectado los vatímetros en las líneas “R” y “S”, para factor de * * potencia en atraso y en secuencia inversa R T S R T S , la lectura PS > PR 2º. Si el factor de potencia total es unitario, las lecturas son iguales. PR = PT 3º. Para factor de potencia total en adelanto, en secuencia (-), se conecta dos vatímetros: * * a. Con los vatímetros en las líneas “R” y “T” R T S R T S , vatímetro conectado en la línea “T” ( vatímetro que se encuentra a la izquierda) tiene mayor lectura que el vatímetro conectado en la línea R (vatímetro que se encuentra a la derecha) es decir PT > PR b. Si se hubiera conectado los vatímetros en las líneas “S” y “T”, para factor de * * potencia en adelanto y en secuencia inversa R T S R T S , la lectura PS > PT c. Si se hubiera conectado los vatímetros en las líneas “R” y “S”, para factor de * * potencia en adelanto y en secuencia inversa R T S R T S , la lectura PR > PS Análisis de circuitos bifásicos y trifásicos 56 LEIV.
  • 58. 4º. En las ecuaciones (196) y (197), al cambiar ϕº por − ϕ º , se permutan las lecturas NOTA.- La Fig. 50 que corresponde al circuito trifásico balanceado en Y en secuencia negativa, y Fig. 53 que corresponde al circuito trifásico balanceado en ∆ en secuencia negativa, ambas figuras son exactamente iguales. Las ecuaciones de potencia indicada por cada vatímetro PR y PT y la potencia total Ptot son las mismas; por lo que se deduce que el método de los dos vatímetros es válido para cualquier circuito trifásico, balanceado y desbalanceado, y en cualquier secuencia; debido que las lecturas de cada vatímetro dependen únicamente de los valores de líneas, y del cos(30º ± ϕº ). MEDIDA DE LA POTENCIA REACTIVA EN UN SISTEMA Y o ∆ BALANCEADO EN SECUENCIA DIRECTA En todo sistema triásico balanceado en ∆ o Y, la potencia reactiva total es medida mediante el método del vatímetro monofásico en cuadratura. Vatímetro en cuadratura en el sistema Y o ∆ en secuencia directa Es la conexión de un vatímetro monofásico en una línea en un sistema trifásico balanceado, mediante el cual se demostrará que su lectura es directamente proporcional a la potencia reactiva total, absorbida por una carga trifásica balanceada. Para la secuencia de fases (+) I Si el vatímetro monofásico se conectara en la línea R, E ←− * * , conectando el RSTRST vatímetro monofásico en la línea R, La corriente de línea I R ingresa por la marca de polaridad de la bobina de corriente; y a la bobina de tensión se aplica el voltaje E ST I E Si el vatímetro monofásico se conectara en la línea S, la corriente de * *←− RSTRST línea IS ingresa por la marca de polaridad de la bobina de corriente; y a la bobina de tensión se le aplica el voltaje E TR I E Si el vatímetro monofásico se conectara en la línea T, de línea I T la corriente * *←− RST RST ingresa por la marca de polaridad de la bobina de corriente; y a la bobina de tensión se le aplica el voltaje E RS Análisis de circuitos bifásicos y trifásicos 57 LEIV.
  • 59. Lectura del vatímetro monofásico en cuadratura para un sistema trifásico en “Y” en secuencia directa Conectando el vatímetro monofásico en la línea “R” cuyo circuito se muestra en la Fig. 55. IT T T E TN t N s r Zf ´ PR IR Zf R E RN E SN Zf N S I R + IS + I T IS S ´ la lectura del vatímetro monofásico es: PR = E ST I R cos〈 E ST IR …(200) Del diagrama fasorial correspondiente (Fig. 34) mostrado en la siguiente página, se tiene: …(201) (tensión de línea) E ST = E l …(202) (corriente de línea) I R = Il 〈 E ST IR = 90 º − ϕ º …(203) (ángulo entre E ST e I R ) Reemplazando las ecuaciones (201), (202) y (203) en la ecuación (200) tenemos: ´ PR = E l I l cos( 90 º − ϕº ) siendo cos(90 º − ϕº ) = senϕº …(204) …(205) ´ Reemplazando la ecuación (205) en la (204) se tiene: PR = E l I l senϕº Pero la potencia reactiva total es: …(206) Q tot = 3 E l I l senϕº var es …(207) Reemplazando la ecuación (206) en (207) se tiene: ´ Q tot = 3 PR var es …(208) Análisis de circuitos bifásicos y trifásicos 58 LEIV.
  • 60. ´ Luego PR = Q tot 3 …(209) E TS − E RN 30º E TR S 30º E TN E RS º 30 − E SN IT 30º ϕº 30º ϕº IS 90º ϕº 30º E RN IR E SN 30º E ST − E TN Fig. 34.- Diagrama fasorial de tensiones de fase, tensiones de línea y de corrientes de línea, de un sistema trifásico tetrafilar balanceado en secuencia (+), con factor de potencia en atraso Enunciado 1. La ecuación (209) indica que en un circuito trifásico balanceado en Y en secuencia directa, cualquiera sea su factor de potencia, un vatímetro monofásico en cuadratura indica que su lectura igual a la potencia reactiva total trifásica, entre raíz de tres. Lectura del vatímetro monofásico en cuadratura para un sistema trifásico en “∆” en secuencia directa Conectando el vatímetro monofásico en la línea “R” cuyo circuito se muestra en la Fig. 56 . Análisis de circuitos bifásicos y trifásicos 59 LEIV.
  • 61. T T IT I TR E Tt t ´ PR R r s Zf I ST IR R Zf E Rr Zf E Ss S S I RS IS ´ La lectura del vatímetros es PR = E ST I R cos〈 E ST IR …(210) Del diagrama fasorial correspondiente (Fig. 38) se tiene: E ST = E l …(211) (tensión de línea) I R = Il …(212) (corriente de línea) …(213) (ángulo entre E ST e I R ) 〈 E ST IR = 90 º − ϕ º Reemplazando las ecuaciones (211), (212) y (213) en la ecuación (210) tenemos: ´ PR = E l I l cos( 90 º − ϕº ) siendo cos(90 º − ϕº ) = senϕº …(214) …(215) ´ Reemplazando la ecuación (215) en la (214) Se tiene: PR = E l I l senϕº …(216) Pero la potencia reactiva total es Q tot = 3 E l I l senϕº var es …(217) reemplazando la ecuación (216) en (217) se tiene: ´ Q tot = 3 PR var es …(218) ´ Luego PR = Q tot 3 …(219) Análisis de circuitos bifásicos y trifásicos 60 LEIV.
  • 62. E TR E TS − IST I T (30 º − ϕº ) I TR ϕº 30 º 60 º IS 30º ϕº − I RS IST ϕº 30 º 90 º − ϕº E RS I RS − I TR IR E ST Fig. 38.- Diagrama fasorial de tensiones y corrientes de un sistema trifásico trifilar en , balanceado en secuencia (+), con factor de potencia en atraso Enunciado 2. La ecuación (219) muestra que en un circuito trifásico balanceado en ∆ en secuencia directa, cualquiera sea su factor de potencia, un vatímetro monofásico en cuadratura indica que su lectura igual a la potencia reactiva total trifásica, entre raíz de tres. De los enunciados 1 y 2 correspondientes a las ecuaciones (209) y (219) se deduce: En todo circuito trifásico balanceado en secuencia directa, en Y o ∆, un vatímetro conectado en cuadratura, indicará una lectura igual a la potencia reactiva trifásica total absorbida, entre raíz de tres. Las expresiones de las ecuaciones (206) y (216) es válida para todo circuito trifásico balanceado en secuencia directa, cualquiera sea el factor de potencia de la carga trifásica, debido a que la lectura depende de los valores de tensión de línea y corriente de línea y del seno del ángulo ϕº Análisis de circuitos bifásicos y trifásicos 61 LEIV.
  • 63. PROBLEMAS RESUELTOS DE CIRCUITOS TRIFÁSICOS BALANCEADOS Problema 1 Una carga trifásica balanceada conectada en estrella, tiene 16 Ω de resistencia y 12 Ω de reactancia inductiva en serie en cada fase, y es alimentada por una línea trifásica de 230 V. Determinar: a. El valor eficaz de la corriente de línea. b. La potencia activa total. Solución La carga equilibrada está conectada en estrella, luego las corrientes de fase y de línea son las mismas, y están desfasadas en 120 grados eléctricos. La impedancia de fase es: Z f ,Y = 16 + J 12 Ω Z f ,Y = 20 / 36.87 º Ω El valor eficaz de la tensión de línea es: Vℓ = 230 voltios El valor eficaz de la tensión de fase es: V 230 Vf = l = = 132.79 V 3 3 El valor eficaz de la corriente fase If = Il = Vf 132.79 = = 6.64 A Rpta. a. Z f ,Y 20 La potencia activa total trifásica es: 2 Ptot = 3I f R f R f = 16 Ω Ptot = 3 x 6.64 2 x 16 = 2116.30 vatios Rpta. b. Otra forma de calcular la potencia activa total es mediante la fórmula: Ptot = 3 E l I l Cos ϕ Ptot = 3 x 230 x 6.64 x 0.8 = 2116.15 vatios Rpta. b. Problema 2 Para el problema 1; si las tres impedancias se conectan en triángulo y si se colocan a través de los mismos voltajes de línea, determinar: a. El valor eficaz de las corrientes de líneas. b. El valor eficaz de las corrientes de fase. Análisis de circuitos bifásicos y trifásicos 62 LEIV.
  • 64. c. La potencia activa total. Solución La carga está conectada en delta, luego la impedancia por fase en delta es: Z f ,∆ = 16 + J12 Ω Z f ,∆ = 20 / 36.87º Ω Con la carga conectada en triángulo o delta, la tensión de línea es también la tensión de fase. Luego: E l = Ef El valor eficaz de la corriente de fase de la carga en delta es: If = Ef 230 = = 11.50 A. Rpta. b. Z f ,∆ 20 El valor eficaz de la corriente de línea es: Il = 3 If I l = 3 x 11.50 = 19.92 A Rpta. a. 2 La potencia activa total Ptot = 3Pf = 3I f R f Ptot = 3 x 11.50 2 x 16 = 6348 vatios Rpta. c. Otra forma de calcular la potencia es a través de la fórmula: Ptot = 3 E l I l Cos ϕ Cos ϕ = Cos 36.87 º = 0.8 Ptot = 3 x 230 x 19.92 x 0.8 = 6348.45 vatios Rpta. c. Problema 3 Dos cargas en paralelo están alimentadas por una línea trifásica a una tensión de 240 voltios, 60 Hz. Una de las impedancias de fase de la carga en delta es 12 / – 60º Ω. Y la otra impedancia de fase de la carga en estrella es 10 / 25º Ω. Determinar: a. La corriente de línea. b. El factor de potencia total. Solución Sea: Vl : Tensión de línea: Vl = 240 voltios f = 60 Hz. Z f ,∆ : Impedancia de fase de la carga trifásica equilibrada conectada en triángulo. Análisis de circuitos bifásicos y trifásicos 63 LEIV.
  • 65. Z f ,Y : Impedancia de fase de la carga trifásica equilibrada en estrella. Z f ,∆ = 12 / –60º Ω , Z f ,Y = 10 / 25º Ω Como Z f ,∆ y Z f .Y son cargas trifásicas equilibradas, el sistema trifásico total es equilibrado. Luego para la solución es conveniente desarrollar mediante el equivalente monofásico; para lo cual, se pasa la carga delta a estrella. ' Sea Z f ,Y = Z f ,∆ / 3 la impedancia por fase equivalente en estrella, de la carga delta ' I1 : Corriente en Z f ,Y E RN I 2 : Corriente en Z f ,Y I1 ' Zf , Y I2 Zf ,Y E RN : Tensión de fase a 0º (referencia) E RN = I1 = I2 = 240 / 0º V 3 E RN ' Z f ,Y E RN Z f ,Y = = E RN = 138.564 / 0º V 138.56 / 0º 4 / − 60º = 34.64 / 60º A 138.56 / 0º = 13.85 / − 25º A 1 0 / 25º a. La corriente de línea I R = I1 + I 2 I R = 34.64 / 60º + 13.85 / − 25º A I R = 29.87 + J 24.15 = 38.41 / 38.94º A Rpta. a. Otro método es determinar la impedancia por fase total en estrella Z f ,Y ,tot = ' Z f ,Y Z f ,Y ' Z f ,Y + Z f ,Y = 4 / − 60º x 10 / 25º = 3.607 / − 38.941º Ω 4 / − 60º + 10 / 25º La corriente de línea es: I R = 138.56 / 0º E RN = = 38.413/ 38.941º A OK. Z f ,Y ,tot 3.607 / − 38.941 b. El factor de potencia total es Cos -38.94º = 0.7777 Rpta.b. Análisis de circuitos bifásicos y trifásicos 64 LEIV.
  • 66. Problema 4 Un motor de inducción de 50 HP trifásico, conectado en estrella a una línea trifásica de 440 voltios y 60 Hz está trabajando al 80% de su potencia nominal. El motor tiene el factor de potencia de 0.76 y eficiencia de 72%. Determinar la capacidad por fase en una conexión en triángulo requerida a través de la línea para corregir el factor de potencia a 0.92 en atraso. Solución Motor: 50HP conectado en estrella. Eℓ = 440 voltios f = 60 Hz. η = 72% = 0.72 Cos ϕ m = 0.76 %PC = 0.8 La potencia aparente total del motor trifásico es: Sm = 50 x 746 x 0.8 / Cos −1 0.76 VA 0.76 x 0.72 S m = 54532.16 / 40.54º VA La potencia aparente del motor por fase es la tercera parte de la potencia aparente total Sf ,m = 18177.39 / 40.54º VA ϕ1 = ϕ m = 40.54º Sf ,m = 13813.95 + J 11814.92 VA = Pf ,m + J Q f ,m El circuito equivalente monofásico es: C f ,Y : Capacidad por fase en conexión en estrella, del banco 3φ de condensadores. Sf ,m ϕ 2 = Cos −1 0.92 Como Eℓ = 440 V, se tiene que E f = 440 V. 3 → ϕ 2 = 23.07 º Ef = 254.03 V. Así la capacidad por fase en conexión estrella C f ,Y para mejorar el factor de potencia es: C f ,Y = Pf ,m E RN 2 ω (Tg ϕ1 − tg ϕ 2 ) F , y que al reemplazar datos se tiene: Análisis de circuitos bifásicos y trifásicos 65 LEIV.
  • 67. C f ,Y = 13813.95 254.03 2 x 377 (Tg 40.54º − tg 23.07 º ) C f ,Y = 243.8 uF Siendo X Cf , ∆ la reactancia capacitiva por fase en conexión delta como: X Cf ,Y = X Cf ,∆ entonces 3 C f ,Y 3 de donde: C f ,∆ = 1 1 = 2 π f C f , y 2 π f C f ,∆ (3) → C f ,∆ = 243.8 = 81.27 uF 3 Rpta. Problema 5 Un generador trifásico de 200 voltios de tensión de línea, alimenta a una carga inductiva conectada en estrella, que absorbe 10 amperios a través de una línea aérea trifásica de 1 Ω de resistencia y 5 Ω de reactancia inductiva por conductor. Determinar la tensión de fase en bornes de la carga si: a. El factor de potencia en los terminales de la carga es 0.6 (-) b. El factor de potencia en terminales del generador es 0.6 (-) Solución Eℓ= 200 voltios a. Z f ,Y = Z f ,Y Z f ,Y : impedancia por fase de la carga en estrella / Cos −1 0.6 = / 53.13º Ω Z f ,Y Z f , Y = 0. 6 Z f , Y + J 0. 8 Z f , Y Ω R 2 Zl = 1 + J 5 2 IR Z f ,tot = (1 + 0.6 Z f ,Y ) + (5 + 0.8Z f ,Y ) Ω VZf ,Y Zf , Y Il = E RN = Ef Z f ,tot , Ef = El 200 = V 3 3 N 200 / 0º V (referencia a 0º ) 3 Luego: 200 / 3 10 = 2 (1 + 0.6 Z f ,Y ) + (5 + 0.8Z f ,Y ) Análisis de circuitos bifásicos y trifásicos 66 2 … (1) LEIV.
  • 68. Simplificando la ecuación (1) se tiene: 2 Z f ,Y + 9.2 Z f ,Y − 107.33 = 0 …(2) Resolviendo la ecuación (2) se tiene: Z f ,Y = 6.74 Ω así Z f ,Y = 6.74 / 53.13º Ω Tomando E RN = 115.47 / 0º V (referencia) Por divisor de tensión tenemos que la caída de tensión en la impedancia Z f ,Y es VZf ,Y = E RN Z f ,Y Z f ,Y + (1 + J5) Z f ,Y = R + J X L Reemplazando valores y efectuando operaciones se tiene: V Zf ,Y = 67.4 / − 11º V Rpta. a b. Factor de potencia 0.6 en bornes del generador. IR Zl = 1 + J 5 Tg 53.13º = E RN 5 + XL 1+ R luego: 1 + R = 3.75 + 0.75 XL Z f ,tot = (1 + R ) 2 + (5 + X L ) 2 Ω Z f ,tot = (3.75 + 0.75X L ) 2 + (5 + X L ) 2 Pero: I R = I f = I l = Por lo tanto: 10 = E RN , Z f ,tot ….(A) Ω E RN = 115.47 / 0º 115.47 (3.75 + 0.75 X L ) 2 + (5 + X L ) 2 V …. (B) Resolviendo la ecuación (B), se tiene: XL = 4.237 Ω Análisis de circuitos bifásicos y trifásicos 67 LEIV.
  • 69. Luego por la ecuación (A) se tiene que: R = 5.928 Ω La impedancia equivalente total por fase en bornes del generador es: Z f ,tot = 6.928 + J 9.237 = 11.547 / 53.13º Ω La corriente de línea I R es: IR = 115.47 / 0º E RN = = 10 / − 53.13º A Z f ,tot 11.547 / 53.13º Siendo la impedancia por fase Z f ,Y = R + J X L Ω , entonces: Z f ,Y = 5.928 + J 4.237 Ω La caída de tensión por fase en la impedancia es: V Zf ,Y = I Z = 10 / − 53.13º x 7.287 / 35.55º V Zf ,Y = 72.87 / − 17.57º V Rpta.b. Problema 6 Una carga trifásica balanceada inductiva absorbe 5 KW y 17.32 KVAR. Determinar las lecturas de los dos vatímetros conectados para medir la potencia total. Solución Sea la secuencia de fases positiva, con los vatímetros conectados en las líneas R y T, luego las lecturas de los vatímetros será: PR = E I Cos (30º + ϕ) ... (1) PT = E I Cos (30º - ϕ) ... (2) (1)/(2) da: PR Cos (30º + ϕ) = PT Cos (30º − ϕ) Empleando propiedades de las proporciones: Análisis de circuitos bifásicos y trifásicos 68 LEIV.
  • 70. PR + PT Cos (30º + ϕ) + Cos (30º − ϕ) = PR Cos (30º + ϕ) PR + PT 3 Cos ϕ = PR Cos (30º + ϕ) De (3) : PR = Ptot ϕ = Tg −1 ...(3) Cos (30º + ϕ) 3 Cosϕ ...(4) Q tot 17.32 = tg −1 = 73.9º Ptot 5 Reemplazando el valor de ϕ en (4) se tiene: PR = -2.5 KW Rpta. Rpta. Así PT = Ptot – PR = 5 – (-2.5) = 7.5KW Problema 7 Una carga balanceada conectada en delta, tiene como impedancia de fase un circuito serie de 12 Ω. de resistencia y 16 Ω. de reactancia capacitiva. Las tensiones de línea sonde 115 voltios, determinar las corrientes de línea y de fase. Solución La carga está conectada en delta. La impedancia de fase es: Z f ,∆ = 12 − J 16 ohms. = 20 / − 53.13º Ω La tensión de línea es: Eℓ = 115 voltios Luego la corriente de fase es: If = Eℓ / Zf,∆ If = 115 = 5.75 A 20 Rpta. la corriente de línea es: I l = 3 I f I l = 3 x 5.75 = 9.96 amp. Análisis de circuitos bifásicos y trifásicos Rpta. 69 LEIV.
  • 71. Problema 8 Una carga balanceada en delta tiene 18 Ω. de resistencia y 24 Ω de reactancia capacitiva en serie en cada fase, siendo alimentada por líneas con impedancia cada una de 1 Ω de reactancia y 2 Ω. de reactancia inductiva por conductor. El generador que alimenta al circuito total tiene tensión entre líneas de 250 voltios. determinar. Solución Carga trifásica conectada en delta, con impedancia por fase: Z f ,∆ = 18 − J 24 = 30 / − 53.13º Ω La impedancia de línea Z l = 1 + J 2 Ω. Tensión de línea proporcionada por el generador es: Eℓ = 250 voltios. Como el circuito total es balanceado, se resuelve con un equivalente monofásico. E f = E RN = Sea: 250 = 144.34 V 3 E RN = 144.34 / 0º Zl = 1 + J 2 V (referencia a 0º) E RN Zf , Y La impedancia por fase en conexión estrella: Z f ,Y = Z f ,∆ = 10 / − 53.13º 3 Ω la impedancia equivalente total por fase es: Z f ,tot ,Y = 1 + J 2 + 10 / − 53.13º Ω Z f ,tot ,Y = 1 + J 2 + 6 − J 8 Ω Z f ,tot ,Y = 7 − J 6 = 9.22 / − 40.6º Ω La corriente de línea I R es: IR = E RN Z f ,tot = 144.34 / 0º A 9.22 / − 40.6º I R = 15.66 / 40.6º A Análisis de circuitos bifásicos y trifásicos 70 LEIV.
  • 72. La caída de tensión por fase en la carga en conexión estrella es: V f ,Y = I Z f ,Y = 15.66 / 40.6º x 10 / − 53.13º V V f ,Y = 156.6 / − 12.53º V a. El voltaje entre líneas en los terminales de carga es: Vl = 3 Vf ,Y = 3 x 156.6 = 271.24 V Rpta. a b. La potencia total consumida por la carga es: Ptot = 3 I 2 R f = 3 x 15.66 2 x 6 = 4414.24 vatios Rpta. b Problema 9 Una carga inductiva balanceada conectada en estrella toma 5.4KW a 0.6 de factor de potencia, a 200 voltios de tensión de líneas. Esta carga se encuentra conectada en paralelo con otra carga en estrella balanceada, puramente resistiva, la que toma 5 KW. Determinar la corriente total de línea suministrada a las dos cargas. Solución La primera carga trifásica en estrella, que toma una potencia de 5.4KW a Cos ϕ1 = 0.6 Luego ϕ1 = Cos −1 0.6 = 53.13º , así: P1 = 5.4 KW Q1 = P1 tg ϕ1 = 5.4 tg 53.13 = 7.2 KVAR La potencia aparente total que toma la primera carga trifásica En estrella es: S1 = 5.4 + J 7.2 = 9 / 53.13º KVA La segunda carga trifásica en estrella toma una potencia aparente S 2 = 5 / 0º KVA Así las dos cargas trifásicas en conjunto (ambas conectadas en paralelo) absorben una potencia aparente total: S tot = S1 + S 2 KVA Análisis de circuitos bifásicos y trifásicos 71 LEIV.
  • 73. S tot = 5.4 + J 7.2 + 5 = 10.4 + J 7.2 KVA S tot = 12.65 / 34.7 º KVA Pero S tot = 3 E l I l . Luego: Il = S tot Il = 3 El , siendo E l = 200 V 12,650 = 36.52 A Rpta. 3 x 200 Problema 10 Una conexión delta balanceada tiene una impedancia de fase de 12 / 70º Ω y es alimentada por una línea trifásica de 240 voltios, 60 Hz. Se mide la potencia del circuito por el método de dos vatímetros. Determinar la lectura indicada por cada vatímetro y la potencia trifásica total Solución: Carga balanceada en delta con impedancia de fase: Z ∆ ,f = 12 / 70º Ω , ϕ = 70º (factor de potencia en atraso) Tensión de línea: Ef = Eℓ = 240 voltios, 60 Hz (Por tener la carga en delta) Sea la secuencia de fase positiva, con los vatímetros conectados en las líneas S y T, R S T R S T, teniendo la carga factor de potencia en atraso, se tiene: PS > PT La corriente de fase es: If = E Ef 240 = l = = 20 A Z ∆ ,f Z ∆ ,f 12 La corriente de línea es: I l = 3 I f = 3 x 20 = 34.64 A La potencia indicada por cada vatímetro es: PS = E l I l Cos (30º − ϕ) vatios, y que al reemplazar valores se tiene : PS = 240 x 34.64 x Cos (30º − 70º ) = 6368.59 vatios Rpta. PT = E l I l Cos (30º + ϕ) vatios, y que al reemplazar valores se tiene : PS = 240 x 34.64 x Cos (30º + 70º ) = − 1443.60 vatios Rpta. Análisis de circuitos bifásicos y trifásicos 72 LEIV.
  • 74. La potencia total es: Ptot = PS + PT Ptot = 6368.59 + (-1443.68) = 4924.91 vatios Rpta Problema 11 Una delta balanceada toma 14.5 / 30º KVA de una línea trifásica, con tensión E RS = 440 / 0º V a 60 Hz y en secuencia negativa. Determinar: a. La impedancia de fase de la carga en delta Z ∆ ,f b. Las corrientes de fase en forma polar. c. Las corrientes de líneas en forma rectangular. d. La bancada trifásica de condensadores conectados en estrella, para corregir el factor de potencia total a 0.93 en atraso. Solución Graficando la delta en secuencia negativa de forma tal que la tensión ERS se esté a cero grados. S R N Así en el triángulo graficado, las tensiones de fase y de líneas indican sus ángulos de fases. Del diagrama fasorial de tensiones de fase y de líneas quedan definidas en módulo y ángulo, tal como se muestra en la figura. T EST El = 440 voltios f = 60 Hz. La potencia aparente total absorbida por la carga trifásica en delta es S tot = 14500 / 30º KVA ESN E RN 30 º 30º E RS 30 º Resolviendo mediante el equivalente 1 φ en bornes R-N, se tiene: E TN E RN E = = 254.03 / 30º V 3 E TR Sea Z Y ,f la impedancia por fase de la carga en conexión estrella. Para el equivalente monofásico, la potencia aparente por fase tomada de la línea es la tercera parte de la potencia aparente total, así: Análisis de circuitos bifásicos y trifásicos 73 LEIV.
  • 75. S Y ,f = Pero: S tot 14500 = = 4833.33 / 30º VA 3 3 IR S Y ,f = E RN I R E RN * Luego: I R = S Y ,f / E RN IR = Zf ,Y 4833.33 / 30º = 19.03 / 0º A 254.03 / 30º Z Y ,f = E RN IR = 254.03 / 30º = 13.35 / 30º Ω 19.03 / 0º La impedancia por fase en conexión en estrella es: 254.03 / 30º E RN Z Y ,f = = = 13.35 / 30º Ω. 19.03 / 0º IR a. La impedancia por fase en conexión delta será: Z ∆ ,f = 3 Z Y ,f = 3 x 13.35 / 30º = 40.05 / 30º Ω Rpta. b. Corrientes de fase en módulo y ángulo E RS = 440 / 0º V E ST = 440 / 120º V E TR = 440 / − 120º V I RS = I ST = I TR = E RS Z ∆ ,f E ST Z ∆ ,f E TR Z ∆ ,f = 440 / 0º = 10.99 / − 30º = 9.52 − J 5.50 A Rpta. 40.05 / 30º = 440 / 120º = 10.99 / − 90º = J 10.99 A Rpta. 40.05 / 30º = 440 / − 12 0º 40.05 / 30º Análisis de circuitos bifásicos y trifásicos = 10.99 / − 150º = − 9.52 − J 5.50 A Rpta. 74 LEIV.
  • 76. c. Las corrientes de línea en forma rectangular: I R = I RS − I TR I R = 10.99 / − 30º −10.99 / − 150º IR I R = 19.03 A Rpta. IS I S = I ST − I RS I S = − 9.52 + J 16.49 Rpta. A IST Z ∆, I S = J 10.99 − (9.52 − J 5.50) = IRS Z ∆ ,f f IT Z∆ ,f ITR I T = I TR − I ST I T = − 9.52 − J 5.50 − J 10.99 I T = − 9.52 − J 16.49 A Rpta d. Bancada trifásica de condensadores en estrella para obtener el factor de potencia total a 0.93 en atraso. Del circuito equivalente monofásico, la potencia aparente por fase es: S Y ,f = 4833.33 / 30º = 4185.79 + J 2416.64 VA La potencia activa que toma cada fase es: PY,f = 4185.79 vatios. En condiciones iniciales ϕ1 = 30º En condiciones finales ϕ2 = Cos −1 0.93 = 21.57 º Luego: C Y ,f = PY ,f 2 Ef V C Y ,f = ( tg ϕ1 − tg ϕ 2 ) F. 4185.79 254.03 2 x 377 C Y ,f = 31.32uF ( tg 30º − tg 21.57 º ) Rpta. Problema 12 Una delta balanceada absorbe 14.4 /-68º KVA de una línea trifásica de 600 voltios a 60 Hz. Determinar los vatios y vares, cuando las impedancias de fase se conectan en estrella a una línea de 440 voltios y 25 Hz. Solución: Inicialmente la carga está conectada en delta, la cual absorbe en total 14400 /-68º VA; a la tensión de línea de 600 voltios, 60 Hz. Análisis de circuitos bifásicos y trifásicos 75 LEIV.
  • 77. E RN = Sea 600 / 0º = 346.41 / 0º V 3 Así el primer equivalente monofásico es: I R1 E RN Z Y ,f : Es la impedancia equivalente por fase en conexión estrella, de la carga balanceada en delta. ZY ,f Sea S Y ,f : potencia aparente que absorbe Z Y ,f S Y ,f = 14400 / − 68º VA = 4800 / − 68º VA 3 Pero: S Y ,f = E RN I * 1 R * I R1 = * de donde I R1 = S Y ,f E RN 4800 / − 68º = 13.86 / − 68º A 346.41 / 0º Así: I R1 = 13.86 / 68º A Por lo tanto: Z Y ,f = E RN 346.41 / 0º = = 25 / − 68º Ω I R1 13.86 / 68º Luego, la impedancia de fase de la carga en conexión en delta es: Z ∆ ,f = 3 Z Y,f = 75 / − 68º = 28.1 − J 69.54 Ω a 60 Hz La disminución de frecuencia de 60 Hz a 25 Hz, tiene el efecto de aumentar la reactancia capacitiva, por ser inversamente proporcional a la frecuencia. Luego la impedancia de fase a 25 Hz en conexión delta es: 60 Ω a 25 Hz 25 ' Z ∆ ,f = 28.1 − J 69.54 x ' Z ∆,f = 28.1 − J 166.89 = 169.24 / − 80.44º Ω a 25 Hz ' Las impedancias de fase Z ∆,f se conectan en estrella, a la tensión de línea de 440 voltios a 25 Hz. El segundo equivalente monofásico es: Análisis de circuitos bifásicos y trifásicos 76 LEIV.